Текст
                    Н. X. Агаханов П. А. Кожевников Д. А. Терешин
МАТЕМАТИКА
МЕЖДУНАРОДНЫЕ
ОЛИМПИАДЫ
ПРОСВЕЩЕНИЕ
ИЗДАТЕЛЬСТВО


о л Н. X. Агаханов П. А. Кожевников Д. А. Терешин МАТЕМАТИКА МЕЖДУНАРОДНЫЕ ОЛИМПИАДЫ Москва «Просвещение» 2010
УДК 372.8:51 ББК 74.262.21 А23 Серия «Пять колец» основана в 2007 г. Руководители проекта серии «Пять колец» С.И.Демидова, И.И.Колисниченко Агаханов Н. X. А23 Математика. Международные олимпиады / Н. X. Агаханов, П. А. Кожевников, Д. А. Терешин. — М. : Просвещение, 2010. — 127 с. : ил. — (Пять колец). — ISBN 978-5-09-019788-5. Книга содержит описание истории Международных математических олимпиад, особенности их проведения и результаты выступления команды России за 1992—2008 гг. В книге приведены задания олимпиад (1997—2008 гг.), а также ответы, решения и указания ко всем заданиям. Материал книги окажет помощь при подготовке учащихся к математическим соревнованиям высокого уровня. УДК 372.8:51 ББК 74.262.21 ISBN 978-5-09-019788-5 © Издательство «Просвещение», 2010 /^^ч © Художественное оформление. //(fVflp4\\ Издательство «Просвещение», 2010 Ц >1# )} Все права защищены
ВВЕДЕНИЕ Развитие международного сотрудничества в различных областях, начавшееся во второй половине XX в., а также широкое распространение в послевоенный период в ряде стран математических олимпиад привели к идее проведения международных соревнований школьников по математике, а затем и по другим дисциплинам естественно-математического цикла. Летом 1959 г. по инициативе Румынского математического и физического общества была проведена I Международная математическая олимпиада (ММО). С тех пор стало традицией ежегодно летом проводить Международную математическую олимпиаду школьников. Олимпиада непрерывно развивается, и участие в ней в последние годы принимают большинство стран Европы, Азии, Америки, Австралии и Океании, а также некоторые африканские страны. Победа в олимпиаде очень почетна, а на право ее проведения ежегодно претендует несколько стран-кандидатов. Награды победителям вручают президенты и члены царствующих домов, руководители правительств тех государств, в которых проходит Международная олимпиада. Россия включилась в международное олимпиадное движение сразу после распада СССР и принимала участие во всех Международных математических олимпиадах начиная с 1992 г. В книге приведены результаты выступлений российских школьников на ММО, тексты заданий и их решения. Каждая задача олимпиады нередко имеет несколько принципиально различных решений. Решения задач в этой книге зачастую основаны на идеях, приведенных российскими участниками ММО в их работах. Перед текстом каждой задачи указана страна, предложившая данную задачу на ММО. Для того чтобы читатели могли получить более полное представление о Международной математической олимпиаде, в книгу включены сведения о порядке проведения ММО, о формировании команды России, а также результаты выступлений ведущих команд за 1992—2008 гг. Порядок проведения олимпиады Претерпев некоторые изменения в период становления, Международная математическая олимпиада проходит по единому регламенту на протяжении многих последних лет. 1* Введение
Олимпиада проводится в два дня (два тура), в каждом школьники решают в течение 4,5 ч по три задачи. Несмотря на их различную сложность, правильное решение каждой задачи оценивается в 7 баллов. Победители олимпиады определяются по сумме всех набранных за два тура баллов по следующим принципам: медалями награждаются не более половины всех участников олимпиады, медали среди победителей распределяются в пропорции 1:2:3 (соответственно золото, серебро, бронза). Участниками олимпиады могут быть молодые люди в возрасте до 20 лет, не обучавшиеся в вузах или университетах на момент начала олимпиады (для России такая возрастная граница кажется странной, но в целом ряде стран Европы, Северной Америки и Азии продолжительность обучения в школах составляет 12—13 лет). Банк задач для олимпиады формируется на основе предложений стран-участниц (до 6 задач от одной страны). Около 30 лучших задач по четырем разделам школьной математики: алгебре и математическому анализу, геометрии, теории чисел, комбинаторике — включается в так называемый шорт-лист олимпиады, по 6—8 задач различной трудности в каждом разделе. На заседаниях, проходящих в течение 2—3 дней, руководители команд, входящие в жюри олимпиады, отбирают из шорт-л иста те 6 задач, которые и будут составлять вариант олимпиады. В последние годы выбор задач для включения в вариант начинается с отбора относительно простых задач — на первые позиции каждого дня. Это ответственный выбор, так как важно предоставить каждому сильному, но, быть может, недостаточно подготовленному школьнику возможность успешно справиться хотя бы с одной задачей ММО (в регламенте олимпиады предусмотрено вручение почетной грамоты тем, кто не стал медалистом олимпиады, но полностью выполнил одно из заданий). «Лицом» Международной математической олимпиады являются красивые сложные задачи, поэтому, после того как пара простых задач определена, происходит отбор двух самых сложных задач олимпиады — на третьи позиции. И уже в конце, соблюдая тематическое разнообразие варианта, выбирают две средние (по позициям и по трудности) задачи. Наконец, на заключительном этапе голосуется распределение задач по двум дням соревнований. Каждая страна имеет на заседаниях жюри один голос, поэтому с учетом большого разнообразия уровня команд и тематических предпочтений разных делегаций обсуждение вариантов иногда проходит в жарких спорах. Рабочий язык заседаний международного жюри — английский, но перед каждым голосованием руководители команд Испании, России, Франции и Германии переводят 4 Введение
предмет голосования на национальные языки, также являющиеся официальными языками ММО. Участники олимпиады получают задания на двух языках: родном и любом дополнительном на свой выбор. Решения участниками выполняются на родных языках. Оценка работ участников осуществляется на так называемой координации, в которой принимают участие, с одной стороны, руководители команд, с другой — члены координационного комитета. Он формируется как из числа математиков страны-организатора ММО, так и из приглашенных математиков других стран. Координация наиболее ответственный этап олимпиады, так как далеко не всегда участники предъявляют на суд жюри идеально написанные решения задач. Участие команд России в ММО Международные математические олимпиады заметно отличаются по содержанию заданий от основных математических соревнований, проводимых в России. Поэтому требуется специальная подготовка наших школьников к ММО, а отбор команды должен включать в себя выполнение кандидатами заданий, по стилю близких к заданиям Международной олимпиады. Подготовка команды осуществляется на летних учебно-тренировочных сборах, проходящих в течение трех недель в июне — июле, непосредственно перед Международной олимпиадой. А формирование команды России на ММО проходит в три этапа. На первом этапе из числа победителей Всероссийской олимпиады выбирается группа из 20—30 кандидатов, и они принимают участие в зимних сборах (второй этап), на которых круг кандидатов сужается примерно до 10—12 человек. Наконец, по итогам выступления кандидатов на Всероссийской олимпиаде следующего года окончательно определяется состав команды России на Международную математическую олимпиаду. Для успешной подготовки команды к олимпиаде летние сборы проводятся в одном из пансионатов, расположенных в Центральной части России, где участники сборов имеют возможность сочетать напряженные учебные занятия с активным отдыхом, спортивными играми. Академическая подготовка национальной сборной включает в себя тематические лекционные и семинарские занятия. Кроме того, во время сборов члены команды участвуют в нескольких тренировочных олимпиадах. В последние годы календарь подготовки команды составлен так, что кандидаты в сборную команду России принимают участие в национальных математических олимпиа- Введение
дах школьников Китая (в январе, Россию представляют в основном учащиеся 11 класса) и Болгарии (в мае, участвуют учащиеся 10 класса, победители Всероссийской олимпиады). На результаты выступлений команд разных стран на Международной математической олимпиаде оказывает влияние несколько объективных факторов: уровень естественно-математического образования, традиции олимпиад- ного движения в стране, государственная поддержка как олимпиадного движения в целом, так и формирования национальной команды и ее участия в ММО в частности. Поэтому страны-участницы на протяжении многих лет демонстрируют стабильные результаты на Международных математических олимпиадах. В России, несмотря на потрясения в конце прошлого столетия, удалось сохранить богатые традиции олимпиадного движения, заложенные во времена существования СССР. Российские школьники успешно выступают на ММО. Как правило, в неофициальном командном зачете наша команда входит в тройку лучших, а по количеству завоеванных за эти годы золотых медалей Россия уступает только Китаю. Ниже приведена таблица, в которой указано общее количество баллов и количество медалей, завоеванных 50 лучшими командами на Международных математических олимпиадах 1992—2008 гг. № п/п 1 2 3 4 5 6 7 8 9 10 11 Страна Китай Россия США Болгария Вьетнам Южная Корея Румыния Иран Венгрия Тайвань Япония Общее количество баллов 3418 3263 3148 2961 2820 2813 2813 2790 2749 2630 2532 Общее количество золотых медалей 81 65 54 39 33 35 33 28 29 22 23 Общее количество серебряных медалей 14 28 40 46 51 48 45 54 51 58 47 Общее количество бронзовых медалей 1 9 7 15 18 17 22 18 18 17 21 Введение
Продолжение № п/п 12 13 14 15 16 17 18 19 20 21 22 23 24 25 26 27 28 29 30 31 32 33 34 35 36 37 38 Страна Германия Украина Великобритания Индия Австралия Белоруссия Польша Турция Канада Израиль Гонконг Сербия Франция Словакия Сингапур Чехия Таиланд Бразилия Казахстан Италия Аргентина Колумбия Грузия Армения Хорватия Молдова Монголия Общее количество баллов 2396 2390 2254 2182 2131 2081 2028 2024 2012 1986 1872 1716 1700 1692 1676 1660 1615 1569 1533 1533 1525 1397 1382 1324 1297 1289 1285 Общее количество золотых медалей 22 23 13 7 9 11 11 8 11 8 3 3 9 3 1 3 5 3 8 4 3 1 2 1 0 5 1 Общее количество серебряных медалей 38 36 37 45 33 32 29 32 24 25 30 25 15 25 21 21 21 18 14 11 18 12 9 9 5 14 15 Общее количество бронзовых медалей 30 25 42 37 44 41 39 43 44 50 46 42 40 43 48 40 34 39 38 43 40 37 39 35 40 27 28 Введение
Продолжение № п/п 39 40 41 42 43 44 45 46 47 48 49 50 Страна Бельгия Латвия Греция Южная Африка Швеция Австрия Мексика Голландия Новая Зеландия Македония Литва Норвегия Общее количество баллов 1275 1226 1220 1217 1209 1183 1114 1087 1047 1039 1035 1023 Общее количество золотых медалей 0 1 0 1 1 2 1 0 1 0 1 2 Общее количество серебряных медалей 6 10 11 8 7 6 6 6 3 3 5 6 Общее количество бронзовых медалей 28 29 30 26 27 26 30 19 26 34 20 19 В последующих таблицах содержится информация о выступлениях сборной России и результаты выступлений первых двадцати команд на Международных олимпиадах за последние 17 лет (1992—2008 гг.). 33-я ММО. 1992 г. Москва (Россия) Результаты сборной России Участник Кожевников Павел Чиликов Алексей Исмаилов Руслан Певцова Юлия Город, школа Калуга, школа № 24 Киров, школа № 35 С.-Петербург, СУМЦ С.-Петербург, СУМЦ Баллы (по задачам) 7 7 7 Т1 2 7 7 7 7 о 7 7 7 7 4 7 7 2 0 5 0 0 0 0 6 4 4 6 3 Сумма баллов 32 32 29 24 Медаль золото золото серебро серебро Введение
Продолжение Участник Никулин Михаил Карпов Дмитрий Город, школа Москва, СУНЦ С.-Петербург, СУМЦ Баллы (по задачам) 1 0 7 2 5 1 3 0 7 4 6 0 5 7 1 6 4 3 Сумма баллов 22 19 Медаль бронза бронза Место 1 2 3 4 5 6 7 8 8 10 10 12 13 14 15 16 17 18 19 20 Командный Страна Китай США Румыния СНГ Великобритания Россия Германия Япония Венгрия Вьетнам Франция Югославия Чехословакия Иран Болгария КНДР Тайвань Южная Корея Австралия Израиль Сумма баллов 240 181 177 176 168 158 149 142 142 139 139 136 134 133 127 126 124 122 118 108 зачет Количество медалей золото 6 3 2 2 2 2 0 1 1 1 1 0 0 0 1 0 0 1 1 0 серебро 0 3 2 3 2 2 4 3 3 2 3 2 2 3 1 3 3 0 1 2 бронза 0 0 2 0 2 2 2 1 1 3 1 4 3 2 3 2 2 4 2 2 Введение
34-я ММО. 1993 г. Стамбул (Турция) Результаты сборной России Участник Федоров Роман Бондарко Михаил Розенблюм Елизавета Поздняков Антон Бирюк Андрей Панов Дмитрий Город, школа Москва, школа № 57 С.-Петербург, школа № 239 С.-Петербург, школа № 239 С.-Петербург, школа № 292 Краснодар, школа № 4 Москва, школа № 57 Баллы (по задачам) 1 7 7 7 0 7 0 2 7 2 3 2 2 7 3 7 7 4 7 0 0 4 3 6 7 7 2 5 5 6 7 6 7 7 5 6 7 7 5 7 7 0 Сумма баллов 37 36 32 30 25 17 Медаль золото золото золото золото серебро бронза Место 1 2 3 4 5 6 7 8 9 10 11 Командный Страна Китай Германия Болгария Россия Тайвань Иран США Венгрия Вьетнам Чехия Румыния Сумма баллов 215 189 178 177 162 153 151 143 138 132 128 зачет Количество медалей золото 6 4 2 4 1 2 2 3 1 1 1 серебро 0 2 4 1 4 3 2 1 4 2 2 бронза 0 0 0 1 1 1 2 2 1 3 3 Введение
Продолжение Место 12 13 14 15 15 17 18 18 20 Страна Словакия Австралия Великобритания Индия Южная Корея Франция Канада Израиль Япония Сумма баллов 126 125 118 116 116 115 113 113 98 Количество медалей золото 1 1 0 0 0 2 1 1 0 серебро 3 2 3 4 3 1 1 2 2 бронза 1 3 3 1 3 1 3 2 3 35-я ММО. 1994 г. Гонконг (Гонконг) Результаты сборной России Участник Бондарко Михаил Карасев Роман Норин Сергей Дюбина Анна Добринская Наталья Борисов Александр Город, школа С.-Петербург, школа № 239 Лобня (Московская обл.), школа № 5 г. Долгопрудный С.-Петербург, школа № 239 С.-Петербург, школа № 239 Саратов, ФТЛ № 1 Нижний Новгород, школа № 40 Баллы (по задачам) 1 7 7 7 7 1 0 2 7 7 7 7 7 7 3 7 7 7 7 5 7 4 7 7 7 7 7 7 5 7 7 7 7 7 5 6 7 7 7 3 7 0 Сумма баллов 42 42 42 38 34 26 Медаль золото золото золото серебро серебро бронза Введение и
Место 1 3 4 5 6 7 8 9 10 11 12 13 13 13 16 17 18 19 20 Командный Страна США Китай Россия Болгария Венгрия Вьетнам Великобритания Иран Румыния Япония Германия Австралия Южная Корея Польша Тайвань Индия Украина Гонконг Франция Аргентина Сумма баллов 252 229 224 223 221 207 206 203 198 180 175 173 170 170 170 168 163 162 161 159 зачет Количество медалей золото 6 3 3 3 1 1 см CVJ 0 1 1 0 0 CVJ 0 0 1 0 1 0 серебро 0 3 2 CVJ 5 5 см CVJ 5 см см см 2 0 4 3 1 см 1 3 бронза 0 0 1 1 0 0 см см 1 3 3 3 4 3 1 3 см 4 3 1 36-я ММО. 1995 г. Торонто (Канада) Результаты сборной России Участник Норин Сергей Островский Михаил Город, школа С.-Петербург, школа № 239 Москва, школа № 57 Баллы (по задачам) 1 7 7 2 7 7 3 7 7 4 7 7 |5 7 7 6 7 4 Сумма баллов 42 39 Медаль золото золото Введение
Продолжение Участник Запорожец Дмитрий Челкак Дмитрий Кацев Илья Есаулова Вероника Город, школа С.-Петербург, школа № 239 С.-Петербург, школа № 30 С.-Петербург, школа № 30 С.-Петербург, школа № 239 Баллы (по задачам) 1 7 7 7 7 2 7 7 7 7 3 7 7 7 7 4 7 7 7 7 5 7 7 7 7 6 3 2 1 0 Сумма баллов 38 37 36 35 Медаль золото золото серебро серебро Место 1 2 3 4 5 6 7 8 9 10 11 12 13 14 15 16 17 Командный Страна Китай Румыния Россия Вьетнам Венгрия Болгария Южная Корея Иран Япония Великобритания США Тайвань Израиль Индия Германия Польша Югославия Сумма баллов 236 230 227 220 210 207 203 202 183 180 178 176 171 165 162 161 154 зачет Количество медалей золото 4 4 4 2 3 1 2 2 1 2 0 0 1 0 1 0 0 серебро 2 CVJ 2 4 1 4 3 3 3 1 3 4 2 3 3 1 2 бронза 0 0 0 0 2 1 1 1 2 3 3 1 CVJ 3 1 5 3 Введение 13
Продолжение Место 17 19 20 Страна Чехия Канада Гонконг Сумма баллов 154 153 151 Количество медалей золото 0 0 0 серебро 1 2 2 бронза 5 3 3 37-я ММО. 1996 г. Мумбай (Индия) Результаты сборной России Участник Дуров Николай Норин Сергей Есаулова Вероника Рудо Елена Салихов Константин Макарычев Юрий Город, школа С.-Петербург, ФМЛ № 239 С.-Петербург, ФМЛ № 239 С.-Петербург, ФМЛ № 239 С.-Петербург, ФМЛ № 239 Москва, СУНЦ МГУ Москва, школа № 57 Баллы (по задачам) 1 7 7 4 см 2 6 2 7 7 7 1 7 7 3 7 7 3 6 5 5 4 7 7 7 7 7 1 5 2 1 1 0 1 0 6 7 7 3 7 0 0 Сумма баллов 37 36 25 23 22 19 Медаль золото золото серебро серебро серебро бронза Командный зачет Место 1 2 3 4 5 Страна Румыния США Венгрия Россия Великобритания Сумма баллов 187 185 167 162 161 Количество медалей золото 4 4 3 см серебро 2 см см 3 4 бронза 0 0 1 1 0 Введение
Продолжение Место 6 7 8 9 10 11 11 13 14 15 16 17 18 19 20 Страна Китай Вьетнам Южная Корея Иран Германия Болгария Япония Польша Индия Израиль Канада Словакия Украина Турция Тайвань Сумма баллов 160 155 151 143 137 136 136 122 118 114 111 108 105 104 100 Количество медалей золото 3 3 2 1 3 1 1 0 1 1 0 0 1 0 0 серебро 2 1 3 4 1 4 3 3 3 2 3 2 0 2 2 бронза 1 1 0 1 1 1 1 3 1 2 3 4 5 3 3 38-я ММО. 1997 г. Мар-дель-Плата (Аргентина) Результаты сборной России Участник Дуров Николай Лепчинский Михаил Черепанов Евгений Митрофанов Михаил Город, школа С.-Петербург, ФМЛ № 239 Челябинск, лицей № 31 Рыбинск(Ярославская обл.), школа № 17 С.-Петербург, ФМЛ № 239 Баллы (по задачам) 1 7 6 4 7 2 7 7 7 7 3 7 7 7 1 4 4 7 7 7 5 7 7 7 7 6 7 3 5 5 Сумма баллов 39 37 37 34 Медаль золото золото золото серебро Введение 15
Продолжение Участник Уздин Сергей Анно Ирина Город, школа С.-Петербург, ФМЛ № 239 Москва, школа № 57 Баллы (по задачам) 1 7 7 2 7 7 3 7 0 4 4 1 5 7 7 6 1 0 Сумма баллов 33 22 Медаль серебро бронза Место 1 2 3 4 5 6 7 7 9 10 11 12 13 14 15 16 17 18 19 20 20 Командный Страна Китай Венгрия Иран США Россия Украина Болгария Румыния Австралия Вьетнам Южная Корея Япония Германия Тайвань Индия Великобритания Белоруссия Чехия Швеция Польша Югославия Сумма баллов 223 219 217 202 202 195 191 191 187 183 164 163 161 148 146 144 140 139 128 125 125 зачет Количество медалей золото 6 4 4 2 3 3 2 2 2 1 1 1 1 0 0 1 0 1 1 0 0 серебро 0 2 2 4 2 3 3 3 3 5 4 3 3 4 3 2 2 2 0 2 2 бронза 0 0 0 0 1 0 1 1 1 0 1 1 2 2 3 2 4 2 3 2 3 16 Введение
39-я ММО. 1998 г. Тайбэй (Тайвань) Результаты сборной России Участник Дуров Николай Дремов Владимир Черепанов Евгений Анно Ирина Розенберг Антон Шаповалов Данил Город, школа С.-Петербург, ФМЛ № 239 Волгодонск (Ростовская обл.), школа № 24 Рыбинск(Ярославская обл.), школа № 17 Москва, школа № 57 С.-Петербург, школа № 419 Иваново, школа № 33 Баллы (по задачам) 1 7 5 CVJ 6 7 4 2 7 7 7 7 7 7 3 7 7 2 3 2 2 4 7 7 7 5 7 7 5 7 0 4 7 3 0 6 4 7 7 0 0 0 Сумма баллов 39 33 29 28 26 20 Медаль золото золото серебро серебро серебро бронза Место 1 2 3 3 5 6 7 8 9 10 Страна Иран Болгария США Венгрия Тайвань Россия Индия Украина Вьетнам Югославия Командный Сумма баллов 211 195 186 186 184 175 174 166 158 156 зачет золото 5 3 3 4 3 см 3 1 1 0 Количество медалей серебро 1 3 3 см 2 3 3 3 3 5 бронза 0 0 0 0 1 1 0 см см 0 2 - Аг; Введение 17
Продолжение Место 11 12 13 14 15 16 17 17 19 20 Страна Румыния Южная Корея Австралия Япония Чехия Германия Великобритания Турция Белоруссия Канада Сумма баллов 155 154 146 139 135 129 122 122 118 113 Количество медалей золото 3 2 0 1 0 0 0 0 0 1 серебро 0 2 4 1 3 3 1 2 1 1 бронза 2 2 2 3 3 2 4 4 4 2 40-я ММО. 1999 г. Бухарест (Румыния) Результаты сборной России Участник Дремов Владимир Петров Федор Поярков Алексей Лифшиц Юрий Евсеев Антон Лебедев Алексей Город, школа Волгодонск (Ростовская обл.), школа № 24 С.-Петербург, ФМЛ № 239 Рыбинск(Ярославская обл.), лицей № 2 С.-Петербург, ФМЛ № 239 Москва, школа № 57 Нижний Новгород, лицей № 40 Баллы (по задачам) 1 7 7 7 7 7 7 2 7 7 7 7 2 7 3 7 3 6 3 2 0 4 7 7 6 7 7 0 5 7 7 7 7 3 6 6 1 5 3 0 1 1 Сумма баллов 36 36 36 31 22 21 Медаль золото золото золото золото серебро серебро 18 Введение
Место 1 1 3 4 5 6 7 8 9 10 11 12 13 14 15 16 17 18 19 20 Командный Страна Китай Россия Вьетнам Румыния Болгария Белоруссия Южная Корея Иран Тайвань США Венгрия Украина Япония Югославия Австралия Турция Германия Индия Польша Великобритания Сумма баллов 182 182 177 173 170 167 164 159 153 150 147 136 135 130 116 109 108 107 104 100 зачет Количество медалей золото 4 4 3 3 3 3 3 2 1 2 1 CsJ CsJ 1 1 1 0 0 1 0 серебро 2 2 3 3 3 3 3 4 5 3 4 2 4 2 1 1 2 3 0 3 бронза 0 0 0 0 0 0 0 0 0 1 1 1 0 3 3 4 4 3 5 2 41-я ММО. 2000 г. Тэджон (Южная Корея) Результаты сборной России Участник Гайфуллин Александр Город, школа Раменское (Московская обл.), гимназия Баллы (по задачам) 1 7 2 7 3 7 4 7 5 7 6 7 Сумма баллов 42 Медаль золото 2* Введение 19
Продолжение Участник Поярков Алексей Лифшиц Юрий Дремов Владимир Федотов Алексей Халявин Андрей Город, школа Рыбинск(Ярославская обл.), гимназия № 2 С.-Петербург, ФМЛ № 239 Волгодонск (Ростовская обл.), школа № 24 С.-Петербург, ФМЛ № 239 Киров, ФМЛ № 35 Баллы (по задачам) 1 7 7 7 7 7 2 7 7 7 7 1 3 7 7 1 1 2 4 7 4 7 6 7 5 7 7 7 7 7 6 7 6 4 5 3 Сумма баллов 42 38 33 33 27 Медаль золото золото золото золото серебро Место 1 2 3 4 5 5 7 8 9 10 11 11 13 14 Командный Страна Китай Россия США Южная Корея Болгария Вьетнам Белоруссия Тайвань Венгрия Иран Румыния Израиль Украина Индия Сумма баллов 218 215 184 172 169 169 165 164 156 155 139 139 135 132 зачет Количество медалей золото 6 5 3 3 CVJ 3 CVJ 3 1 2 1 см CVJ 0 серебро 0 1 3 3 3 см 2 2 5 3 3 1 2 5 бронза 0 0 0 0 1 1 2 1 0 1 2 3 0 1 20 Введение
Продолжение Место 15 16 17 18 18 20 20 Страна Япония Австралия Канада Турция Словакия Германия Армения Сумма баллов 125 122 112 111 111 108 108 Количество медалей золото 1 1 1 0 0 1 0 серебро 2 3 2 3 2 1 2 бронза 3 1 1 1 3 2 3 42-я ММО. 2001 г. Вашингтон (США) Результаты сборной России Участник Спиридонов Сергей Воробьев Андрей Соколов Сергей Гарбер Михаил Глазырин Алексей Халявин Андрей Город, школа Ижевск, школа № 41 С.-Петербург, ФМЛ № 239 Рыбинск(Ярославская обл.), школа № 30 Ярославль, школа № 33 Челябинск, лицей № 11 Киров, ФМЛ № 35 Баллы (по задачам) 1 6 7 7 7 7 7 2 7 4 0 7 3 7 3 7 1 5 4 7 1 4 7 7 7 7 7 7 5 7 7 7 7 7 4 6 5 7 7 0 0 CsJ Сумма баллов 39 33 33 32 31 28 Медаль золото золото золото золото золото серебро Командный зачет I Место 1 2 Страна Китай Россия Сумма баллов 225 196 Количество медалей золото 6 5 серебро 0 1 бронза 0 0 Введение 21
Продолжение Место 2 4 4 6 7 8 9 10 11 12 13 14 15 16 17 18 19 19 Страна США Болгария Южная Корея Казахстан Индия Украина Тайвань Вьетнам Турция Белоруссия Япония Германия Румыния Бразилия Израиль Иран Гонконг Польша Сумма баллов 196 185 185 168 148 143 141 139 136 135 134 131 129 120 113 111 107 107 Количество медалей золото 4 3 3 4 2 1 1 1 1 1 1 1 1 0 1 0 0 0 серебро CsJ 3 3 1 2 5 5 4 3 CsJ 3 3 CsJ 4 2 2 2 3 бронза 0 0 0 0 2 0 0 0 2 3 2 1 2 2 1 4 4 1 43-я ММО. 2002 г. Глазго (Великобритания) Результаты сборной России Участник Халявин Андрей Бадзян Андрей Гольберг Олег Город, школа Киров, ФМЛ Челябинск, ФМЛ № 31 Ростов-на-Дону, школа № 8 Баллы (по задачам) 1 7 7 7 2 7 7 7 3 7 5 1 4 7 7 7 5 7 7 7 6 7 3 7 Сумма баллов 42 36 36 Медаль золото золото золото 22 Введение
Продолжение Участник Сухов Кирилл Дубашинский Михаил Стырт Олег Город, школа С.-Петербург, ФМЛ № 239 С.-Петербург, ФМЛ № 239 Омск, лицей № 64 Баллы (по задачам) 1 7 7 7 2 7 7 7 3 1 1 1 4 7 7 7 5 7 7 7 6 3 0 0 Сумма баллов 32 29 29 Медаль золото золото золото Место 1 CsJ 3 4 5 6 7 8 9 10 11 12 12 14 14 16 16 18 19 20 Командный Страна Китай Россия США Болгария Вьетнам Южная Корея Тайвань Румыния Индия Германия Иран Канада Венгрия Белоруссия Турция Казахстан Япония Израиль Франция Украина Сумма баллов 212 204 171 167 166 163 161 157 156 144 143 142 142 135 135 133 133 130 127 124 зачет Количество медалей золото 6 6 4 3 3 1 1 2 1 CsJ 0 1 1 1 1 0 1 0 0 1 серебро 0 0 1 CsJ 1 5 4 3 3 1 4 3 CsJ 2 1 3 3 3 CsJ 3 бронза 0 0 0 1 2 0 1 1 CsJ 2 2 1 3 3 4 3 1 3 3 0 Введение 23
44-я ММО. 2003 г. Токио (Япония) Результаты сборной России Участник Гольберг Олег Гравин Николай Бадзян Андрей Волков Юрий Ширяев Дмитрий Смирнов Александр Город, школа Ростов-на-Дону, школа № 8 С.-Петербург, ФМЛ № 239 Челябинск, ФМЛ № 31 Кемерово, классический лицей С.-Петербург, ФМЛ № 239 С.-Петербург, ФМЛ № 239 Баллы (по задачам) 1 7 7 7 7 7 2 2 3 7 7 3 7 5 3 7 7 0 0 0 0 4 7 7 7 7 7 7 5 7 7 7 7 1 1 6 7 1 1 1 1 1 Сумма баллов 38 36 29 25 23 16 Медаль золото золото золото серебро серебро бронза Место 1 2 3 4 5 6 7 8 9 10 10 Командный Страна Болгария Китай США Вьетнам Россия Южная Корея Румыния Турция Япония Великобритания Венгрия Сумма баллов 227 211 188 172 167 157 143 133 131 128 128 зачет Количество медалей золото 6 5 4 2 3 2 1 1 1 1 1 серебро 0 1 2 3 2 4 4 3 3 2 3 бронза 0 0 0 1 1 0 1 1 2 3 1 24 Введение
Продолжение Место 12 12 14 15 16 17 17 19 19 Страна Канада Казахстан Украина Индия Тайвань Германия Иран Белоруссия Таиланд Сумма баллов 119 119 118 115 114 112 112 111 111 Количество медалей золото 2 1 1 0 1 1 0 1 1 серебро 0 2 2 4 2 2 3 2 1 бронза 3 2 3 1 2 1 2 2 3 45-я ММО. 2004 г. Афины (Греция) Результаты сборной России Участник Бадзян Андрей Дубашинский Михаил Исаев Михаил Петухова Надежда Шнурников Игорь Пермяков Дмитрий Город, школа Челябинск, ФМЛ № 31 С.-Петербург, ФМЛ № 239 Барнаул, гимназия № 42 С.-Петербург, ФТШ Краснодар, гимназия №36 Снежинск (Челябинская обл.), гимназия № 127 Баллы по задачам 1 7 7 7 7 7 7 2 7 7 6 5 7 2 3 7 7 4 3 0 4 4 7 "7 7 7 7 7 5 7 7 7 7 4 3 6 7 7 7 6 0 0 Сумма баллов 42 42 38 35 25 23 Медаль золото золото золото золото серебро бронза Введение 25
Место 1 2 3 4 5 6 7 8 9 10 11 12 13 14 15 16 17 18 19 20 Командный Страна Китай США Россия Вьетнам Болгария Тайвань Венгрия Япония Иран Румыния Украина Южная Корея Белоруссия Индия Израиль Польша Молдова Сингапур Монголия Великобритания Сумма баллов 220 212 205 196 194 190 187 182 178 176 174 166 154 151 147 142 140 139 135 134 зачет Количество медалей золото 6 5 4 4 3 3 CsJ 2 1 1 1 2 0 0 1 2 2 0 0 1 серебро 0 1 1 2 3 3 3 4 5 4 5 2 4 4 1 1 0 3 3 1 бронза 0 0 1 0 0 0 1 0 0 1 0 2 2 2 4 1 4 3 2 4 46-я ММО. 2005 г. Мерида (Мексика) Результаты сборной России Участник Гаврилюк Андрей Город, школа Долгопрудный (Московская обл.), ФМШ № 5 Баллы по задачам 1 7 2 7 3 7 4 7 5 7 6 7 Сумма баллов 42 Медаль золото 26 Введение
Продолжение Участник Магазинов Александр Калинин Никита Козлов Павел Катышев Алексей Астахов Василий Город, школа Ярославль, лицей № 33 С.-Петербург, ФМЛ № 239 с. Шурскол (Ярославская обл.), школа С.-Петербург, ФМЛ № 239 Саратов, ФТЛ № 1 Баллы по задачам 1 7 7 7 7 7 2 7 7 7 7 7 3 6 7 0 0 0 4 7 7 7 7 7 5 7 7 7 7 0 6 7 1 7 CsJ 7 Сумма баллов 41 36 35 30 28 Медаль золото золото золото серебро серебро Место 1 2 3 4 5 6 7 8 9 9 11 12 13 14 15 Командный Страна Китай США Россия Иран Южная Корея Румыния Тайвань Япония Украина Венгрия Болгария Германия Великобритания Сингапур Вьетнам Сумма баллов 235 213 212 201 200 191 190 188 181 181 173 163 159 145 143 зачет Количество медалей золото 5 4 4 CsJ 3 . 4 3 3 CsJ CsJ CsJ 1 1 0 0 серебро 1 2 CsJ 4 3 1 CsJ 1 2 3 3 3 3 4 3 бронза 0 0 0 0 0 1 1 2 2 1 1 2 2 2 3 Введение 27
Продолжение Место 16 17 18 19 20 Страна Чехия Гонконг Белоруссия Канада Словакия Сумма баллов 139 138 136 132 131 Количество медалей золото 1 1 1 1 0 серебро CsJ 3 3 2 4 бронза CsJ 1 1 2 2 47-я ММО. 2006 г. Любляна (Словения) Результаты сборной России Участник Магазинов Александр Девятое Ростислав Образцов Тимофей Затицкий Павел Катышев Алексей Глазман Александр Город, школа Ярославль, лицей № 33 Москва, лицей «Вторая школа» С.-Петербург, ФМЛ № 239 С.-Петербург, ФМЛ № 239 С.-Петербург, ФМЛ № 239 С.-Петербург, ФМЛ № 239 Баллы по задачам 1 7 7 7 7 7 7 2 7 7 7 1 7 7 3 7 0 0 CsJ 2 0 4 7 7 7 7 7 7 5 7 7 7 7 0 0 6 7 7 0 1 0 0 Сумма баллов 42 35 28 25 23 21 Медаль золото золото золото серебро серебро серебро Место 1 CsJ 3 4 Страна Китай Россия Южная Корея Германия Командный Сумма баллов 214 174 170 157 зачет золото 6 3 4 4 Количество медалей серебро 0 3 CsJ 0 бронза 0 0 0 2 Введение
Продолжение Место 5 6 7 8 9 10 11 12 13 14 15 15 17 18 19 19 Страна США Румыния Япония Иран Молдова Тайвань Польша Италия Вьетнам Гонконг Канада Таиланд Венгрия Словакия Великобритания Турция Сумма баллов 154 152 146 145 140 136 133 132 131 129 123 123 122 118 117 117 Количество медалей золото CsJ 3 2 3 CsJ 1 1 CsJ 2 1 0 1 0 1 0 0 серебро 4 1 3 3 1 5 2 CsJ 2 3 5 3 5 2 4 4 бронза 0 CsJ 1 0 3 0 3 0 2 2 1 CsJ 1 3 1 1 48-я ММО. 2007 г. Ханой (Вьетнам) Результаты сборной России Участник Матвеев Константин Илюхина Мария Есин Алексей Город, школа Омск, лицей № 66 Москва, лицей «Вторая школа» ст. Старониже- стеблиевская (Краснодарский край), СОШ №55 Баллы по задачам 1 7 7 7 2 7 7 7 3 2 5 2 4 7 7 7 5 7 7 7 6 7 1 1 Сумма баллов 37 34 31 Медаль золото золото золото Введение 29
Продолжение Участник Дроздов Сергей Митрофанов Иван Волков Владислав Город, школа С.-Петербург, ФТШ Коломна (Московская обл.), гимназия № 2 С.-Петербург, ФМЛ № 239 Баллы по задачам 1 7 7 7 2 7 7 7 3 1 1 1 4 7 7 7 5 7 7 2 6 0 0 0 Сумма баллов 29 29 24 Медаль золото золото серебро Место 1 CsJ 3 3 5 6 6 8 9 9 11 12 12 14 15 16 17 Командный Страна Россия Китай Вьетнам Южная Корея США Украина Япония КНДР Болгария Тайвань Румыния Гонконг Иран Таиланд Германия Венгрия Турция Сумма баллов 184 181 168 168 155 154 154 151 149 149 146 143 143 133 132 129 124 зачет Количество медалей золото 5 4 3 2 2 3 2 1 CsJ CsJ 1 0 1 1 1 0 1 серебро 1 CsJ 3 4 3 1 4 4 3 3 4 5 3 3 3 5 2 бронза 0 0 0 0 1 2 0 0 1 1 1 1 CsJ 2 1 0 2 30 Введение
Продолжение Место 18 19 20 Страна Польша Белоруссия Молдова Сумма баллов 122 119 118 Количество медалей золото 1 1 0 серебро 2 1 3 бронза CsJ 4 2 49-я ММО. 2008 г. Мадрид (Испания) Результаты сборной России Участник Волков Владислав Бабичев Дмитрий Бойкий Роман Кудык Никита Бажов Иван Горинов Евгений Город, школа С.-Петербург, ФМЛ № 239 Долгопрудный (Московская обл.), ФМШ № 5 С.-Петербург, ФМЛ № 239 Омск, гимназия № 117 Екатеринбург, гимназия № 9 Киров, ФМЛ Баллы по задачам 1 7 7 7 7 7 7 2 7 7 4 4 7 4 3 7 7 7 7 1 6 4 7 7 7 7 7 7 5 7 7 7 7 7 7 6 CsJ 1 0 0 CsJ 0 Сумма баллов 37 36 32 32 31 31 Медаль золото золото золото золото золото золото Место 1 CsJ 3 3 Страна Китай Россия США Южная Корея Командный Сумма баллов 217 199 190 188 зачет золото 5 6 4 4 Количество медалей серебро 1 0 2 CsJ бронза 0 0 0 0 Введение 31
Продолжение Место 5 6 7 8 9 10 11 12 13 14 15 16 17 17 19 20 20 Страна Иран Таиланд КНДР Турция Тайвань Венгрия Япония Вьетнам Польша Болгария Украина Бразилия Перу Румыния Австралия Сербия Германия Сумма баллов 181 175 173 170 168 165 163 159 157 154 153 152 141 141 140 139 139 Количество медалей золото 1 2 2 3 CsJ CsJ 2 CsJ CsJ 2 2 0 1 0 0 1 1 серебро 5 3 4 1 4 3 3 2 3 1 2 5 3 4 5 3 CsJ бронза 0 1 0 2 0 1 1 CsJ 1 3 2 1 2 2 1 0 3 32 Введение
38-я олимпиада. 1997 г. 9LJL (Белоруссия) Координатная плоскость разделена на единичные квадраты с вершинами в точках с целочисленными координатами. Квадраты раскрашены в черный и белый цвета в шахматном порядке. Для каждой пары натуральных чисел тип рассматривается прямоугольный треугольник с вершинами в точках с целочисленными координатами, катеты которого параллельны осям координат и имеют длину тип. Пусть Sx — суммарная площадь окрашенной черным части треугольника, 8lS2 — суммарная площадь части, окрашенной белым. Пусть/(m, n) = |S1-iS2|. а) Вычислите /(/п, п) для чисел тип одной четности. б) Докажите, что f(m, п) ^ - max {/n, п) для любых тип. в) Докажите, что не существует такого числа С, что f(m9 n) < С для любых тип. 97.2 (Великобритания) В треугольнике ABC угол А наименьший. Пусть U — точка на той дуге ВС описанной около треугольника окружности, которая не содержит точку А. Серединные перпендикуляры к отрезкам АВ и АС пересекают прямую AU в точках V и W соответственно. Прямые BV и CW пересекаются в точке Т. Докажите, что AU =ТВ + ТС. 97.3 (Россия) Пусть х19 х2у ..., хп — такие действительные числа, что п + 1 \ХЛ + Хп + + ХА = 1 И \Х: для всех i = 1, 2, ..., п. Докажите, что существует такая перестановка Уи У2> •••> Уп чисел х19 х2у ..., х„9 что 2z/2+ п+ 1 Задачи Международных математических олимпиад 33 о - Агахан
97^4 (Иран) Таблица п х п, в каждой клетке которой записано одно из чисел множества S = {1, 2, ..., 2п - 1} называется серебряной9 если для каждого i=l, 2, ..., пв объединении i-й строки и £-го столбца содержатся все элементы множества S. Докажите, что: а) не существует серебряной таблицы для п = 1997; б) серебряные таблицы существуют для бесконечного множества натуральных чисел п. 97.5 (Чехия) Найдите все пары (а, Ъ) натуральных чисел такие, что аь2 = Ъа. 97.6 (Литва) Для каждого натурального п через f(n) обозначим количество различных представлений числа п в виде суммы степеней двойки с целыми неотрицательными показателями. (Представления, отличающиеся только порядком слагаемых, считаются одинаковыми.) Например, /(4) = 4, так как число 4 может быть представлено следующими четырьмя способами: 4; 2 + 2; 2 + 1 + 1; 1 + 1 + 1 + 1. Докажите, что для любого натурального п> 3 выполнено неравенство f(2n) 39-я олимпиада. 1998 г. 98,1 (Люксембург) В выпуклом четырехугольнике ABCD диагонали АС и BD перпендикулярны, а стороны АВ и CD не параллельны. Серединные перпендикуляры к сторонам АВ и CD пересекаются в точке Р, лежащей внутри четырехугольника ABCD. Докажите, что около четырехугольника ABCD можно описать окружность тогда и только тогда, когда площади треугольников АВР и С DP равны. 98-2 (Индия) На соревновании выступили а участников, их оценивали Ъ судей, где Ъ — нечетное число, не меньшее 3. За выступление участника каждый судья ставил оценку «удовлетворительно» или «неудовлетворительно». Число k таково, что для любых двух судей имеется не более k участников, получивших у них одинаковые оценки. Докажите неравенство k Ь- 1 Задачи Международных математических олимпиад
98.3 (Белоруссия) Пусть d (n) — количество всевозможных натуральных делителей натурального числа п, включая 1 и само п. их « ж. d(n2) Найдите все такие натуральные числа k> что —-—- = k d(n) при каком-либо п. 9ВА_ (Великобритания) Найдите все пары (а, Ь) натуральных чисел такие, что a2b + а + b делится на ab2 + 6 + 7. 98-5 (Украина) Пусть / — центр окружности, вписанной в треугольник ABC. Обозначим через К> L, М точки, в которых эта окружность касается сторон ВС, С А, АВ соответственно. Прямая, проведенная через точку В параллельно прямой МКУ пересекает прямые LM и LK в точках R и S соответственно. Докажите, что угол RIS острый. 98.6 (Болгария) Рассматриваются все функции f:N-+ N9 удовлетворяющие равенству для любых натуральных s и t. Найдите наименьшее возможное значение /(1998). 40-я олимпиада. 1999 г. 99.1 (Эстония) Найдите все конечные множества S точек плоскости, содержащие не менее трех точек, удовлетворяющие следующему условию: для любых двух различных точек А и Б из множества S серединный перпендикуляр к отрезку АВ является осью симметрии множества S. 99.2 (Польша) Пусть п — целое число, п ^ 2. а) Найдите наибольшее число С такое, что неравенство 5>£*;.(*2+*?)<с( 2*«] (1) выполняется для всех неотрицательных действительных чисел х19 х2, ..., хп- б) Для найденного числа С определите условие, при котором неравенство (1) обращается в равенство. з* Задачи Международных математических олимпиад 35
99-3 (Белоруссия) В квадрате клетчатой бумаги размером пх п клеток, где п — четное число, отмечены N клеток таким образом, что каждая клетка квадрата (отмеченная или неотмеченная) имеет хотя бы одну отмеченную соседнюю клетку. (Соседними считаются клетки, имеющие общую сторону.) Определите наименьшее возможное значение N. 99.4 (Тайвань) Найдите все пары (п, р) натуральных чисел такие, чтор — простое, п ^ 2р и (р — 1)п + 1 делится нат^"1. 99-5 (Россия) Две окружности Гх и Г2, содержащиеся внутри окружности Г, касаются Г в различных точках М и N соответственно. Окружность Гг проходит через центр окружности Г2. Прямая, проходящая через две точки пересечения окружностей Г\ и Г2, пересекает окружность Г в точках А и Б. Прямые МА и MB пересекают Гх в точках СиВ соответственно. Докажите, что CD касается Г2. 99,6 (Япония) Найдите все функции /: jR —► jR такие, что /(* " Ш) = f(f(y)) + xf(y) + fix) - 1 для всех х € R9 у е R. 41-я олимпиада. 2000 г. 00.1 (Россия) Окружности Гх и Г2 пересекаются в точках М и N. Прямая / — общая касательная к окружностям Гх и Г2 такая, что точка М расположена к прямой / ближе, чем точка N. Прямая / касается окружности Г1 в точке А, а окружности Г2 в точке В. Прямая, проходящая через точку М параллельно Z, пересекает вторично окружность Г1 в точке С, а окружность Г2 в точке D. Прямые С А и DB пересекаются в точке Е> прямые AN и CD — в точке Р, прямые BN и CD — в точке Q. Докажите, что ЕР = EQ. 00,2 (США) Положительные числа а, Ь, с таковы, что аЪс = 1. Докажите, что с)[ а 36 Задачи Международных математических олимпиад
00.3 (Белоруссия) Дано натуральное число п ^ 2. Пусть сначала на горизонтальной прямой сидят п блох, не все в одной точке. Для положительного числа X определим прыжок следующим образом: выбираются две блохи, сидящие в произвольных точках А и Б, причем А левее В9 и блоха, сидящая в А, прыгает в точку С, расположенную на данной прямой справа от В такую, что ВС _ . ~АВ~ К' Определите все значения X такие, что для любой точки М на этой прямой и для любого начального расположения п блох существует конечная последовательность прыжков, после которой все блохи окажутся справа от точки М. 00L4 (Венгрия) У фокусника 100 карточек, занумерованных натуральными числами от 1 до 100. Он раскладывает все карточки в три ящика — красный, белый и синий — так, чтобы в каждом ящике лежала хотя бы одна карточка. Один из зрителей выбирает два из трех ящиков, вынимает из них по одной карточке и объявляет сумму номеров вытянутых карточек. Зная эту сумму, фокусник определяет тот ящик, из которого карточки не вынимались. Сколькими различными способами можно разложить карточки по ящикам так, чтобы этот фокус всегда удавался? (Способы, при которых хотя бы одна карточка попадает в разные ящики, считаются различными.) 00.5 (Россия) Существует ли такое натуральное число п, что п имеет ровно 2000 различных простых делителей и 2п + 1 делится на п? 00,6 (Россия) Пусть AHl9 ВН2> СН3 — высоты остроугольного треугольника ABC. Окружность, вписанная в треугольник ABC 9 касается сторон ВС, С А, АВ в точках Tl9 Т2, Т3 соответственно. Прямые ll9 12> /3 являются образами прямых Н2Н3, HsHl9 HXH2 при симметрии относительно прямых Т2Т3, TsTl9 TXT2 соответственно. Докажите, что прямые ll9 12> ls образуют треугольник с вершинами на окружности, вписанной в треугольник ABC. Задачи Международных математических олимпиад 37
42-я олимпиада. 2001 г. 0JL1 (Южная Корея) Точка О — центр окружности, описанной около остроугольного треугольника ABC. Точка Р — основание высоты, опущенной из вершины А на сторону ВС. Известно, что ZBCA ^ ZABC + 30°. Докажите, что ZCAB + ZCOP < 90°. 01.2 (Южная Корея) Докажите, что a b с ^ ., у! a2 +Sbc yjb2 + Sea yjc2 + Sab для любых положительных чисел a, b и с. 01,3 (Германия) В математической олимпиаде приняли участие 21 мальчик и 21 девочка. Известно, что: — каждый из них решил не более шести задач; — для каждого мальчика и каждой девочки найдется по крайней мере одна задача, которая была решена ими обоими. Докажите, что найдется задача, которую решили хотя бы 3 мальчика и 3 девочки. 01L4 (Германия) Пусть п — нечетное число, п > 1, и kl9 ft2, ..., kn — данные целые числа. Для каждой из п\ перестановок а = (а19 а2, ..., ап) чисел 1, 2, ..., п положим п S(a)= ^ktat. Докажите, что найдутся такие различные перестановки Ъ и с, что S(b) — S(c) делится на п\. 01.5 (Израиль) В треугольнике ABC биссектриса угла ВАС пересекает сторону ВС в точке Р, а биссектриса угла ABC пересекает сторону СА в точке Q. Известно, что ZBAC = 60° и АВ + ВР = AQ + QB. Чему могут равняться величины углов треугольника ABC? 01.6 (Болгария) Пусть а, Ъу су d — такие целые числа, что а> b > > с > d > 0. Предположим, что ас + bd = (b + d + а - с) (b + d - а + с). Докажите, что число ab + cd не является простым. 38 Задачи Международных математических олимпиад
43-я олимпиада. 2002 г. 02.1 (Колумбия) Дано натуральное число п. Обозначим через Т множество точек (Xj у) координатной плоскости, где х и у — неотрицательные целые числа такие, что х + у < п. Каждая точка из Т окрашена в красный или синий цвет. Если точка (х, у) красная, то все точки (х\ у') из Т, для которых х' ^ х и у' ^ у9 также красные. Назовем Х-множеспгвом множество, состоящее из п синих точек, имеющих различные координаты х> a Y-множеством множество, состоящее из п синих точек, имеющих различные координаты у. Докажите, что количество Х-множеств равно количеству У-множеств. 02.2 (Южная Корея) Дана окружность Г с центром О и диаметром ВС. Пусть А — такая точка окружности Г, что 0° < ZAOB < 120°, a D — середина дуги АВ, не содержащей С. Прямая, проходящая через точку О параллельно DA, пересекает прямую АС в точке J. Серединный перпендикуляр к отрезку О А пересекает окружность Г в точках Е и F. Докажите, что точка J является центром окружности, вписанной в треугольник CEF. 02.3 (Румыния) Найдите все пары натуральных чисел т ^ 3, п ^ 3, для которых существует бесконечно много таких натуральных чисел а, что число — целое. ап + а2- 1 02.4 (Румыния) Дано натуральное число п, большее 1. Обозначим через dl9 d2, -.., dk все его делители так, что 1 = dx < < d2 < ... < dk = п. Пусть D = dxd2 + d2ds + ... + dk_1dk. а) Докажите, что D < n2. б) Найдите все n, для которых число D — делитель числа п2. 02.5 (Индия) Найдите все функции /: jR —> R, такие, что (/(*) + f(2))(f(y) + f(t)) = f(xy - Zt) + f(xt + yz) для всех действительных х, г/, г, t. Задачи Международных математических олимпиад 39
02.6 (Украина) На плоскости расположены окружности Г19 Г2, ..., Гп радиуса 1 каждая с центрами.О19 О2, ..., Оп соответственно, где п> 3. Известно, что любая прямая плоскости имеет общие точки не более чем с двумя из этих окружностей. Докажите, что ; i<у < n 44-я олимпиада. 2003 г. 03,1 (Бразилия) Пусть А — подмножество множества S = {1, 2, ..., 1000000}, содержащее в точности 101 элемент. Докажите, что найдутся такие числа tl9 t2, ..., *юо из &, что множества Aj = {x + tt\xeA} для j = 1, 2, ..., 100 будут попарно не пересекающимися. 03-2 (Болгария) Найдите все такие пары (а, Ь) натуральных чисел, что число а2 2ab2 - Ь3 + 1 является натуральным. 03.3 (Польша) Дан выпуклый шестиугольник, у которого для каждой из трех пар противоположных сторон выполняется условие: отношение расстояния между серединами этих сторон к сумме длин этих сторон равно —. До- кажите, что все углы этого шестиугольника равны. JH3L4 (Финляндия) Пусть ABCD — вписанный четырехугольник. Обозначим через Р, Q и R основания перпендикуляров, опущенных из точки D на прямые ВС> С А и АВ соответственно. Докажите, что PQ — QR тогда и только тогда, когда биссектрисы углов ABC и ADC пересекаются на прямой АС. J33J5 (Ирландия) Пусть п — натуральное число и х19 х2, ,.., хп — такие действительные числа, что х1 ^ х2 ^ ... ^ хп. Задачи Международных математических олимпиад
а) Докажите, что = iy=l б) Докажите, что равенство достигается тогда и только тогда, когда числа х19 х29 ..., хп образуют арифметическую прогрессию. 03.6 (Франция) Пусть р — простое число. Докажите, что существует такое простое число q, что при любом целом п число пР - р не делится на q. 45-я олимпиада. 2004 г. 04.1 (Румыния) Пусть ABC — остроугольный треугольник, в котором АВ ^ АС. Окружность с диаметром ВС пересекает стороны АВ и АС в точках М и N соответственно. Обозначим через О середину стороны ВС. Биссектрисы углов ВАС и MON пересекаются в точке R. Докажите, что окружности, описанные около треугольников BMR и CNR, имеют общую точку, лежащую на стороне ВС. 04.2 (Южная Корея) Найдите все многочлены Р(х) с действительными коэффициентами, удовлетворяющие равенству Р(а - Ъ) + Р{Ъ - с) + Р(с -а) = 2Р(а + b + с) для любых действительных чисел а, 6, с таких, что ab + be + са = 0. 04кЗ (Эстония) Назовем крюком фигуру, состоящую из шести единичных квадратов, как показано на рисунке 1, а также любую фигуру, которую можно получить из нее с помощью поворотов и переворотов. Найдите все прямоугольники т х п, которые можно замостить крюками. Рис. 1 Iff —Г" ! р- - L I ! I | L i \— I i i i : i j Задачи Международных математических олимпиад
04Л (Южная Корея) Пусть п ^ 3 — натуральное число. Пусть tl9t2, ..., tn — положительные действительные числа такие, что Докажите, что числа tt, tp tk являются длинами сторон треугольника при всех £, у, k таких, что 04.5 (Польша) В выпуклом четырехугольнике ABCD диагональ BD не является ни биссектрисой угла ABC, ни биссектрисой угла CD А. Точка Р, лежащая внутри четырехугольника ABCD, удовлетворяет условиям: ZPBC = /.DBA, /PDC = /BDA. Докажите, что четырехугольник ABCD является вписанным тогда и только тогда, когда АР = СР. 04.6 (Иран) Назовем натуральное число полосатым, если любые две соседние цифры в его десятичной записи имеют разную четность. Найдите все натуральные п, для каждого из которых существует полосатое число, делящееся на п. 46-я олимпиада. 2005 г. 05.1 (Румыния) На сторонах равностороннего треугольника ABC выбраны шесть точек: Аг и А2 на ВС; Вг и В2 на СА; Сг и С2 на АВ. Эти точки являются вершинами выпуклого шестиугольника А1А2В1В2С1С2, стороны которого имеют равные длины. Докажите, что прямые АгВ2, ВгС2 и СгА2 пересекаются в одной точке. 05.2 (Нидерланды) Пусть аг, а2, ..., ап, ... — последовательность целых чисел, в которой содержится бесконечно много как положительных, так и отрицательных членов. Известно, что для каждого натурального п все п остатков от деления чисел а19 а2, ..., ап на число п различны. Докажите, что каждое целое число встречается в этой последовательности ровно один раз. Задачи Международных математических олимпиад
05-3 (Южная Корея) Пусть х, у и z — такие положительные числа, что xyz > 1. Докажите, что уЪ_у2 2Ь_22 ^ 2 2 5 2 2 ^ хъ + у2 + z2 уъ+ z2+ x2 z5+ х2+ у2 05.4 (Польша) Последовательность а19 а2, ... определена следующим образом: ап = 2п + Зп + 6п - 1 (п = 1, 2, ...). Найдите все натуральные числа, которые взаимно просты с каждым членом этой последовательности. 05.5 (Польша) Дан выпуклый четырехугольник ABCD, стороны ВС и AD которого равны, но не параллельны. Пусть Е и F — такие внутренние точки отрезков ВС и AD соответственно, что BE = DF. Прямые АС и BD пересекаются в точке Р, прямые BD и EF пересекаются в точке Q, прямые EF и АС пересекаются в точке R. Рассмотрим треугольники PQR, получаемые для всех таких точек Е и F. Докажите, что окружности, описанные около всех этих треугольников, имеют общую точку, отличную от Р. 05.6^ (Румыния) На математической олимпиаде участникам были предложены 6 задач. Оказалось, что каждую пару задач решили более чем — от общего числа участников, о но никто не решил все 6 задач. Докажите, что найдутся по крайней мере два участника, каждый из которых решил ровно 5 задач. 47-я олимпиада. 2006 г. 06-1 (Южная Корея) Точка J — центр окружности, вписанной в треугольник ABC. Внутри треугольника выбрана такая точка Р, что ZPBA + ZPCA = ZPBC + ZPCB. Докажите, что АР ^ А/, причем равенство выполняется тогда и только тогда, когда точка Р совпадает с точкой /. Задачи Международных математических олимпиад 43
06.2 (Сербия) Диагональ правильного 2006-угольника Р называется хорошей, если ее концы делят границу многоугольника Р на две части, каждая из которых содержит нечетное число сторон. Стороны многоугольника Р также называются хорошими. Пусть 2003 диагонали многоугольника Р, никакие две из которых не имеют общих точек внутри Р, разбивают Р на треугольники. Какое наибольшее число равнобедренных треугольников, каждый из которых имеет две хорошие стороны, может иметь такое разбиение? 06.3 (Ирландия) Определите наименьшее действительное число М такое, что неравенство \ab(a2 - Ь2) + Ьс(Ь2 - с2) + са(с2 - а2)\ ^ М(а2 + Ъ2 + с2)2 выполняется для любых действительных чисел а, Ь, с. 06^4 (США) Найдите все пары (л:, у) целых чисел такие, что 1 + 2* + 22х + 1 = у2. 06.5 (Румыния) Пусть Р(х) — многочлен степени п > 1 с целыми коэффициентами, k — произвольное натуральное число. Рассмотрим многочлен Q(*) = P(P(...P(P (*))...)) (здесь Р применен k раз). Докажите, что существует не более п целых чисел t таких, что Q(t) = t. 06.6 (Сербия) Каждой стороне Ъ выпуклого многоугольника Р поставлена в соответствие наибольшая из площадей треугольников, содержащихся в Р, одна из сторон которых совпадает с Ь. Докажите, что сумма площадей треугольников, соответствующих всем сторонам многоугольника Р, не меньше удвоенной площади этого многоугольника. 48-я олимпиада. 2007 г. 07-1 (Новая Зеландия) Даны действительные числа а19 а2, ..., ап. Для каждого i (I ^ i ^ п) положим dt = max {a,j \ 1 ^ у ^ i) - min {ay | i ^ j ^ п}. Задачи Международных математических олимпиад
Пусть d = max{dj: 1 ^ i ^ n). а) Докажите, что для любых действительных чисел хг ^ х2 ^ ... ^ хп справедливо неравенство fljC; - at\ : 1 ^ i ^ п) ^ —. (1) б) Покажите, что существуют такие действительные числа Х-^ ^ Х2 ^ ... ^ ^я' что неравенство (1) обращается в равенство. 07^ (Люксембург) Даны пять точек А, Б, С, Z), £ такие, что ABCD — параллелограмм, а около четырехугольника BCED можно описать окружность. Прямая I проходит через точку А, пересекает отрезок DC в его внутренней точке F, а прямую ВС в точке G. Предположим, что EF = EG = ЕС. Докажите, что прямая I является биссектрисой угла DAB. 07.3 (Россия) Среди участников математического соревнования некоторые дружат между собой; если А дружит с Б, то и Б дружит с А. Назовем группу участников кликой, если каждые двое из них дружат. (В частности, любая группа, состоящая менее чем из двух человек, является кликой.) Назовем количество человек в клике ее размером. Известно, что наибольший размер клики, состоящей из участников соревнования, является четным числом. Докажите, что всех участников можно рассадить в две комнаты так, чтобы наибольший из размеров клик, находящихся в одной комнате, был равен наибольшему из размеров клик, находящихся в другой комнате. 07.4 (Чехия) Биссектриса угла ВС А треугольника ABC пересекает описанную около этого треугольника окружность вторично в точке R и пересекает серединные перпендикуляры к сторонам ВС и АС в точках Р и Q соответственно. Точки К и L — середины отрезков ВС и АС соответственно. Докажите, что площади треугольников RPK и RQL равны. 07.5 (Великобритания) Положительные целые числа а и Ь таковы, что число (4а2 - I)2 делится на число 4аЬ — 1. Докажите, что а = Ь. Задачи Международных математических олимпиад 45
07.6 (Нидерланды) Пусть п — целое положительное число. Рассмотрим множество S = {(х, у9 z)\ х, у, z е {0, 1, ..., п}, х + у + z > 0}, состоящее из (п + I)3 - 1 точек трехмерного пространства. Найдите наименьшее возможное количество плоскостей, объединение которых содержит все точки множества S, но не содержит точку (0, 0, 0). 49-я олимпиада. 2008 г. 08.1 (Россия) Пусть Н — точка пересечения высот остроугольного треугольника ABC. Окружность с центром в середине стороны ВС, проходящая через точку Н, пересекает прямую ВС в точках Аг и А2. Аналогично окружность с центром в середине стороны С А, проходящая через точку Н, пересекает прямую СА в точках Вг и Б2, а окружность с центром в середине стороны АВ, проходящая через точку Н, пересекает прямую АВ в точках С1 и С2. Докажите, что точки Al9 А2, Вг, Б2, Cl9 С2 лежат на одной окружности. 08.2 (Австрия) а) Докажите, что неравенство х2 . У2 , *2 >ъ 1 (х - I)2 (у - I)2 (г - I)2 выполняется для любых отличных от 1 действительных чисел л:, г/, z таких, что xyz = 1. б) Докажите, что указанное неравенство обращается в равенство для бесконечного числа троек отличных от единицы рациональных чисел х, у, z таких, что xyz = 1. 08^3 (Литва) Докажите, что существует бесконечно много натуральных чисел п таких, что число п2 + 1 имеет простой делитель, больший числа 2п + 08.4 (Южная Корея) Найдите все функции /: (0, +оо) —► (0, +оо) такие, что (f(w))2+(f(x))2 = w*+ х2 f(y2)+f(z2) У2+ z2 для любых положительных ьи, х, у, г, удовлетворяющих равенству vox = yz. 46 Задачи Международных математических олимпиад
qajl (Франция) Пусть пик — такие натуральные числа, что к ^ я, а число к — п четное. Имеется 2п лампочек, занумерованных числами 1, 2, ..., 2 я, каждая из которых может находиться в одном из двух состояний: вкл. (включена) или выкл. (выключена). Вначале все лампочки были выключены. Рассматриваются упорядоченные последовательности шагов: на каждом шаге ровно одна из лампочек меняет свое состояние на противоположное (с вкл. на выкл. либо с выкл. на вкл.). Обозначим через N количество последовательностей из k шагов, приводящих к ситуации, в которой все лампочки с 1-й по п-ю включены, а все лампочки с (п + 1)-й по (2п)-ю выключены. Обозначим через М количество последовательностей из k шагов, приводящих к ситуации, в которой также все лампочки с 1-й по п-ю включены, все лампочки с (п + 1)-й по (2п)-ю выключены, но при этом ни одна из лампочек с (п + 1)-й по (2п)-ю ни разу не меняла своего состояния. Найдите значение отношения —. М 08JB (Россия) Пусть ABCD — выпуклый четырехугольник, в котором В А ^ ВС. Обозначим окружности, вписанные в треугольники ABC и ADC, через щ и со2 соответственно. Предположим, что существует окружность со, которая касается продолжения отрезка ВА за точку А, продолжения отрезка ВС за точку С, а также касается прямых AD и CD. Докажите, что общие внешние касательные к окружностям (дг и со2 пересекаются на окружности со. Задачи Международных математических олимпиад
1997 год 97.1 J Ответ, а) 0 для четных тип,- для нечетных тип. а) Обозначим рассматриваемый прямоугольный треугольник через ABC (ZA = 90°, АВ = т, АС = п) и достроим его до прямоугольника ABDC. Если числа тип имеют одинаковую четность, то раскраска этого прямоугольника симметрична относительно середины его диагонали ВС. Следовательно, SX(ABC) = 8г(ВСО) и S2(ABC) = S2(BCD). Значит, f(m, п) = \SiiABC) - S2(ABC)\ = = -1 Sx (ABDC) - S2 (ABDC) |. Поэтому f(m, n) = 0, если числа тип оба четные, f(m, n) = —, если числа тип оба нечетные. 2 б) Если числа тип имеют одинаковую четность, то требуемый результат немедленно вытекает из решения пункта «а». Пусть теперь т нечетно, а п четно. (Если т четно, а п нечетно, то достаточно переобозначить тип.) Рассмотрим на отрезке АВ (рис. 2) точку L, такую, что AL = т - 1. Так как число т - 1 четное, то f(m - 1, п) = 0, т. е. S1(ALC) = S2(ALC). Следовательно, f(m, n) = |Si(ABC) - S2(ABC)\ = \8г(ЬВС) - S2(LBC)\ ^ ^ S(LBC) = - ^ - max {/n, n). в) Вычислим f(2k + 1, 2k). Как и в решении пункта «б», рассмотрим на АВ такую точку L, что AL = 2k, и аналогично получим f(2k 2k) = - S2(LBC)\. 48 Ответы. Решения. Указания
Рис. 2 Площадь треугольника LBC равна k. Без ограничения общности будем считать, что отрезок LC черный (см. рис. 2). Тогда белая часть треугольника LBC состоит из треугольников BLN2k, M2k- iL2k- iN2k- i» •••» M1L1Nl9 каждый из которых, очевидно, подобен треугольнику ABC. Их суммарная площадь равна оь ((оь\2 (оь л\2 ( 2k \\2k\ ■ \2k~ 1\ + +I- 2Л+1 l2ftJ I 2k ) ■" \2k S2(LBC)=± 4k(2k+ 1) (12+22 + ... +(2/г)2) = 4k + 1 12 Значит, 12 8k- 12 и f(2k + 1, 6 Ясно, что принимает сколь угодно большие значения. 6 _97.2.j На дугах АВ и АС (не содержащих соответствен- но точки С и Б) построим точки Сх и В1 так, что АСХ = CU и АБХ = БС7 (рис. 3). (Так как угол А — наименьший угол треугольника, то ВС < АВ, ВС < АС и такие точки Вг и Сг существуют.) Тогда AU и СС1 симметричны относительно серединного перпендикуляра к АС, значит, отрезки AU и CCY равны и пересекаются в точке W. Аналогично отрезки AU и ВВг равны и пересекаются в точке V. Следовательно, отрезки ВВг и ССг пересекаются в точке Т. Агаханов Ответы. Решения. Указания 49
Рис. 3 Из равенства дуг BUC и В1АС1 вытекает, что ВСВ1С1 — равнобокая трапеция с основаниями ВСг и СВг. Из симметрии равнобокой трапеции следует, что ТВ — ТСг, поэтому ТВ + ТС = ТС1 + ТС = СС1 = AU, что и требовалось доказать. 97.3-1 Допустим, что требуемой перестановки не существует, т. е. для любой перестановки выполнено неравенство Изменив, если это необходимо, знаки чисел и их нумерацию, мы можем считать, что хг + ... + хп = 1 и хг ^ х2 ^ ... ^ хп. Рассмотрим перестановки х19 х2, ..., хп и хп, хп_1У ..., хг. Пусть Sx = хх + 2х2 + ... + пхп, S2 = хп пхг. 50 Ответы. Решения. Указания
Легко понять, что S2 ^ Sx. Действительно, если в наборе у19 ..., уп поменять местами yk и yk + 1, то при yk + 1> yk получим S'k + 1- S'k = (ух + 2у2 + ... + kyk + (k + l)yk + 1 + ... + nyn) - - O/i + 2y2 + ... + kyk + 1 + (k+ l)yk + ... + nyn) = Поэтому если мы последовательно поменяем местами числа хг и х2, х1 и х3, ..., хх и д:п, затем jc2 и д:3, ..., х2 и д:п, ..., хп_х и л:п, то из х19 х2, ..., д:л получим хпУ хп_19 ..., JCj, причем на каждом шаге рассматриваемая нами сумма не уменьшалась. Заметим теперь, что поэтому iS2 ^ ^ Sx. Но согласно предположению и I Ь2 I > , следовательно, С другой стороны, Поэтому если iS^ > —-—, то и S^ + 1 > —-— (иначе, если < , то | S'k + 1 - Sk\ > n + 1). Но S2 > , значит, dL £ о, М+1 в результате наших перестановок мы получим Sx > , что противоречит полученному ранее неравенству Sx < . Итак, накге предположение неверно и искомая перестановка существует. 97.Ж.1 а) Пусть п — натуральное число, болькгее 1. Предположим, что серебряная таблица п х п существует. Пусть k — элемент из множества {1, 2, ..., 2п — 1}, который не стоит на главной диагонали таблицы (такой элемент найдется, так как п < 2п — 1). Назовем объединение £-го столбца и i-й строки таблицы i-м крестом. Число k появляется в каждом кресте ровно один раз. Если число k стоит на пересечении i-ro столбца и у-й строки, то оно входит и в i-й, и в у-й крест. Назовем эти кресты k-связанными. Таким образом, все п крестов разбиваются на пары k-связанных, т. е. п — четное число. Но 1997 — число нечетное. 4* Ответы. Решения. Указания 51
б) При п = 2 таблица 1 2 3 1 является серебряной. Из 1 3 4 7 2 1 6 4 5 7 1 3 61 5 2 1 или (1 3 7 6 нее легко получить серебряную таблицу 4x4: 2 4 5^| 16 7 5 13 4 2 1 Конструкция первой из этих таблиц несложным образом обобщается: если А — серебряная таблица п х п, то построим таблицу D размером 2п х 2п так: Не 1 где таблица В получена из таблицы А добавлением 2п к каждому ее элементу, а таблица С получена из таблицы В заменой всех ее элементов, стоящих на главной диагонали, на 2/г. Таблица D будет серебряной. Действительно, пусть i ^ п (случай i > n разбирается аналогично). Рассмотрим i-й крест таблицы D. Он состоит из i-ro креста таблицы Ау i-й строки таблицы В и i-ro столбца таблицы С; i-й крест таблицы А содержит числа 1, 2, ..., 2п - 1, а i-я строка таблицы В и i-й столбец таблицы С содержат числа 2п, 2п + 1, ..., An - 1. 97,5-j Ответ. {(1, 1); (16, 2); (27, 3)}. Из равенства аь2 = Ъа следует, что а = Ьь2. Пусть — = —, где (fe, I) = 1. Тогда Ь 1 а = & (ь^У=ЬьЬ[ откуда kb2 *. b~r = Ъь\ (1) Если 6=1, то и а = 1. Если же 6 > 1, то из равенст- ва (1) вытекает, что — б2 = Ь1, т. е. у = Ь7 • (2) I случай. Пусть fe - 21 > 0. Тогда из равенства (2) k следует, что число целое, поэтому 1=1 (так как (fc, /) = 1), т. е. а = Ьк и /г = 6*-2. (3) 52 Ответы. Решения. Указания
Из неравенств bk~2^2k~2>k при fe ^ 5 получаем, что равенство (3) возможно лишь при k < 5. Перебором убеждаемся, что k = 4, Ъ = 2, а = 16 или k = 3, fe = 3, а = 27. II случай. Пусть k - 21 < 0. Из равенства (2) получа- I 2- — /г ем, что — = Ь 1, где 2 > 0, т. е. fe = 1. Тогда 6 = а', - 1 _ следовательно, Z ^ 22' - г9 что невозможно при I ^ 2. 97.6.1 Если п = 2k + 1 — любое нечетное число, большее 1, то каждое его представление в требуемом по условию задачи виде содержит единицу в качестве слагаемого. Убрав эту единицу, мы получим представление числа 2fe. Верно, очевидно, и обратное. Следовательно, /(2fe + 1) = /(2fe). (1) Если п = 2k — любое четное число, то каждое его представление в требуемом виде принадлежит к одному из двух типов: либо оно содержит слагаемое 1, либо не содержит таких слагаемых. В первом случае, убрав одно слагаемое 1, мы получим представление числа 2fe — 1. Как и выше, легко заметить, что есть взаимно однозначное соответствие между всеми представлениями числа 2fe — 1 и представлениями числа 2fe первого типа. Во втором случае мы можем разделить все слагаемые на 2 и получить представление числа fe. Это соответствие также взаимно однозначно. Итак, /(2fe) = /(2fe - 1) + /(fe). (2) Обе полученные формулы выполнены для всех натуральных k ^ 1. Очевидно, что /(1) = 1. Пусть по определению /(0) = 1, тогда формула (1) выполнена и при fe = 0. Заметим еще, что из формул (1) и (2) следует, что f(n) не убывает. Согласно формуле (1) число /(2fe - 1) в формуле (2) можно заменить на /(2fe - 2), откуда следуют равенства /(2fe) - /(2fe - 2) = /(fe) для fe = 1, 2, 3, ... . Суммируя эти равенства от 1 до fe, получаем, что f(2n) = /(0) + /(1) + ... + f(n)9 n = 1, 2, 3, ... . (3) В правой части равенства (3) каждое слагаемое не больше последнего, а так как 2 = /(2) ^ f(n) для п ^ 2, то f(2n) = 2 + (/(2) + ... + f(n)) ^ 2 + (п - l)f(n) ^ для п = 2, 3, 4, ... . Ответы. Решения. Указания 53
Следовательно, f(2n) ^ 2""1 • /(2"-!) < 2" "х • 2"- 2 • /(2" "2) И так как 2 2 • 2 < 2 2 для гс ^ 3, то верхняя оценка для /(2") получена. Чтобы получить нижнюю оценку, докажем сначала, что f(b + 1) - f(b) > f(a + 1) - f{a), (4) если fe ^ а ^ 0, где а, Ъ — целые числа одинаковой четности. Действительно, если числа а и Ь — четные, то из формулы (1) следует, что каждая часть неравенства (4) обращается в нуль, а если они оба нечетные, то из формулы (2) следует, что l)-f(b)= f(±±±) и Остается вспомнить, что / не убывает. Возьмем произвольные натуральные числа г и k такие, что r^k, r — четное, и в неравенство (4) подставим а = г — у, Ъ = г + j для j = 0, ..., k — 1. Сложив полученные неравенства, имеем f(r + к) - f(r) > f(r + 1) - f(r - k + 1). Так как г четно, то f(r + 1) =/(г), и, следовательно, f(r + k) + f(r - k + 1) > 2f(r) для k = 1, ..., г. Суммируя эти неравенства для k = 1, ..., г, запишем В силу равенства (3) левая часть полученного неравенства равна /(4г) — 1. Поэтому /(4г) ^ 2г/(г) + 1 > 2г/(г). Возьмем г = 2т " 2. Тогда f(2m)>2m-1f(2m-2). (5) (Заметим, что г = 2т " 2 четно при т > 2, т е N; однако неравенство (5) справедливо и при т = 2.) Наконец, пусть м > 1, п е ЛГ. Если Z — такое натуральное число, что 21 ^ /г, то, применяя неравенство (5) к т = п, п — 2, ..., л — 21 + 2, получаем, что f(2n)>2n~1 • f(2n~2)>2n-1 • 2n~s 54 Ответы. Решения. Указания
Теперь, взяв I = —, если п четно, или I = , если п нечетно, получим: f(2n)>2T- /(2°) = 2х, где п — четное число; f(2n) > 2 4 • /(21) = 2 4 • 2 > 2 4 , где п — нечетное число. Нужный результат доказан для п ^ 2. Непосредственно проверяется, что и для п—\ соответствующее неравенство справедливо. 1998 год 98.1.1 а) Докажем, что если около четырехугольника ABCD можно описать окружность, то SAPB = SCPD. Действительно, в этом случае серединные перпендикуляры к непараллельным сторонам АВ и CD пересекаются в центре описанной окружности, т. е. РА = РВ = PC = PD (рис. 4). Кроме того, ZAPB + ZCPD = 180°, так как 90° = ZAOB = = | (АВ + CD). Значит, sin ZAPB = sin ZCPD => SAPR = - PA • РВ • sin ZAPB = APB PD • sin ZCPD = S CPD. б) Докажем, что если SAPB = SCPD, то около четырехугольника ABCD можно описать окружность. Пусть это не так, тогда без ограничения общности можно записать, что РА = РВ > PC = PD. Проведем окружность радиуса РА с центром в точке Р. Пусть она пересечет второй раз прямую АС в точке К, прямую BD в точке L (рис. 5). Тогда D Рис. 4 Рис. 5 Ответы. Решения. Указания 55
точки К и С лежат по одну сторону от перпендикуляра, опущенного из Р на прямую АС (так как точки А и С, а также А л К лежат по разные стороны от него), значит, точка С лежит между точками А и К. Также точки L и D лежат по одну сторону от перпендикуляра, опущенного из точки Р на прямую BD, и точка D лежит между точками В и L. Тогда точка Р и отрезок KL лежат по разные стороны от прямой CD. Отсюда следует, что если Нг — середина KL, то отрезки РД\ и CD пересекаются в некоторой точке N. Заметим, что РНХ — высота треугольника KLP и РНХ > PN ^ РН9 где РН — высота треугольника С DP. Кроме того, CD = JOC2 + OD2 < ^ОК2 + OL2 = KL, поэтому skpl = ~ PHi' KL > \ PH ' CD = scpd- Ho из пункта «а» следует, что SKPL = SAPB, т. e. SAPB > SCPD — противоречие, значит, РА = PB = PC = PD, что и требовалось доказать. 98.2,1 Назовем тройкой двух судей и одного участника, если оценки, выставленные участнику этими судьями, совпадают. Пусть I — количество различных троек. Оценим число Z. С одной стороны, по условию для любых двух судей существует не более k троек, включающих этих судей, поэтому (где — — количество неупорядоченных пар судей). dL С другой стороны, если Ьх — количество судей, поставивших некоторому определенному участнику оценку «удовлетворительно», а Ь2 — количество судей, поставивших тому же участнику оценку «неудовлетворительно», то число троек, в состав которых входит этот участник, равно Mfri-i) ь2ф2-1) 2 + 2 • Но Ъ = Ьг + b2 — нечетное число, поэтому \ЬХ - Ь2\ ^ 1, и, значит, Mfri-1) Ь2(Ь2 - 1) = (Ь, + Ъ2У ^ фх - Ъ2У Ьг+Ь2 > Суммируя все неравенства, соответствующие всем а участникам, получаем «,..£^. (2, 56 Ответы. Решения. Указания
Из неравенств (1) и (2) следует: 2 4 a 2b что и требовалось доказать. 98.3.1 Ответ. В указанном виде представимы все нечетные числа, и только они. Докажем сначала, что k нечетно. Действительно, если п = 1, то d(ri) = d(n2) = 1 => k = 1. Если п > 1, то (разложение числа п по степеням простых чисел). Тогда о 2ri 2гч , (2(п2) п — Р\ ••• ' Ps * поэтому число k = нечетное, так d(n) как числитель этой дроби d(7i2) = (2r1 + l). ... • (2rs+l)- нечетное число. Значит, k = 2т + 1. Индукцией по т докажем, что для каждого нечетного k найдется такое п, что k = —-—-, т. е. d{n) (2r1+l).....(2rs+l) *" ( 1)( 1) " (1) Для т = 1 получим (4+1).(2+1) Пусть для всех т < М каждое число 2т + 1 представи- мо в виде дроби (1). Докажем, что число k = 2М + 1 пред- ставимо в том же виде. Пусть k + 1 = 21 • t9 где t нечетно, тогда k+ \ ^ k +1 1 так как I ^ 1 и k > 1. Рассмотрим числа г19 ..., гх вида гг = 21 2° 2° 2l + 1 21 21 2l + l1 р 1 , х г = 21 • t - 2° • t - 2°, r2 = 2l + 1 • t - 21 • t - 21, ..., rl = 2l + l~1 • t- -2l~1-t-21-1, тогда для щ = p^1 •... • p[l запишем: (2lt-2°t-2° + l)- ...-(2l+l-1-t-2l-1-t-2l 22lt-2lt-2l+l (2l- l)(2^f- 1) 2l-t- 2l- t - 2°. t - 2°+ 1 (2l-l)t t По предположению индукции так как t < fe, то найдется число п2 = g"1 -...-g"8 такое, что t представимо в виде d(n2) t = . Выбрав различные простые числа р19 ..., р19 от- d(n2) Ответы. Решения. Указания 57
личные от ql9 ..., qSJ получим, что для п = пх • п2 справедливо равенство d(n2) d(n\) d(n\) L d(n) din,) d(n2) 3 _^8^kj Ответ. (11, 1), (49, 1), (7/e2, 7fc), k e N. Если a2b + a + b делится на ab2 + b + 7, то b(a2b + a + b)- a(ab2 + b + 7) = b2-7a делится на ab2 + b + 7. I случай, b2 — 7a = 0. Тогда b = 7fe, k e N, откуда a = 7fe2. Легко проверить, что при таких а и b условие выполнено. II случай, b2- 7a>0. Тогда 0 < Ь2-7а < ab2 + b+7— делимость невозможна. III случай. Ь2 - 7а < 0. Если Ь2 - 7а делится на ab2 + b + 7, то 7a - b2 ^ ab2 + b + 7, откуда 7a > ab2, 7 > b2, т. e. b = 1 или b = 2. Если b = 1, то 7a — b2 = 7a - 1 делится на ab2 + b + 7 = = a + 8, откуда 7(a + 8) - (7a - 1) = 57 делится на a + 8. Получаем, что a + 8 = 19 или a + 8 = 57. Проверка показывает, что оба варианта подходят. Если b = 2, то 7а - Ь2 = 7а - 4 делится на ab2 + b + 7 = = 4a + 9, откуда 7 (4a + 9) - 4 (7a - 4) = 79 делится на 4a + 9, отсюда 79 = 4a + 9, что невозможно. __JHL5J Заметим, что ZRMB = ZAML = ZLKM = ZKSB (рис. 6). Аналогично ZMRB = ZSKB, отсюда вытекает подобие треугольников RMB и К SB. Из подобия следует, что 58 Ответы. Решения. Указания
BR • BS = BK • BM. Из равных прямоугольных треугольников BKI и BMI получаем BK = BM < BI, BI J_ KM. Ha отрезке BI найдется такая точка P, что ВР = BK. Имеем RS J_ ВР (так как МК \\ RS), ВР2 = BR • BS, откуда треугольник PRS — прямоугольный с прямым углом RPS. Треугольник PRS лежит внутри треугольника IRS, поэтому ZRIS < ZRPS = 90°. „JM-JLJ Ответ. 120. Положим /(1) = k. Тогда f(kt2) = if it))2 и /(/(*)) = k2t. Далее, (f(kt))2 = 1 . (f(kt))2 = f((kt)2 . /(I)) = f(kH2) = = f(k2 - (fe*2)) = fififikt2))) = k2f(kt2) = k2(f(t))2. Отсюда вытекает, что /(fef) = fe/(£) (можно выносить ft за скобку). Докажем индукцией по тг, что f(k2n "х *2#1)= (/(О)2"- Для п — \ равенство верно. Пусть оно верно для п = т, тогда 1-42m + 1)= f(k-(k2m-1t2m)2) = 1-1*2"1))2 = (fit)2"1)2 = т. е. верно для тг = m + 1. Для любого тг выполнено равенство (f(t))2n = k2n-1f{t2n), значит, (f(t))2n делится на Л2""1. Предположим, что f(t) не делится на ky тогда найдется простое число р такое, что степень а вхождения числа р в разложение k больше степени Ь вхождения числа р в раз- Ъ 2л — 1 ложение f(t). Тогда найдется такое п, что - < , откуда а 2п 2пЬ < (2п — 1) а, что противоречит делимости (fit))2*1 на k2n ~г. Значит, f(t) делится на k. Теперь можем считать, что k = 1, иначе рассмотрим функцию g: N —> N такую, что f(t) = kg(t). Для функции g верно равенство из условия задачи (так как f(t2f(s)) = f(kt2g(s)) = kf(t2g(s)) = k2g(t2g(s)) и s(f(t))2 = = k2s(g(t))2 и значение £(1998) меньше, чем /(1998). Итак, мы имеем /(1) = 1, /(/(*)) = * и f(t2) = (f(t))2. Отсюда (f(ts))2 = f(t2s2) = f(t2f(f(s2))) = f(s2)(f(t))2 = (f(s))2(f(t))2, следовательно, f(ts) = f(t)f(s) (т. е. / мультипликативна и определяется однозначно по значениям на простых чис- лах: f(p^р? ...ptk) = (/(A))ai (/(ft))a2 • • • (/(ft V»). Далее, если t > 1, то f(t) > 1, иначе t = f(f(t)) = /(1) = 1. Пусть число р — простое и число f(p) — составное: f(p) = ts, где t > 1, s > 1. Тогда р = f(f(p)) = fits) = fit)fis) — составное число, что неверно. Таким образом, для простого числа р значение f(p) = q — простое число, причем fiq)=p. Ответы. Решения. Указания 59
Наоборот, любая мультипликативная функция, задающая такое соответствие pt <-> qt на множестве простых чисел, удовлетворяет условию. Действительно, если t = P\lp22... р^\ s = pllpi2 ... plk (показатели о^, (3^ неотрицательны), то = р\1 р\г ••• Pi" (q?lQ? ••• qlkf = Имеем /(1998) = /42 • 33 • 37) = /42) (/(3))3/437) ^ pq^r (где p9 q9 r — различные простые числа), что не меньше 23 - 3 • 5 = 120. Равенство /(1998) =120 достигается для любой мультипликативной функции /, заданной взаимно однозначным соответствием простых чисел, при котором 2нЗи 5 <-> 37. 1999 год 99.1.1 Ответ. S — множества, состоящие из вершин правильного тг-угольника. Пусть G — центр тяжести множества S. Так как S переходит в себя при симметрии гАВ относительно серединного перпендикуляра к отрезку АВ (где А е S, Be S), то rAB(G) = G. Отсюда вытекает, что любые две точки множества S равноудалены от G, и, значит, все точки множества S лежат на одной окружности с центром G. Таким образом, точки множества S — вершины выпуклого многоугольника АгА2 ... Ап9 вписанного в окружность. Заметим, что rAiA переводит каждую из двух полуплоскостей, ограниченных прямой АХА39 в себя, поэтому образом точки А2 может быть только она сама. Отсюда АгА2 = А2А3. Аналогично А2А3 = А3А4 = ... = АпА19 и многоугольник АгА2 ... Ап — правильный. Наоборот, множество S, состоящее из вершин правильного м-угольника, удовлетворяет условию задачи. 99.2J Ответ. С = -. о Неравенство в задаче симметрично и однородно, поэтому можно считать, что хг > х2 ^ ... ^ хп ^ 0 и ^xL = 1. Положим ' F(xl9...9xn)= ^xtXj(xf +xj). 60 Ответы. Решения. Указания
Попробуем увеличить значение F> заменив х = (Xij ..., xkJ xk + ]_, О, ..., О) на х' = (х19 ..., хк_х, xk + xk + l9 О, ..., 0) (здесь xk + 1 — последнее ненулевое число набора, k ^ 2). Получим (xft + дс* + J + дс* + дс*+ i, то 3 2 — >—^лсй + хЛ + 1, следовательно, ^(д;') - F(x) > 0. Повторив указанную выше замену несколько раз, получим F(x) < F(a, b, 0, ..., О) = ab(a2 + Ь2) = = |(2аЬ)(1 - 2аЪ) ^ i = F [±, |, 0, ..., о). Значит, С = —. 8 Равенство выполнено тогда и только тогда, когда два числа набора х19 ..., хп равны между собой, а остальные числа равны нулю. . |(jUlj. Ответ. Пусть п = 2т. Раскрасим клетки в шахматном порядке. j-r т(т + 1) Покажем, что — минимальное количество отмеченных белых клеток. То же самое будет верно для отмеченных черных клеток, и мы получим в ответе число —— + 11. 2 V2 ) Длиной диагонали будем называть количество клеток, из которых она состоит. Выделим черные диагонали нечетной длины через одну (рис. 7). Каждая отмеченная белая клетка является соседней для не более чем двух клеток одной из этих диагоналей. Так как длины выделенных черных диагоналей равны 1, 3, 5, ..., 2т — 19 то должно быть Л о т(т+1) - отмечено не менее l + 2 + d + ... + m= белых клеток. Ответы. Решения. Указания 61
• т • • • • • • • ■ • • X X • • X X х • _\ X X • X X X d' d Рис. 8 Выделим белые диагонали нечетной длины через одну, и в каждой из них отметим белые клетки через одну, начиная от края доски (рис. 8). Каждая черная диагональ d, параллельная выделенным белым, имеет соседнюю выделенную белую диагональ d\ Поэтому для любой клетки диагонали d найдется соседняя отмеченная белая клетка на диагонали d\ Длины выделенных белых диагоналей равны 1, 3, 5, ..., 2т — 1, поэтому в нашем примере отмече- -• л о т(т + 1) ^ но ровно 1 + 2 + 3+... + m = — белых клеток. 99.4.1 Ответ. (1, р), (2, 2), (3, 3) (р — любое простое число). При п = 1 можно взять любое простое р. Если п — четно, то р - 1 нечетно, откуда р = 2 и п = 2. Найденная пара подходит. Пусть п > 1 — нечетно, q — наименьший простой делитель числа п (в частности (п, q — 1) = 1). Из условия (р — 1)п = — 1 (mod п) вытекает, что (р — 1)п = — 1 (mod q) => => (р - 1)2п = 1 (mod q). Из условия следует, что р - 1 и п взаимно просты, поэтому р - 1 не делится на д, и согласно малой теореме Ферма (р — l)q~x = 1 (mod q). Пусть у — наименьшее натуральное число, для которого (р — 1)у = = 1 (mod q). Разделим 2п на у с остатком: 2п = ky + г, О ^ г < у. Имеем 1 ее (р - 1)2* ее ((р - 1)*0* . (р - 1)г = (р _ i)r (mod g). Из определения г/ следует, что г = 0, т. е. 2га делится на г/. Аналогично q - 1 делится на i/. Но пид-1 взаимно просты, поэтому I/ = 1 или у = 2. 62 Ответы. Решения. Указания
При у = 1 получаем (р - 1) = 1 (mod g) => (p - 1)п = == 1 (mod q) => -1 = 1 (mod g) — противоречие. Пусть у = 2, тогда (р - I)2 = 1 (mod q) ^ (р - 1)-^((р- 1)2)V . (/?_ 1} s = р - 1 (mod g) => —1 = /? — 1 (mod q) => p = q. Если n > q, то в силу выбора g имеем q2 ^ ai. Из неравенства n ^ 2p = 2q получаем, что g^ 2 — противоречие. Пусть теперь n = q=p. Число (р- 1)^+1 =С£р - С2р2 + ... (здесь все слагаемые, кроме первого, делятся нар3) должно делиться на рР~ 1. Отсюда р ^ 3. Получаем р = 3, п = 3. Эта пара удовлетворяет условию. 99,5.] Лемма 1. Пусть окружность Yi касается окружности y внутренним образом в точке Р, а хорды XY окружности y в точке Q. Тогда середина R дуги ХУ, не содержащей точки Р, лежит на прямой PQ, и ее степень относительно окружности Yi равна RX2 (рис. 9). Доказательство. Гомотетия с центром Р, переводящая Yi в y> переводит точку Q в точку R' € PQ на окружности у, а прямую XY в прямую Z, касающуюся окружности у, в точке i?\ Получаем, что Z || XY, следовательно, R' — середина дуги ХУ, т. е. R = R' (точка R' совпадает с точкой R). Поскольку ZXPR = ZXYR = ZYXR, треугольники XPR и QXR подобны, откуда RP • RQ = RX2. Лемма доказана. Лемма 2. Пусть окружность ух проходит через центр S окружности у2 и общие касательные к уг и у2 касаются окружности ух в точках X и У. Тогда прямая ХУ касается окружности у2 (рис. 10). Доказательство. Из касания окружности и прямых имеем ZTXS = ZSYX, а из симметрии относительно серединного перпендикуляра к XY получаем, что ZSXY = = ZSYX. Отсюда ZTXS = ZSXY, значит, прямая XY сим- Рис. 9 Ответы. Решения. Указания 63
Рис. 10 метрична прямой XT относительно XS, т. е. XT и XY — пара симметричных относительно XS касательных к у2. Лемма доказана. Перейдем к решению задачи. Проведем хорды EF и GH окружности у, касающиеся окружностей Г\ и Г2 (рис. 11). Заметим, что прямая АВ — радикальная ось окружностей Гх и Г2, т. е. множество точек, имеющих равные степени относительно Г\ и Г2. По лемме 1 середина А' дуги EAF N Рис. 11 64 Ответы. Решения. Указания
имеет одинаковые степени (равные А'Е2) относительно Гг и Г2, поэтому А' € АВУ т. е. А' совпадает с А. Аналогично доказываем, что В — середина дуги GBH. По лемме 1 прямая МА пересекает второй раз окружность Гх в точке касания этой окружности с EF, отсюда С € EF. Аналогично D е GH. Применив теперь лемму 2 для окружностей Г\ и Г2, получим, что CD касается окружности Г2. 99.6J Ответ. f(x) = 1 - —. ........ ____■ 2 Пусть А — множество значений функции / и с = /(0). Положив х = I/ = 0, мы получим /(—с) = /(с) + с - 1, поэтому с Ф 0. Легко найти сужение функции / на множество А: взяв х = /(*/), получим для всех х е А. Основной шаг доказательства состоит в том, чтобы показать, что множество разностей х — i/, где jc, i/ g А, есть все множество R. Для у = 0 мы имеем {/(^ - с) - /(х) | * е Д} = {с* + f(c) -l\xzR} = R, поскольку с Ф 0. Теперь мы можем получить значение f(x) для произвольного х: если мы выберем yl9 y2 s А такие, что х = уг — у2> и используем (1), то получим /(*) = ПУг ~ */2) = ПУ2) + У1У2 + /(l/i) ~ 1 = с + 1 Vl с+1 У1 л О/i - У2)2 х2 /оч = +^ + 1 сс(2) Сравнивая равенства (1) и (2), получим с = 1, и поэто- му /(jc) = 1 для всех х е JR. Проверка показывает, что найденная функция удовлетворяет функциональному уравнению задачи. 2000 год 00.1.1 Пусть К = MN П АВ (рис. 12). По теореме о касательной и секущей К А2 = KN - КМ = КВ29 следовательно, К — середина АВ. Так как PQ \\ АВ9 то М — середина PQ. Поэтому достаточно доказать, что ЕМ JL PQ. Из параллельности прямых CD и АВ вытекает, что А и В — середины дуг СМ и DM, т. е. треугольники АСМ и BDM — равнобедренные. Из сказанного следует, что Z.BAM = ZAMC = ZACM = ZEAB и ZABM = ZBMD = = ZBDM - ZEBAy т. е. точки Е и М симметричны относи- 5-Агаханов Ответы. Решения. Указания 65
Рис. 12 тельно прямой АВ. Значит, прямая ЕМ перпендикулярна АВ, а следовательно, и ЕМ J_ PQ, т. е. треугольник EPQ — равнобедренный и ЕР = EQ. 00,2.j Положим а = —, b = —, с = —, где л: > 0, i/ > 0, у z х z > 0. Тогда исходное неравенство перепишется в виде (л: — I/ + г) (г/ — z + л:) (г — л: + у) ^ jci/2. (1) Одно из возможных доказательств этого неравенства таково. Заметим, что среди чисел и-х-уЛ-z^ v = у - z + х> w = z — х + у не более одного отрицательного, так как сумма любых двух из них положительна. Если отрицательное число одно, то uvw ^ 0 < xyz. Если же и ^ 0, v ^ 0, w ^ 0, то достаточно перемножить три верных неравенства: х2 > х2 - (у - г)2, у2 > у2 - (z - jc)2, Z2 ^ Z2 - (X - у)2, Замечание. При и> 0, v > 0, w > 0 неравенство (1) можно интерпретировать геометрически: в произвольном треугольнике радиус вписанной окружности не превосходит половины радиуса описанной окружности. 1 00.3.1 Ответ. X п-1 Покажем сначала, что при указанных значениях А, мы можем увести всех блох так далеко вправо, как мы того 66 Ответы. Решения. Указания
пожелаем. Будем использовать следующую стратегию: на каждом прыжке выберем самую левую блоху и заставим ее прыгнуть через самую правую блоху. После ft таких прыжков получим конфигурацию блох, для которой обозначим через dk максимальное, а через Ък минимальное расстояние между блохами. Очевидно, что dk> (п - l)bk. После (ft + 1)-го прыжка среди расстояний между соседними блохами появляется новое, равное Xdk. Может оказаться, что bk + 1 = Xdk. Если это не так, то bk+1 ^ bk. В любом из этих случаев -^- > min< I, -z^-\ ^ min{l, (n- 1) А,}. 5* I 8Л J Следовательно, если А, ^ , то bk + 1 ^ 8k для всех ft, п — 1 т. е. наименьшее расстояние между блохами не убывает. Кроме того, Ьп > 0, откуда и вытекает доказываемое утверждение. Докажем теперь, что при X < из любой начальной п — 1 конфигурации нельзя увести всех блох далеко вправо никакой последовательностью прыжков. Введем на данной прямой систему координат. Пусть sk — сумма координат всех блох после ft-ro прыжка в произвольной последовательности прыжков, & wk — наибольшая из этих координат (т. е. координата самой правой блохи). Заметим, что sk ^ nwk. После (ft + 1)-го прыжка блоха из точки А перепрыгнула через В и оказалась в С. Пусть координаты точек А, В и С равны а, Ь, с соответственно. Тогда sk + г = sk + с - а. По условию с - Ь = Х(Ь - а), т. е. Х(с - а) = (1 + X)(с - Ь). Значит, sk + i~ sk = c — а = —-— (с — Ь). Предположим, что с > wk. Блоха, которая только что прыгнула, заняла новую крайнюю правую позицию wk + 1 = c. Так как Ь — положение какой-то блохи после ft-ro прыжка, то b ^ wk. Следовательно, Эта оценка справедлива и в случае с ^ wk: u>k + i-wk=°> sk +1 - sk = с - a > О. Пусть zk -—-—wk - sk для ft = 0, 1, 2, .... Приме- Л. няя сделанную выше оценку, получаем, что zk + г - zk ^ 0. В частности, zk ^ z0 для всех ft. Здесь рассматриваем 5* Ответы. Решения. Указания 67
X < , т. е. 1 + X > пХ. В этом случае zk = (n + \i)wk — sk, п — 1 1 + X г, т-г где \х = — п > О. Поэтому из равенства Л. 2k = \LWk + (riWk - 8k) > \lWk следует, что wk ^ — для всех k. Таким образом, положение самой правой блохи не превосходит некоторой константы (зависящей от п9 X и начального расположения блох, но не от последовательности прыжков). Ответ. 12. Пусть карточка 1 (или число 1) лежит в красном ящике (сокращенно КЯ), а карточка с наименьшим числом fe, не лежащая в КЯ, лежит в белом ящике (БЯ). Тогда карточка с числом k - 1 находится в КЯ. По условию в синем ящике (СЯ) есть хотя бы одна карточка; пусть п — наименьшее число (т. е. карточка с наименьшим числом) в СЯ, т. е. п > k. Если карточка п — 1 лежит в КЯ, то зритель может вытащить либо карточки п - 1 и k из КЯ и БЯ, либо карточки п и k - 1 из СЯ и КЯ. Суммы чисел на карточках одинаковы, значит, в этом случае фокус не удастся. Следовательно, карточка п — 1 находится в БЯ. Предположим, что карточка 2 лежит в КЯ. Тогда, взяв либо карточки 2 и п - 1 из КЯ и БЯ, либо карточки 1 и п из КЯ и СЯ, получим одинаковые суммы, значит, k = 2 и карточка 2 находится в БЯ. Рассмотрим два случая. 1) В КЯ нет других карточек, кроме 1. Покажем, что тогда п = 100. Пусть п < 100. Тогда карточка п + 1 лежит либо в БЯ, либо в СЯ. Пары карточек (1, п + 1) и (2, п) с одинаковой суммой находятся в паре (КЯ, БЯ (СЯ)) и в паре (БЯ, СЯ) — фокус не удался. Значит, п = 100, т. е. в СЯ только одна карточка 100, в КЯ одна карточка 1, в БЯ карточки 2, 3, ..., 99. Покажем, что в этом случае фокус всегда удается: если мы берем карточки из БЯ и КЯ, то получаем суммы 3, 4, ..., 100, если из КЯ и СЯ — сумму 101, если из БЯ и СЯ — суммы 102, 103, ..., 199, т. е. суммы различны. 2) В КЯ есть другие числа, и т — наименьшее из них. Тогда т > 2, значит, карточка т - 1 не лежит в КЯ. Если карточка т - 1 находится в БЯ, то для пар (т - 1, п) из БЯ и СЯ и (п - 1, т) из БЯ и КЯ фокус не удастся. Значит, карточка т - 1 лежит в СЯ. Ответы. Решения. Указания
Лемма 1. Если в двух различных ящиках лежат карточки х и х + 1, а в третьем — карточки у и у + 1, то фокус не удастся. Доказательство. Одинаковые суммы имеют пары (х, у + 1) и (х + 1, у) из разных пар ящиков. Лемма 2. Если в одном ящике лежат карточки хну, а в двух других — карточки х + 1 иу+1 соответственно, то фокус не удастся. Доказательство леммы 2 дословно повторяет доказательство леммы 1. Выше мы показали, что для каждой пары ящиков есть карточки с двумя последовательными числами, а именно КЯ, БЯ: 1, 2 БЯ, СЯ: п - 1, п СЯ, КЯ: т - 1, т Значит, по лемме 1 ни в одном из ящиков нет карточек с двумя последовательными числами. Если карточка х лежит в КЯ, а карточка х + 1 лежит в СЯ, то по лемме 2 (у = 1) фокус не удастся. Аналогично фокус не удастся, если карточка х находится в БЯ, а карточка х + 1 находится в КЯ (у = п - 1) и если карточка х лежит в СЯ, а карточка х + 1 лежит в БЯ (у = т — 1). Итак, если карточка а находится в КЯ, то карточка а + 1 лежит в БЯ, карточка а + 2 — в СЯ, карточка а + 3 — в БЯ и т. д. Значит, в КЯ находятся числа, сравнимые с 1 по модулю 3, в БЯ — сравнимые с числом 2, в СЯ — делящиеся на 3. Покажем, что такое расположение карточек подходит: сумма чисел на карточках из КЯ и БЯ делится на 3, из КЯ и СЯ сравнима с 1 по модулю 3, из СЯ и БЯ сравнима с 2 по модулю 3, т. е. всегда можно определить, из каких ящиков взяты карточки. Из вышеизложенного ясно, что если карточка 1 лежит в КЯ, а карточка с наименьшим числом не из КЯ находится в БЯ, то существует два варианта раскладывания карточек. Аналогично рассуждаем в случае других пяти пар ящиков. Значит, всего имеется 2 • 3 • 2 = 12 различных способов. 00.5.1 Ответ. Существует. Докажем по индукции, что для любого натурального k существует нечетное число nk, имеющее k различных простых делителей, делящееся на 3 и такое, что 2"* +1 делится на nk. Для k = 1 можно взять п = 3. Пусть число nk = Пу кратное 3, имеет k различных простых делителей, причем 2п + 1 делится на п. Ответы. Решения. Указания 69
Число 23п + 1 = (2п + 1)(22л - 2п + 1) делится на Зл. Это следует из того, что 2п + 1 делится на я, а число 22" - 2п + 1 = (2" - 2)(2" + 1) + 3 (1) делится на 3 (поскольку при нечетном п числа 2п + 1 и 2" - 2 делятся на 3). Число 22" - 2Л + 1 не делится на 9, поскольку на 9 делится произведение (2п - 2)(2Л + 1). Значит, поскольку 22п — 2п + 1 > 3 при я > 1, то это число имеет при п > 1 простой делитель р > 3. Так как НОД(2" + 1, 22" - 2п + 1) = 3, что тоже ясно из равенства (1), то р — не делитель п. Из сказанного следует, что число Зрп имеет k + 1 простой делитель, причем 23рп + 1 делится на Зрп. Последнее следует, например, из равенства (23п)р + 1 = (2ЗЛ + ЩСг8*)*-1 - (23"К"2 + ... + 1). Для завершения решения достаточно положить nk + 1 = = Зрп = 3pnk. 00.6.| Пусть окружность со радиуса г с центром J является вписанной в треугольник ABC окружностью. Впишем в эту окружность треугольник А'В'С, гомотетичный треугольнику ABC с отрицательным коэффициентом (рис. 13). Докажем, что прямые ll912> Is содержат стороны треугольника А'В'С'\ отсюда сразу будет следовать решение задачи. Заметим, что IB9 = IC\ ZB'IC = 2ZB'A'C" = 2ZA, откуда следует, что расстояние от J до прямой В'С равно /Б' cos A = r cos А. Н, С В " Рис. 13 Ответы. Решения. Указания
н Рис. 14 Треугольники АН2Нг и ABC подобны (с коэффициентом cos А), поэтому прямые Н2Н3 и В'С \\ ВС составляют равные углы с биссектрисой угла ВАС, а также равные углы с прямой Т2Т3 _L А/. Впишем окружность со' с центром /' в треугольник АН2Н3 (рис. 14). Из подобия треугольников АН2Нг и ABC следует, что радиус окружности со' равен г cos А. Если X — точка касания со' со стороной АВ (1'Х _1_ АВ), то в силу того же подобия АХ = AT2cos А, значит, в треугольнике АХТ2 угол АХТ2 прямой. Отсюда Г е ХТ2 => ГТ2 ± АВ => ГТ2 || 1Т3. Аналогично 1'Т3 \\ 1Т2, следовательно, I'T2IT3 — параллелограмм с перпендикулярными диагоналями, т. е. ромб. Итак, точки /' и / симметричны относительно Т2Т3, прямые Н2Н3 и В'С составляют равные углы с Т2Т3 и удалены соответственно от /' и / на одно и то же расстояние г cos А. Отсюда вытекает, что прямые Н2Н3 и В'С симметричны относительно Т2Т3, т. е. В'С совпадает с прямой 1Х. 2001 год 01.1 ,| Проведем в окружности, описанной около треугольника ABC, диаметр MN, параллельный стороне ВС (рис. 15). По условию 60° ^ ВМА - CNA = МА - NA= 180° - 2NA9 откуда NA ^ 60°. Это означает, что точка А лежит на дуге A0N, где Ао — точка описанной окружности, лежащая Ответы. Решения. Указания 71
Рис. 15 со стороной ВС по разные стороны от MN и такая, что ZA0ON — 60°, т. е. треугольник A0ON правильный (здесь О — центр описанной окружности). Точка Р лежит на отрезке СР0, где Ро — проекция Ао на ВС, поэтому ZCAB + ZCOP ^ ZCAB + ZCOP0. Заметим, что ZCAB = -ВС = - (MN - CN - ВМ) = - (180° - 2CN) = = 90° - ZCON. Тем самым для решения задачи достаточно показать, что ZCOP0 < ZCON. Пусть А0Р0 пересекается с ON в точке К. Так как А0Р0 _L ON, то А0К — высота правильного треугольника A0ON, откуда ОК = KN. Далее, Р0С < KN = ОК < ОР0. Из треугольника СОР0 получаем ZCOP0 < ZOCP0 (против большей стороны лежит больший угол), но ZOCP0 = ZCON, т. е. ZCOP0 < ZCON. 01.2.1 Так как выражение в левой части однородно относительно а, Ь и с (т. е. f(a, Ь, с) = f(ka, ХЬ, А,с)), то можно считать, что аЪс = 1. Из равенства аЪс = 1 следует, что all She Sabc 8 8 8 Пусть 1н = х, 1-\ = и, 1 + — = z, тогда нужно до- a3 b3 с3 казать неравенство <=> xy + xz + yz + 2y]x2yz + 2yjxy2z + 2-yJxyz2 ^ х^г <=> x^ + xz + z/2 + 2y[xyz y-Jx + -yfy + -Jzj > xyz. (1) 72 Ответы. Решения. Указания
Теперь, применив неравенство о среднем арифметическом и среднем геометрическом, находим 8 раз поэтому л[х ^ —-. Аналогично следовательно, и л/х + ^ + л/г ^ 3 • ^/7^ ^ 3 • ^27 = 9. Поэтому для доказательства неравенства (1) достаточно показать, что ху + xz + yz + 2 • 27 • 9 > xz/z. (2) о о о Положим — = А, — = Б, —- = С, тогда неравенство (2) а Ь3 с примет вид (1 + А)(1 + Б) + (1 + А)(1 + С) + (1 + Б)(1 + С) + 486 ^ ^ (1 + А)(1 + Б)(1 + С) <=> А + Б + С + 488 ^ А • Б • С. Но А - Б • С = -^— = 83, отсюда (abcf А + В + О 3-VA-B-C = 24, и, значит, А + Б + С + 488 ^ 512 = 83 = А - Б • С. 01-3-1 Исключим из рассмотрения те задачи, которые никто не решил. Обозначим через А множество задач, каждую из которых решили не более двух мальчиков, а через А остальные задачи олимпиады, т. е. решенные не менее чем тремя мальчиками. Аналогично для девочек определим множества Б и Б. Если бы некоторый мальчик решил задачи только из множества Б, то не более 12 девочек решили бы общие с ним задачи (хотя бы по одной), что противоречит условию задачи. Значит, каждый мальчик решил хотя бы одну задачу из множества Б и тем самым не более пяти задач из множества Б. Поэтому у каждого мальчика не более чем с 10 девочками общие задачи из множества Б и, следовательно, не менее_чем с 11 девочками общие задачи только из множества Б. (Если у некото- Ответы. Решения. Указания 73
рой девочки общие с ним задачи как из В, так и из В, то она в число этих 11 девочек не входит.) Для всех мальчиков мы получили множество М из не менее 21-11 пар «мальчик — девочка» с общими задачами только из множества В. Точно так же рассматривая девочек, получаем множество N из не менее 21-11 пар «девочка — мальчик» с общими задачами только из множества А. Всего число пар «мальчик — девочка» равно 21 • 21, следовательно, какая-то пара «мальчик — девочка» входит как в множество М, так и в множество N, т. е. у_них_есть общая решенная задача, входящая в множества В л А. Это означает, что найденную задачу решили по крайней мере 3 девочки и по крайней мере 3 мальчика. Утверждение доказано. 01.4.1 Просуммируем все S(a). На i-м месте каждое из чисел от 1 до п встретится по (п — 1)! раз. Поэтому сумма выражений S(a) для всех п\ перестановок есть M = (k1(l + ... + n) + k2(l + ... + n) + ... + kn(l + ... = (/1-1)1 2 t(kl + ... + kn) = = п!^-^(^ + ... + kn)= 0(modn!), п+1 так как из нечетности п следует, что целое число. Предположим, что искомых перестановок не существует. Тогда остатки от деления всех S(a) на п\ различны. Всего этих остатков п!, поэтому Таким образом, (nl+l)nl Л/ , ,ч (тг!+1)тг!. . п\ . , * ^— = 0 (modn!) =» ^ ^— : п\ =* — -. п!, так как числа п\ + 1 и п\ взаимно просты. Получили противоречие; значит, искомые перестановки бис существуют. 01.5.1 Ответ. ZABC = 80°, ZACB = 40°. Пусть ZABC = 25. Отложим на продолжении отрезка АВ за точку В отрезок ВХ, равный ВР, а на продолжении отрезка AQ за точку Q отрезок QY, равный BQ (рис. 16). Из условия следует, что АХ = AY; точки X и У симметричны относительно биссектрисы АР угла ВАС, поэтому PX = PY, ZPXB = ZPYQ. Кроме того, ZABQ = ZCBQ = = ZBPX = ZBXP = 5. Из равенств BQ = QY и ZQBP = ZQYP следует, что лучи ВР и YP симметричны относительно биссектрисы угла BQY. Рассмотрим два случая. 74 Ответы. Решения. Указания
Рис. 16 1) Пусть прямые ВР и YP совпадают. Тогда совпадают точки У и С, ZACB = 5, ZABC + ZACB = 35 = 120°, откуда 8 = 40°. 2) Если прямые ВР и YP различны, то их точка пересечения Р лежит на биссектрисе угла BQY, отсюда PY = РВ. Получаем, что ВХ = ВР = РХ =» ZPBX = 60°, значит, АС — противоречие. 01.6.1 Пусть x = & + d + a-c, тогда х > 1 (так как 5 + d + a-c>ft + d>l)HC = a + ft + d, d = с - а - Ь (здесь и далее сравнения по модулю х). Отсюда следует, что 0 = х(Ь + d - а + с) = ас + bd = а(а + Ь + d) + 6d = (a + b)(a + d) и 0 = ac + bd = ас + 6(с - a - Ъ) = (a + Ъ)(с - 6), т. е. (a + b)(a + d) I jc, (a + b)(c - b) \ x. Возможны два случая. 1) (a + b) • x => (a + b) ! (a + Ь + d - с). Но с > d, поэтому a + b > x. С другой стороны, в силу Ь > с имеем 2* = 2а + 2Ь + 2d - 2с> 2а + 2d > 2а > а + Ь. Итак, 2х > а + b > х и (а + Ь) • х, что невозможно. 2) (а + Ь) /х. Пусть х = р"1 • ... • p%k — разложение х на простые множители. Тогда существует i (I ^ i < й) такое, что (а + Ь) /pta/. Отсюда, учитывая, что (a + b)(a + d) • jc, л: i р"*, получаем (а + d) ! р^. Аналогично (Ь — с) • р£. Следовательно, ab -\- cd = ((a + d)b - (b - c)d) \ pt, т. е. ab + cd не является простым, так как ab + cd > ас + bd ^ х ^ pt. Ответы. Решения. Указания 75
2002 год 02,1.1 Рассмотрим всевозможные подмножества i?cT красных точек, удовлетворяющие условию задачи. Пусть kt — число синих точек с абсциссой £, 1} — число синих точек с ординатой у (i, у e {0, 1, ..., п - 1}). Докажем индукцией по количеству точек в множестве i?, что упорядоченные наборы К = (ft0, kl9 ..., kn _ х) и L = = (Zo, ll9 ..., ln_i) получаются один из другого перестановкой. Если R = 0, то К = L = (п9 п - 1, ..., 1). Предположим, что утверждение доказано для всевозможных множеств из (т — 1) красных точек (т ^ 1). Рассмотрим некоторое подмножество Д из m красных точек. Выберем красную точку (i, j) e R с наибольшей суммой i + у. Перекрасив эту точку в синий цвет, мы перейдем к множеству красных точек R' из (т — 1) точек, причем легко видеть, что R удовлетворяет условию. В наборе К для множества R при переходе к R' изменилось лишь kt = п — 1 — (i + j) на k[ = n — (i + у), а в наборе L изменилось лишь lj = п - 1 - (i + у) на Ц = п - (i + у) (на рисунке 17 точки обозначены так: • — красные, о — синие). Остается заметить, что количество Х-множеств равно kok1k2...kn_1, а количество У-множеств равно lolih---ln-i- Рис. 17 г — У1 К с л \ л л 0 и > f l Л v Л \ Л 1 Я xl ч 1 g ч J V 1 j' 1 f % J V L J 12 3 и b L N N N N H fn-l I I X 02-2,1 Из условия (рис. 18) следует, что OF = FA, ОЕ = ЕА9 и, кроме того, OF - О А = ОЕ = R, где R — радиус окружности Г. Значит, треугольники OF А и ОЕА правильные. Отсюда Z.AOE — Z.AOF — 60°, следовательно, точка Е 76 Ответы. Решения. Указания
Е Рис. 18 лежит на дуге АС, не содержащей точки В (так как ZAOB < 120°). Заметим, что ZECA = -EA = - ZAOE = 30°, ZACF = - AF = - ZAOF = 30°, т. е. С А — биссектриса угла ECF. Далее, OD — биссектриса внешнего угла равнобедренного треугольника АОС, отсюда OD || АС =» ADOJ — параллелограмм => AJ = OD = R = АЕ. Таким образом, точка J однозначно определена равенством AJ = АЕ и тем, что она лежит на луче АС. Так как центр Т окружности, вписанной в треугольник CEF, лежит на СА, то осталось доказать, что AT = АЕ. Пусть биссектриса FT пересекает окружность в точке X (рис. 19). Тогда из равенства дуг АЕ и AF, а также дуг ХС и ХЕ вытекает, что /.TAX — Z.EAX и Z.TXA = ZEXA, откуда ААТХ = ААЕХ, т. е. AT = АЕ. Е Рис. 19 Ответы. Решения. Указания 77
02.3.1 Ответ, т = 5, п = 3. Пусть Р(х) = хп + х2 - 1, Q(x) - хт + х - 1. Если т ^ п, то при а > 1 имеем 0 < ат + а - 1 < ап + а2 - 1 и, значит, qTTI _|_ д ^ число не целое. Поэтому т > п. Коэффициенты ап + а2- 1 многочленов Р(х) и Q(x) — целые, и старший коэффициент многочлена Р(х) равен 1, поэтому Q(x) = Д^*) • Р(х) + Д2(х), где degi?2 < п = degP (deg / — степень многочлена /), и многочлены Rx (x) и R2 (x) имеют целочисленные коэффициенты. Далее, из равенства Ria) + Р(а) 1У ' Р{а) следует, что Р(а) Р(а) Но так как степень многочлена R2 меньше степени многочлена Р, можно выбрать такое число N, что |i?2(a)| < Р(о) при всех а > N. Если R2 Ф 0, то многочлен R2 имеет конечное число корней, поэтому можно выбрать число М > N такое, что R2(a) Ф 0 при а > М. Но тогда при а> М число ° '"' е (-1; 0) U (0; 1) Р(а) не является целым, что противоречит условию. Значит, R2 = 0. Поэтому нужно найти такие тип, что многочлен Q(x) делится на многочлен Р(х). Если Q(x) ■ Р(х), то любой корень многочлена Р (х) является корнем многочлена Q(x). Но Р(х) = 0 <=> хп + х2 = 1. На отрезке [0; 1] функция f(x) = хп + х2 является возрастающей, /(0) = 0, /(1) = 2 > 1. Значит, сп + с2 = 1 при некотором с е (0; 1). Тогда Q(c) = 0 => ст + с = 1. Нос + сп + 1> > с2 + сп при любом с е (0; 1), так как с + сп+1 - с2 - сп = с(1 - с - с71'1 + сп) = = с(1 - с)(1 - сп~х) > 0. Значит, т Ф п + 1 => т> п + 2. Тогда т > 5 при п = 3. Если m = 5, то х5 + х - 1 = (х3 + х2 - 1)(х2 - х + 1), т. е. пара п = 3, m = 5 подходит. При т ^ 6 получаем сб + с<с5 + с=1, так как с5 + с = с3 + с2 = 1, значит, пары п — 3, m ^ 6 не подходят. Пусть теперь п ^ 4. Если п четно, то Р (с) = 0 =» Р (-с) = 0, поэтому Q(c) = Q(-c) = 0, т. е. ст + с = 1 и (-с)т - с = 1. Это невозможно при нечетном т: (-с)т - с - -(ст + с) = -1. 78 Ответы. Решения. Указания
При четном т получаем (ст + с =1 п ст-с=1)=>с = О — противоречие. Значит, п нечетно, т. е. п > 5. Покажем теперь, что т < 2п. Пусть с - 1 - е, е е (0; 1). Тогда с2 = 1 - 2е + е2, сп = 1 - с2 = 2е - е2, ст = 1 - с = е. Если т ^ 2п, то г=1 - с = ст^с2п = (2г- г2)2 « е ^ е2(2 - е)2 « « 1 ^е(2-е)2<4е^ 1, если е ^ -. Значит, е > -, но тогда 4 4 с < - => сп + с2 < с5 + с2 $ 4 ~ 64 16 ~ 64 < Следовательно, т < 2п, т. е. т = п + к, где 2 ^ й ^ ^ п - 1. Тогда — xk(xn + х2 — Л\ + (Л — jrHjr*n -I- jri —11 Поэтому из делимости Q(x) на Р(х) следует, что (1 - Х)(ХЛ(1 + X) - 1) : (Хл + X2 - 1). Но многочлены 1 — хихл + х2 — 1 взаимно просты, так как 1 не является корнем многочлена хл + х2 — 1. Значит, (Xk(l + X) - 1) : (Хл + X2 - 1), откуда deg(xfe(l + х) - 1) ^ deg(xn + х2 - 1), т. е. к + 1 ^ и <=> к ^ ть — 1.. Но к <, п — 1, значит, к — п — 1. Получаем <=> (хл + хл -г - 1) ; (хл + х2 - 1) <=> « (хл-х - х2) ; (хл + х2 - 1), т. е. п — 1 = 2, п = 3 — противоречие. Значит, п = 3, m - 5 — единственная подходящая пара. 02.4-1 Ответ, б) п — любое простое число. тва — = dk + 1_t следует, ч dt D — dxd2 + d2ds + ... + dk _ Из равенства — = dk + 1_t следует, что dk-\dk Ответы. Решения. Указания 79
а) Докажем, что ь ... + < 1, откуда и будет d\d2 dh_Ydk следовать неравенствоD<n2. Действительно, di^di_1+ 1 => =» dt> i, где i = 1, ..., й, поэтому ^ + ... + ^^_JL_ + __J__ + ...+ dk x{dk !+ 1) 1-2 2-3 /г2 б) Если D — делитель п2, то— = m, где т е N. Значит, TT Ж. О 111 Предположим, что k > 2, тогда — > = —, значит, m d1d2 d2 m < d2. Но из пункта «а» следует, что т > 1, поэтому т содержит простой делитель р, т. е. р ^ т < d2 и п2 • m :p=»n • р => d2<: р. Но d2> m ^Р — противоречие. Итак, fe < 2, но м > 1, значит, й = 2, т. е. п — простое (^ = 1, d2 = ti). Проверкой убеждаемся в том, что любое простое п подходит: D=l-n = nnn2 • п. O2L5J Ответ. f(x) = 0, f(x) = |, f(x) = х2. В данное равенство (f(x) + f(z))(f(y) + f(t)) = fixy - zt) + fixt + yz) (1) подставим x = y = z = t = O. Имеем 4 (/(О))2 = 2/(0), откуда /(0) = 0 или /(0) = |. Пусть /(0) = —. В равенство (1) подставим у = t — 0, х = z = а е R. Имеем 2f(a) = 1, т. е. f(a) = — при всех а е R. Функция f(x) = - — один из ответов. Пусть /(0) = 0. В равенство (1) подставим z = t = 0. Получим f(x)f(y) = f(xy), т. е. функция / мультипликативна. Тогда из равенства (1) следует равенство fixy) + f(yz) + f(xt) + f(zt) = = f(xy - zt) + f(xt + yz). (2) 80 Ответы. Решения. Указания
Сделав в равенстве (2) замены х <-» 2, у <-> t, получим /(г*) + f(xt) + /(z/г) + /(jcz/) = = f(2t - ХУ) + f[yz + **)• (3) Сравнивая равенства (2) и (3), получаем f(xy - zt) = - f(zt - ху). Но в виде ху - zt = а можно представить любое число a е R (х = а, у = 1, г = 0). Итак, f(a) = /(-а), т. е. функция / четна. Далее, из мультипликативности / имеем f(x2) = (f(x))2 ^ 0. Но в виде а = х2 можно представить любое неотрицательное число а. Значит, f(a) > 0 при а > 0. Но f(-a) = f(a) =» f(x) > О при всех х g R. Теперь в равенство (2) подставим х = а9у = J—, г = vab, t = 1, где а > О, Ъ > 0. Тогда ху - zt = л[аЬ9 xt = a, z/г = 6, и получаем /(а) + /(6) + 2f(Jab) = /(0) + f(a + 6) = /(а + 6), так как /(0) = 0. Из последнего равенства с учетом того, что f(a) = 0(л^))2, fib) = (/О))2, f(4ab) = следует, что т. е. в силу неотрицательности / Отсюда так как Пусть £(*) = д//(л:), jc ^ 0. Тогда ^(a) + g(b) = g(a + 6), т. е. функция g аддитивна. С другой стороны, ё(ху) = yjfixy) = Jf(x)f(y) = V/W • 4Ш = g(x)g(y), т. е. функция g мультипликативна. Таким образом, функция g аддитивна и мультипликативна. Выведем из этого, что либо g = 0, либо g(x) = х. Итак, функция g удовлетворяет равенствам y) = g(x) + g(yh (4) g(xy) = g(x)g(y). (5) В равенство (5) подставим х = у = 1. Получим g(l) = 12 откуда ^(1) = 1 или £(1) = 0. Ответы. Решения. Указания 81
Если g(l) = 0, то g(x) = g(x . 1) = g(x) . g(l) = О =* g(x) = 0 =* f(x) = 0. Если g(l) = 1, то из равенства (4) следует, что g(m) = = mg(l) = m при любом т е JR. Кроме того, m — т. e. g(r) = г для любого r e Q, r > 0. Из равенства (4) и неотрицательности g(z/) при у ^ 0 следует монотонность (возрастание) функции g. Но g(x) = х при г g Q, г > 0. Значит, g(jc) = jc при jc ^ 0. Действительно, пусть g(x) Ф х при некотором х > 0. Возьмем число г е Q между числами jc и g(jc). Тогда g(r) = г, а пары (г, х) и ((g(r), ^(jc)) упорядочены по-разному, что противоречит монотонности функции g. Итак, если g(l) = 1, то g(x) = х при х ^ 0. Отсюда f(x) = х2 при jc ^ 0. Но /(jc) — четная функция, следовательно, f(x) = х2 при всех х е R. J02-6J Заметим, что окружности не пересекаются и (поскольку sin х < х при х > 0) величина меньше радиан- ной меры угла КОр} (рис. 20), где ОД и OtL — касательные к окружности Гу. Будем обозначать через Vl} угол KOtL9 через W^ объединение угла Vij с вертикальным ему углом, через Ф (F/y) ради- анную меру угла Vijy Ф(Т^у) - 2Ф(^у). Тогда 1 К 2 Рассмотрим выпуклую оболочку центров данных окружностей. При этом три центра не могут лежать на одной прямой, так как в противном случае проходящая через них прямая пересекает три окружности. Следовательно, в выпуклой оболочке k вершин, где k ^ 0. Обозначим через Р19 Рис. 20 82 Ответы. Решения. Указания
Р2у ..., Pk верн1ины выпуклой оболочки по порядку, например по часовой стрелке, а через Qft + i> •••» Qn вершины, не вошедшие в выпуклую оболочку. Пусть Д = {1, ..., ft}, S = {k + 1, ..., n}. Предположим, что S не пусто, т. е. k < п. Выберем произвольно / е S. Углы Wij9 j Ф I, пересекаются только в вершине QL, так как в противном случае прямая, проходящая через Qt и общую точку этих углов, пересекает три окружности. Поэтому 4Д4 Сложив эти неравенства по всем /eS, получим 1 . V""4 1 , -ш ч 7С (1) Выберем теперь произвольно I е R. Рассмотрим углы Vlj9 j * I. Они все лежат внутри угла, образованного касательными из точки Pt к окружностям с центрами PL _ г иР/ + 1 (здесь и далее Ро = РЛ, Pk + 1 = рг)9 лежащими вне выпуклой оболочки (рис. 21). Поэтому если обозначить углы i, /б Д, через ah то Отсюда /-1, /, / + Рис. 21 6* Ответы. Решения. Указания 83
Сложив эти неравенства по всем / е R и воспользовавшись равенством ^ at = п (к — 2), получим m * f ± 1; т, I <е R /ей, t <е S 2 Но ^- < Ф(У;у.), значит, +2 " т, le R t (E S Оценим первую из сумм. Проведем общие внутренние касательные к окружностям с центрами Р^_ l9 Ph Pl + l9 как показано на рисунке 22. Пусть они пересекаются в точке W и пересекают отрезки РЬРЬ _ i и Р^Рг + г в точках V и U соответственно. Тогда ZUWV < п9 так как если ZUWV^ я, то найдется прямая, пересекающая все три окружности с центрами Pt_l9 Pt и Р1+1 (рис. 23). Итак, получаем, что ZWVPl + Z.PJJW + а, < тг, т. е. ZWTP, + ZHT7Pj < тг - а,. Из равенства радиусов окружностей следует, что V — середина отрезка Р1Р1_1У поэтому . /ТЖ7ТЛП 2 1 sin ZWVPt ZWVPt ZWVPt ZWUPt n - 1 < Рис. 22 84 Ответы. Решения. Указания
Рис. 23 Сложив полученные неравенства по всем I e R, получим L /ей k так как ^at = (k- 2)я. Итак, i = i ., Сложив неравенства (1), (2) и (3), получим i,jeS je R iJeR ^ n t n(k- 2) t n(n- k) л ^ + += Сумма в левой части полученного неравенства равна i поэтому ^ —{п— 1), что и требовалось до- 4 казать. 2003 год 5?bJLJ Пусть k e A, jcz g A, x больше 101 D — множество разностей xk — xl9 где k^ xt. Число элементов множества Z> не 100 = 10100. Пусть Т = {tl9 t2, ..., S б tm) — поду {l 2 m) множество множества S, состоящее из наибольшего возможного числа пг элементов с условием tt — t} £ D при i ^ j. Тогда для любого t e S\T можно выбрать tt e T и d e D такие, что t - tv = d. Число пар (ti9 d) не больше, чем 10100/п. С другой стороны, оно должно быть не меньше, чем число элементов множества S\T. Получаем 10100т > 1000000 - /п, откуда m > 99. Так как из равенства xk + tt = xt + t} следу- Ответы. Решения. Указания 85
ет, что 11 — tj g £>, то множества At = {x + tt\ x e А} попарно не пересекаются. Это означает, что tl9 ..., t100 — искомые числа. ^3,2j Ответ, (ft, 2ft), (8ft4 - ft, 2ft), (2ft, 1), ft е ЛГ. Пусть п = 2а&2 - Ь3 + 1, тогда из условия следует, что п е N и а2 ! п. Отсюда 2а&2 = Ь3 - 1 (mod л) => 2а2&2 = а(Ь3 -1) (mod л) => => а(63 - 1) =0 (mod п) => а(263 - 2) =0 (mod n). Но 2а&3 - ф4 - Ь) = 0 (mod п) => 2а = б4 - & (mod n). Итак, (б4 - 6 - 2а) : п. (1) По условию п = (2а — Ь)Ь2 + 1 > 0, откуда 2а — Ъ ^ 0. I случай. 2а-Ь = 0, что дает серию (ft, 2ft). II случай. 2а — Ъ > 0, т. е. 2а — Ъ ^ 1. Тогда из условия а2 : п следует: а2 ^ п = (2а - Ь)Ь2 + 1 =» а2 ^ Ь2 + 1 => а> Ь. Значит, 2а-&>а=>п> аб2 + 1 > аб2 => а2 > а&2 => а> Ь2. Следовательно, п = (2а - Ъ)Ъ2 + 1 > (2&2 - Ъ)Ъ2 + 1 = = 264 - Ь3 + 1 > Ь4 > б4 - Ъ - 2а. Тогда из утверждения (1) следует, что б4 — Ъ — 2а < 0. Случай Ь4 - Ъ - 2а = 0 дает серию (8ft4 - ft, 2ft). Если же Ь4 - Ъ — 2а < 0, то из утверждения (1) получаем -(б4 - Ъ - 2а) > п = 2аЪ2 - б3 + 1 => =» 2а(1 - б2) ^ б4 - б3 - Ь + 1 = (6 - 1)(63 - 1). При Ъ > 1 имеем 2а(1 - &2) < 0 < (Ь - 1)(63 - 1). Значит, 6=1, что дает серию (2ft, 1). 03.3.1 Пусть диагонали AD, BE и FC шестиугольника ABCDEF попарно пересекаются в точках Р, Q, R (рис. 24), а точки М и N — середины сторон АВ и DE соответственно. Предположим, что Z.APB = ZDPE ^ 60°. Тогда Р лежит внутри или на окружности со, описанной около равностороннего треугольника АВХ, построенного на стороне АВ внутрь шестиугольника (рис. 25). Окружность со находится л/3 внутри окружности Q радиуса MX = — АВ с центром М; 86 Ответы. Решения. Указания
Рис. 24 Е N окружности со и Q касаются внутренним образом в точ- л/З ке X. Отсюда следует, что МР < — АВ, причем равенство достигается, только если Р совпадает с X. Аналогично л/3 NP < —DE, причем равенство достигается, только если треугольник DPE правильный. Имеем — (АВ + DE) = MN < МР + NP^—AB + —DE, л/3 л/3 откуда МР =— АВ, NP = —DE и треугольники АВР и DEP правильные. Рассуждая аналогично, получаем, что ZBQC = ZEQF и ZCRD = ZFRA и каждый из этих углов не может быть больше 60°. Так как сумма ZAPB + ZBQC + ZCRD = 180°, то все углы между диагоналями AZ), BE и FC равны по 60°. В таком случае, как мы доказали, треугольники АВР, DEP и аналогично треугольники BCQ, CDR, EFQ и FAR правильные. Отсюда сразу следует утверждение задачи. Рис. 25 Ответы. Решения. Указания 87
03.4,1 Заметим, что точки С, £>, Р, Q лежат на одной окружности с диаметром CD, так как ZCQD = ZCPD = 90° (рис. 26). Поэтому по теореме синусов PQ = CD sin ZPCQ. Аналогично RQ= ADsinZRAQ =* *« = RQ ADsinZRAQ Рис. 26 Ho sin ZPCQ = sin ZBCA, так как эти углы либо равны, либо в сумме дают 180°. Аналогично • у ту лгл • у ту ал sin ZPCQ sin ZBCA AB sin Z.RAQ = sin ZBAC => — = — = —— sin ZRAQ sin ZBAC ВС по теореме синусов для треугольника ВАС. Значит, PQ _ CD AB RQ~ AD' ВС9 °ТКУДа AB Пусть BL и DM — биссектрисы треугольников ABC и ADC соответственно. Тогда ABAL АГ^ AM И ВС ~ CL И CD ~ СМ Значит, что и требовалось доказать. Замечание. Условие того, что четырехугольник ABCD вписанный, как видно из приведенного решения, является лишним. 03.5-1 Доказываемое неравенство можно переписать в виде 88 Ответы. Решения. Указания
Пусть xt + i - xt = yt> О, тогда Xj~ Xt = (Xj~ Х;_г) +(Х;_г ~Xj_2)+ ... +(Xi+1-Xt) = = Vi+ ••• + l/y-1 и для n + 1 переменной jcx, ..., лгл + 1 неравенство (1) примет вид £ J <2) n k= i Левая часть неравенства (2) содержит z/m, если £ < т < у, поэтому при фиксированном m в левой части неравенства (2) слагаемое ут встречается т(п - т + 1) раз, т. е. левая часть равна ( У Заменив каждое слагаемое ^ yh\ на (J' — i + I)2 слагаемых вида j \2 г^ 9 в правой части неравенства (2) получим / = ^ (п — s + 1) s2 слагаемых, так как число сумм ^ yk> co- держащих s слагаемых, равно п — s + 1. Можно показать, что / = _ л 2 . Применим к сумме А = а( + ... + a в правой части неравенства (2) неравенство А получим k=i Заметим, что В при фиксированном т содержит т(п - т+ 1)(п +1) г> — слагаемых ут. оначит, Ответы. Решения. Указания 89
т. е. неравенство доказано. Осталось заметить, что оно обращается в равенство тогда и только тогда, когда аг = ... = а,<=> уг = ... = уп, т.е. когда х1, ..., хп + 1 образуют арифметическую прогрессию. jl§J Лемма 1. Пусть а е iV, a ^ 2, k е iV, / е iV, тогда где (т, п) — наибольший общий делитель чисел /пип. Доказательство. Докажем лемму индукцией по k + /. При k + I = 2 утверждение очевидно. Пусть для всех k > /, таких, что к + К п, утверждение леммы верно. Тогда при k + / = п + 1 выполняются равенства (ak - 1, а1 - 1) = (а* - а', а' - 1) = (al(akl- 1), а' - 1) = = (а*-<- 1, а1- 1) = а(*'0_ i = а(*. о _ 1. Лемма доказана. Возьмем М = рР~1=рР'1+рР~2 + ... + 1 =» М^ 2, так р - 1 как /?^2. Лемма 2. Существует простой делитель s числа М, такой, что s Ф 1 (mod /?2). Доказательство. Предположим, что для любого простого делителя s числа М выполняется условие s = I (mod p2). Пусть М = рг- р2- ... • Pk* гДе Pi — простые числа, тогда Pi = 1 (mod /?2) => М = 1 (mod /?2) — противоречие, так как М=1+р+р2-А = 1+рФ 1 (mod p2). Лемма доказана. Из леммы 2 следует, что число М имеет простой делитель q такой, что (q - 1) / р2. Докажем, что тогда q — искомое число. Из того, что (рр — 1) ■ q, следует (р, q) = 1, поэтому (р?"1 - 1) ; q (малая теорема Ферма). Из леммы 1 имеем (р^'У-1) - 1) ; q. Число р простое, поэтому либо (q - 1) • р, либо (р, g - 1) = 1, и тогда (р - 1) I q> что невозможно, так как если р = 1 (mod g), то М = р^-1 + ...+р+1 = 1 + ... + 1=р=1 (mod q) => =» М / q. Значит, (q — 1) • р и при этом (# — 1) /р2. Отсюда q = 1 + pm, m /p. Предположим, что выбранное g не подходит. Тогда существует п е Z, такое, что -р) \ q=$ (n, <?) = 1, 90 Ответы. Решения. Указания
так как (р, q) = 1. Отсюда по малой теореме Ферма (п<*- 1 - 1) I q => (пРт - 1) : q => 1 = n^m = (nP)m = рт (mod g), так как пр = р (mod g). Значит, (pm - 1) • q. Но {р> - 1) ■ g, и по лемме 1 (р(т, р) _ 1) : ду где (m? p) = 1? поэтому (р — 1) • д, что невозможно, как было показано выше. Следовательно, для любого п е Z получаем (пР - р) X q. 2004 год 04,1.1 Пусть AL — биссектриса треугольника ABC. Докажем, что окружности, описанные около треугольников BMR и CNR, пересекаются в точке L (рис. 27). Заметим, что О — центр окружности, проходящей через точки В, Му N, С. Значит, треугольник MON равнобедренный (ОМ = ON) и биссектриса угла MON является серединным перпендикуляром к отрезку MN. Опишем окружность около треугольника AMN и обозначим через X. середину той дуги MN, которая не содержит точку А. Отрезки MX и NX равны как хорды, стягивающие равные дуги, следовательно, точка X лежит на серединном перпендикуляре к отрезку MN. С другой стороны, из равенства дуг MX и NX следует равенство вписанных углов МАХ и Рис. 27 Ответы. Решения. Указания 91
NAX, поэтому точка X лежит на биссектрисе AL. Итак, мы получаем, что точка X лежит одновременно и на биссектрисе угла ВАС, и на биссектрисе угла MON, т. е. точка X совпадает с точкой R. Из того, что четырехугольники BMNC и AMRN вписанные, следуют равенства 180° - ZMRL = ZARM = ZANM = ZABC, значит, точки В9 М, й, L лежат на одной окружности. Аналогично точки С, N, R, L также лежат на одной окружности. 04.2.1 Ответ. Р(х) = ах4 + рд:2, где а е R, p e R. В решении используем следующий факт: если многочлены Р(х) и Q(x) таковы, что P(t) = Q(t) для любого действительного t9 то Р и Q совпадают как многочлены (т. е. у них равны коэффициенты при соответствующих степенях х). Для доказательства рассмотрим разность R(x) = = Р(х) — Q(x). Если бы многочлен R(x) имел хотя бы один ненулевой коэффициент, то он имел бы конечное число корней — противоречие. Многочлен Р(х) = 0, очевидно, удовлетворяет условию и присутствует в ответе. Пусть Р(х) = апхп + ап _ гхп ~ г + ... + агх + а0 (ап ^0) — ненулевой многочлен, удовлетворяющий условию. Подставим а = t9 Ъ = с = 0 в равенство из условия задачи (для таких а, Ь, с выполнено ab + be + са = 0). При t = 0 имеем ЗР(0) = 2Р(0), откуда Р(0) = а0 = 0. При любом действительном t получаем P(t) + Р(0) + Р(-0 - 2P(t) => Р(0 = Р(-0- Отсюда следует, что все коэффициенты Р(х) при нечетных степенях х равны 0, в частности п четно. Далее, подставим а = 3t, Ь = 6t, с = —2t (при этом ab + be + са = (18 - 12 - 6)t2 = 0), получаем P(-3t) + P(8t) + + Р(-5*) = 2Р(7*). В равенстве Р(-Здг) + Р(8х) + Р(-5д:) = = 2Р(7лг) приравняем коэффициенты при старшем члене х левой и правой частей: ап(-ЗГ + ап • 8" + ап(-5)л = 2ад • 7", с учетом четности п получим Зп + 8П + 5п = 2 • 7» или j^|j + ^j + ^|j= 2. При /г ^ 8 левая часть последнего равенства больше, чем При п = 6 равенство Зд + 8Д + 5п = 2 • 7п невозможно, в чем легко убедиться и не прибегая к вычислениям, заметив, Ответы. Решения. Указания
что числа З6, 86, 56 при делении на 7 дают остаток 1 (это следует, например, из малой теоремы Ферма), поэтому число З6 + 86 + 56 при делении на 7 дает остаток 3. Итак, остается случай п ^ 4, который приводит (так как а0 = 0, аг = а3 = 0) к многочлену вида Р(х) = ах4 + (Зд;2. Остается проверить ответ. Рассмотрим Р(а - Ь) + Р(Ь - с) + Р(с -а)- 2Р(а + Ь + с) = = Р((а - Ъ)2 + (Ъ- с)2 + (с - а)2 - 2(а + Ь + с)2) + + а((а - Ь)4 + (Ь - с)4 + (с - а)4 - 2(а + Ь + с)4). Первая скобка равна —4(аЬ + Ьс + са) = 0. Преобразуем вторую скобку с учетом равенства (а + Ь + с)2 = а2 + Ь2 + с2. Получим (а - Ь)4 + (Ь - с)4 + (с - а)4 - 2 (а2 + Ь2 + с2)2 = = (а4 - 4а3Ь + 6а2Ь2 - 4аЬ3 + Ь4) + (Ь4 - 4Ь3с + 6Ь2с2 - 4Ьс3 + с4) + + (с4 - 4с3а + 6с2а2 - 4са3 + а4) - 2 (а4 + Ь4 + с4 + 2а2Ь2 + 2Ь2с2 + + 2с2а2) = 2(а2Ь2 + Ь2с2 + с2а2) - АаЬ(а2 + Ь2) - 4bc(b2 + с2) - - 4са(с2 + а2) = 2(а2Ь2 + Ь2с2 + с2а2 + 2а2Ьс + 2Ь2са + 2с2аЬ) - - 4аЬ(а2 + Ь2 + с2) - 4Ьс(а2 + Ь2 + с2) - 4са(а2 + Ь2 + с2) = = 2(ab + Ьс + са)2 - 4(аЬ + Ьс + са)(а2 + Ь2 + с2) = 0. JD4.3.J Ответ. Прямоугольники тхп, для которых m • 3, 72 : 4 (ИЛИ /I : 3, ТП : 4), И ПРЯМОУГОЛЬНИКИ ГП X 71, ДЛЯ КОТОРЫХ m • 12, /г Ф 1, 2, 5 (или п • 12, /тг ^ 1, 2, 5). Пусть прямоугольник тхп замощен крюками. Для каждого крюка А рассмотрим клетку, три стороны которой граничат с клетками крюка (рис. 28). Назовем такую клетку выделенной для крюка А. Она покрыта каким-то другим крюком Б. Так как крюк Б не пересекается с крюком А, возможно 3 варианта его расположения (рис. 29), причем третий вариант отпадает, потому что на доске остается непокрытая клетка. В оставшихся вариантах крюк А покрывает выделенную клетку крюка Б. Следовательно, все крюки разбиваются на пары крюков, которые покрывают выделенные клетки друг друга. Каждая такая пара крюков образует фигуру из 12 клеток одного из двух видов — «кирпич» 4x3 или «зигзаг». r—i—I 1—i - —■—i—' — i 1 ••-■•■ ••■. 1 LJ Hi- 1 ! \ i i i i i .'-.■ I Рис. 28 Рис. 29 "93 Ответы. Решения. Указания
Рис. 30 Итак, прямоугольник т х п разбит на кирпичи и зигзаги из 12 клеток. В частности, отсюда следует, что тп делится на 12, т. е. одно из чисел т9 n делится на 3. Пусть для определенности т ■ 3. Рассмотрим три возможных случая. I случай. Пусть числа тип четные, но не делятся на 4. Тогда общее количество равное кирпичей V 12 ) и зигзагов нечетно. Кирпич может быть ориентирован двумя способами — горизонтально и вертикально, зигзаг может быть ориентирован четырьмя способами (два вертикальных и два горизонтальных) (рис. 30, 31). Закрасим в прямоугольнике тх п каждую четвертую вертикальную полосу (рис. 32). Общее количество закрашенных клеток четно, так как в каждой полосе их четное число. Заметим, что в каждом вертикальном кирпиче и горизонтальном зигзаге 0, 2 или 4 закрашенные клетки, зна- ! i 1 4- | 1 I l вертикальные ill i i i | i 1 1 1 1 1 ГОрИЗОл. 1 eui-onmt: II II Рис. 31 Рис. 32 94 Ответы. Решения. Указания
чит, все вертикальные кирпичи и горизонтальные зигзаги накрывают четное число закрашенных клеток. Значит, все горизонтальные кирпичи и вертикальные зигзаги накрывают четное число закрашенных клеток. Но в каждом горизонтальном кирпиче или вертикальном зигзаге ровно 3 закрашенные клетки, поэтому суммарное количество горизонтальных кирпичей и вертикальных зигзагов четно. Рассматривая аналогичную раскраску горизонтальными полосами, докажем, что суммарное количество вертикальных кирпичей и горизонтальных зигзагов четно. Итак, мы получаем, что общее количество кирпичей и зигзагов четно — противоречие. II случай. Пусть п • 4. Тогда прямоугольник т х п разбивается на кирпичи. III случай. Пусть т \ 4. Тогда т • 12, т. е. т = 121. Если п молено представить в виде п = Зх + 4у с целыми неотрицательными х и у, то прямоугольник т х п разбивается на х прямоугольников 12/ х 3 и у прямоугольников 12Z х 4, каждый из которых разбивается на кирпичи. Представим в виде Зх + 4у все п Ф 1, 2, 5: 3 = 3-1 + 4-0; 4=30 + 41; 8=30 + 42. Все оставшиеся п получаются из чисел 3, 4, 8 прибавлением нескольких троек. Итак, если п Ф 1, 2, 5, то разбиение на крюки возможно. В прямоугольники т х 1 и т х 2 невозможно поместить ни одного крюка. Наконец, предположим, что прямоугольник т х 5 удалось замостить кирпичами и зигзагами. Но тогда две фигуры, покрывающие две угловые клетки в строке длины 5, обязательно перекрываются — противоречие. 04.4.] Предположим противное: пусть при некоторых l^i<j<k^n числа ti9 tj, tk не являются длинами сторон треугольника. Обозначим тройку ti9 tj, tk как a, ft, с, где а + ft ^ с. Докажем для чисел х > у > г > 0 вспомогательное неравенство — + — > - + -. После эквивалентных преобразова- 2 X 2 у ний получим ~2 2 "" у X9 2 Ху Домножая на общий знаменатель, получаем верное неравенство _, Ху(Х -у)> 22(Х- у). Неравенство —I— ^ — Н— доказано. ^ 2 X 2 у " Ответы. Решения. Указания 95
В частности, получаем с , а . а+ Ъ а с Ъ . а + Ъ , Ъ - + -> + и - + -> ——- + -. ас а а + Ъ Ъ с Ъ а + Ъ Как частный случай доказанного при z = у9 получим JC U известное неравенство —Ь — ^ 2 для положительных х и у. Имеем aj ^a + b a + bj Ъ а ) а+Ъ (Фактически, мы решили задачу для п = 3.) При раскрытии скобок в произведении t2 получается п слагаемых вида 11 — = 1 и слагаемых вида I — + — I для всевозможных 1 ^ k < I ^ /г. Каждое слагаемое вида I — + — I не меньше 2, а кроме того, сумма трех —I— | + | —I— | + | —I— I Ъ а) \с Ъ) \а с) слагаемых I —I— | + | —I— | + | —I— I не меньше 7 = 2-3 + 1. \Ъ ) Ъ) \ ) Значит, сумма всех слагаемых вида —Ь — не меньше, чем удвоенное количество слагаемых плюс 1. Итак, что противоречит условию. 04.5.j Пусть четырехугольник ABCD вписан в окружность (рис. 33). Продолжим отрезки ВР и DP до пересечения с окружностью в точках D' и В' соответственно. Так как равные вписанные углы опираются на равные дуги, то 96 Ответы. Решения. Указания
Рис. 33 D равны дуги АВ и СВ\ а также равны дуги AD и CD'. Следовательно, точки В и В', а также точки D и D' симметричны относительно серединного перпендикуляра I к отрезку АС. Значит, отрезки BD' и DB' симметричны относительно Z, поэтому их точка пересечения Р лежит на I, т. е. равноудалена от Л и С. Наоборот, пусть теперь дано, что АР = СР9 т. е. Р лежит на серединном перпендикуляре I к отрезку АС (рис. 34). Предположим, что четырехугольник ABCD не является вписанным. Тогда окружности сов и со^, описанные соответственно около треугольников ABC и ADC, различны. Пусть прямая BD пересекает окружность сов в точках В и Dl9 а окружность (0D в точках D и Вг. Обозначим через В\ D\ B[, D[ образы точек В, D, Bl9 Dx соответственно при симметрии относительно I. Очевидно, что точки В\ D\ B[,D[ лежат на одной прямой, точки В' mD[ лежат на окружности соБ, а точки В[ и D' лежат на окружности сор. Из симметрии следует, что дуги ADX и CD[ окружности сов равны, поэтому ZABDX = ZCBD[. Но по условию ZABDX = ZCBP, поэтому BD[ проходит через точку Р. В силу симметрии B'DX проходит через точку Р. Аналогично Рис. 34 D в А Ответы. Решения. Указания 97
Рис. 35 доказываем, что прямые DB[ и D'BX проходят через точку Р. Отметим, что по условию прямые ВР и BD различны, поэтому прямые BD и B'D' не проходят через точку Р. Пусть ZABC + ZADC < 180°. Так как ABCD — выпуклый четырехугольник, то отрезок BD пересекается с отрезком АС. Отсюда следует, что точки В и Вг лежат по одну сторону от прямой АС9 а точки D и Dx — по другую. Заметим, что ZADXC > ZADC9 поэтому точка Dx лежит внутри окружности С0р. Отсюда следует, что точка DX лежит на прямой BD между точками В и D. Аналогично точка Вг тоже лежит между точками В и D. Итак, порядок следования точек на прямой — В9 Bl9 Dl9 D, в частности, точка Dx лежит на отрезке BXD. В силу симметрии точка D[ лежит на отрезке B[D'. Углы BXPD и B[PD' в симметричных относительно I треугольниках BXPD и B[PD' либо вертикальные (если точка Р лежит на отрезке DB[)9 либо совпадают (если точка Р лежит на продолжении отрезка DB[) (рис. 35). Так как точка Dx лежит на отрезке DBl9 то в любом случае прямая PDX пересекает отрезок D'B[ (а не его продолжение). Получаем, что точка Б' = PDX П D'B[ лежит на отрезке D'B[ — противоречие. Случай ZABC + ZADC > 180° разбирается аналогично, лишь точки В и Bl9 D и Dl9 В' и В[, D' и D[ поменяются ролями. 04.6-1 Ответ. Все числа, не кратные 20. Любое число, делящееся на 20, имеет последнюю цифру 0 и четную предпоследнюю цифру. Следовательно, полосатое число не может делиться на число п, кратное 20. Докажем по шагам, что все остальные числа подходят. Шаг 1. Существует нечетное ft-значное полосатое число Xk (в качестве первой четной цифры допускается 0), делящееся на 5Л. Докажем индукцией по fe. Положим Хг = 5. 98 Ответы. Решения. Указания
Пусть имеется ft-значное полосатое число Xk = bkm. Чтобы получить число Xk + l9 подберем подходящую цифру а и припишем ее слева к числу Xk9 т. е. будем искать Xk+ г в виде aXk = 10ka + Xk = 5k(2ka + m). Нам нужно, чтобы число Sa = 2ka + т делилось на 5. Кроме того, если ft нечетно, то цифру а выберем четной (чтобы Xh + г было полосатым). Покажем, что это возможно. Числа 0, 2, 4, 6, 8 дают различные остатки от деления на 5, поэтому пять чисел So, S2, S4, S6, S8 дают различные остатки от деления на 5 (значит, среди этих остатков есть 0). В самом деле, если бы какие-то St и Sj (i Ф у) давали одинаковый остаток от деления на 5, то их разность SL - Sy = 2k(i - j) должна была бы делиться на 5, но это не так. Если лее ft четно, то искомую цифру а выберем нечетной. Аналогично доказываем, что это возможно, используя то, что числа 1, 3, 5, 7, 9 дают различные остатки от деления на 5. Шаг 2. Существует fc-значное полосатое число Yky делящееся на 2k, причем если k четно, то число Yk делится на 2k + г, а если k нечетно, то число Yk не делится на 2k + 1. Докажем индукцией по fe. Yx = 2. Пусть имеется fe-значное число Yk9 удовлетворяющее данным условиям. Определим Yk + 1. Пусть k нечетно, k = 2t - 1. Тогда Y2i_ г = 22t~ lm, где т нечетно. Получим полосатое число Y2t = aY2t_l9 подобрав нечетную цифру а. Запишем Y2t = 102i" га + Y2t _ ! = 22t ~ 1 (52< - 1а + т). Для доказательства нам требуется, чтобы 52i ~la + m делилось на 4. Заметим, что 5 в любой степени дает остаток 1 от деления на 4, поэтому если т = 1 (mod 4), положим а = 3, а если т = 3 (mod 4), положим а = 1. Пусть k четно, ft = 2t. Тогда Y2t = 22t + 1m. Получим полосатое число Y2t + 1aY2t, подобрав четную цифру а = 2Ь. Имеем Y2t + 1 = 1Q2ta + Y2t = 22t + 1 (52tb + m). Нам нужно, чтобы 52tb + m не делилось на 2. Для этого положим Ъ = 1 (т. е. а = 2), если т нечетное, и Ь = 2 (т. е. а = 4), если т четное. Шаг 3. Существует нечетное полосатое число, делящееся на 5kp (ft = 0, 1, 2, ...), где НОД(р, 10) = 1. Пусть X — нечетное полосатое число из четного количества цифр (возможно, с первой цифрой 0), делящееся на 5Л (если ft = 0, то положим X = 01, если ft четно, то пусть X = Xk из первого шага, если же ft нечетно, то пусть 7* Ответы. Решения. Указания 99
X = Xk + ! из первого шага). Нужное полосатое число Z, делящееся на 5кр, найдем в виде Z = XX...X = Х(1 + 10* + 102* + ... + 10'*), 1 + 1 где k — количество цифр в числе X. Достаточно доказать, что найдется I, для которого число 1 П(1 + 1) k _ 1 Sz = 1 + 10* + 102* + ... + 10z* = 1U —- делится на p. Для доказательства воспользуемся теоремой Эйлера'. если НОД(а, 6) = 1, то а*(Ь) - 1 делится на Ь, где ф(Ь) — количество натуральных чисел, меньших Ъ и взаимно простых с Ъ. Положим Z+l = (p((10* — l)p), тогда число 10(/ + 1)/г - 1 делится на 10z + l — 1, а число 10/ + г — 1 в силу теоремы Эйлера делится на (10* — 1)р. (Молено обойтись и без помощи теоремы Эйлера: заметим, что найдутся такие различные 1г> 12, что SZi и Slz дают один и тот же остаток при делении на р. Тогда число Sh- Sh = 10<z2 + 'i)* + ... +10'1/г = 10<|2 + |i)*Sll_l2 делится нар, т. е. SZi_Zz делится нар.) ИГаг 4. Существует полосатое число, делящееся на 2кр (к = 0, 1, 2, ...), где НОД(р, 10) = 1. Доказательство аналогично третьему шагу с использованием построенного на втором шаге полосатого числа, делящегося на 2*. Шаг 5. Существует полосатое число, делящееся на 2- 5*р, гдеНОД(р, 10) = 1. Достаточно приписать 0 в конце нечетного полосатого числа, делящегося на 5*, полученного на третьем шаге. 2005 год 05.1.| Запишем векторное равенство АгА2 + А2в\ + ВгВ2 + В2С\ + СгС2 + С2А\ = 0. Заметим, что сумма векторов АгА2, ВгВ2, СгС2 равна 0, так как, если последовательно отложить их от некоторой точки, получится равносторонний треугольник. Следовательно, сумма векторов А2Вг, В2С1У С2Аг также равна 0, поэтому если эти векторы последовательно отложить от некоторой точки, то получится треугольник, причем равносторонний, так как длины векторов равны. Следовательно, прямые А2Вг, В2Сг, С2Аг образуют правильный треугольник, откуда вытекает подобие треугольников АСгВ2, ВАгС2 и СВгА2 (рис. 36). Более того, эти треугольники рав- Ответы. Решения. Указания
Рис. 36 ны, поскольку А2Вг = В2Сг = С2Аг, и, значит, они совмещаются поворотом на угол 120° вокруг центра О треугольника ABC. Так как точки Al9 Bl9 Сг переходят одна в другую при повороте на 120° вокруг точки О, то треугольник А1В1С1 правильный и О — его центр. Так как ВгАг = С1А1 и ВгВ2 = СгВ29 то АгВ2 — серединный перпендикуляр к В1С1, поэтому АгВ2 проходит через О. Аналогично ВгС2 и СгА2 проходят через О. 05.2.j Если для пары номеров т < п выполнено равенство ат = ап9 то числа ат и ап при делении на п дают равные остатки — противоречие. Таким образом, в последовательности каждое целое число встречается не более одного раза. Предположим, что целое число N не встретилось в последовательности. Так как среди членов последовательности бесконечно много положительных и отрицательных чисел, то найдется такое t, что среди чисел аи а2, ..., at есть как большее N> так и меньшее N. Пусть ХиУ соответственно наименьшее и наибольшее среди чисел а1у а2, ..., at; X < N <Y. Среди (У - X + 1) целых чисел промежутка [X, У] есть не больше чем (У - X) членов последовательности, так как N не встречается в последовательности. Значит, среди чисел а19 а2, ..., ау_х+1 найдется число ak9 лежащее вне промежутка [X, У]. Тогда либо |аЛ-Х|, либо |аЛ-У| не меньше У-Х+1. Пусть для определенности s = \ak - Y\ > У - X + 1. Тогда среди чисел а19 а2, ..., as нашлись два числа (ak и У), дающие равные остатки при делении на s, что противоречит условию. Замечание. Из решения легко следует, что для каждого п множество первых п членов последовательности является множеством п последовательных целых чисел. Ответы. Решения. Указания 101
05.3-j (Решение Ю. Борейко (Молдова) удостоено спецприза ММО.) Заметим, что 1 1 (*3 - 1)Q/2 + z2) JC5 + у2 + Z2 Х3(Х2 + у2 + Z2) JC3(JC5 + у2 + Z2)(jt2 + I/2 + 22) ' поэтому - х2 х2 _ х2(х3- l)2Q/2 + z2) JC5 + у2 + 22 JC3(jt2 + I/2 + 22) JC3(JC5 + у2 + Z2)(jt2 + I/2 + Z2) Значит, jc5 - х2 | г/5 - у2 | г5 - г2 > хъ + у2 + г2 г/5 + г2 + jc2 г5 + jc2 + у2 " ХЪ_ Х2 + у5 _ у2 ^ 25 _ 22 JC3(JC2 + I/2 + 22) 1/3(JC2 + у2 + 22) 23(JC2 + у2 + 1 f 2 1 ^ 2 1 2 I"! = о—т h^ — + у — + z — • JC2+l/2+22^ X У у Z) Но так как — ^ i/г, — ^ гд:, — ^ jci/, то х * у у z vy х2 + у2 + z2 ^ х2 + у2 + z2 - ху - yz - zx = х * у z * у у = Ых - у)2 + (у- z)2 + (г - х)2) > 0. 05.4.1 Ответ. 1. Достаточно показать, что для любого простого числа р найдется такой номер п, что число ап делится на р. Прир = 2 положим /1=1, тогда аг = 10 — делится на 2. При р = 3 положим /1 = 2, тогда а2 = 48 — делится на 3. При р > 3 возьмем п = р — 2. Согласно малой теореме Ферма, если натуральное а не делится на р, то ар~1 при делении на р дает остаток 1. Поэтому число 6ап = 6(2р~2 + ЗР-2 + 6Р-2 - 1) = 3-2^1 + 2-3^-1 + 6^-1-6 при делении на р дает остаток 3-1 + 2-1 + 1 — 6 = 0. Итак, 6ап делится на р, откуда ап делится на р, так как р и 6 взаимно просты. _ 05.5.| Пусть прямые AD и ВС пересекаются в точке Н; положим для определенности, что Н лежит на лучах DA и СВ (рис. 37). Пусть К — середина дуги окружности, описанной около треугольника BHD> не содержащей точку Н; очевидно, ВК = DK. Четырехугольник BHDK — вписанный, поэтому ZKDA = ZKBC. Треугольники KDA и КВС равны по двум сторонам и углу между ними (DA = ВС по условию), поэтому К А = КС9 ZAKD = ZCKB. Отсюда 102 Ответы. Решения. Указания
Рис. 37 ZAKC = ZAKB + ZCKB = ZAKB + ZAKD = ZBKD. В равнобедренных треугольниках AJfC и BKD углы при вершинах равны, следовательно, и углы при основаниях равны, значит, ZKAP = ZKDP и точки A, D, К, Р лежат на одной окружности, отсюда Z(KP9 АР) = Z(KD, AD). Предыдущие рассуждения останутся без изменений, если точки А, С, Р заменить соответственно на Fy E> Q. Отсюда вытекает, что точки F, Dy К, Q лежат на одной окружности и Z{KQy FQ) = Z{KDy FD). Получаем, что Z(KP, RP) = Z(KP, АР) = Z(KD9 AD) = Z(KD9 FD) = = Z(KQ, FQ) = Z(KQ9 RQ), поэтому точки К, Ry P, Q лежат на одной окружности. Итак, К — общая точка всевозможных окружностей, проходящих через точки Р, Q, R. Замечание 1. Из решения вытекает следующее описание искомой точки: точка К — центр поворота, переводящего точки A, Fy D соответственно в точки С, Еу В. Замечание 2. Из решения следует, что К является точкой Микеля пяти прямых AD9 ВС, AC, BD> EF> т. е. лежит на окружности, описанной около треугольника, образованного любой тройкой из этих прямых. 05.6,1 Занумеруем всех участников олимпиады числами от 1 до п и представим результат олимпиады в виде таблицы, которая содержит п строк и 6 столбцов. На пересечении £-й строки и у-го столбца таблицы стоит плюс, если £-й участник решил задачу номер у. По условию нет строки с шестью плюсами. Для каждой из пар (у, k) столбцов, 1 ^ у < k ^ 6, определим параметр by k, равный количеству Ответы. Решения. Указания 103
строк, на пересечении которых с у-м и k-м столбцами стоят плюсы. По условию Ь} k > - п, поэтому где ai k — целое неотрицательное число. Предположим, что утверждение задачи неверно, т. е. строк с пятью плюсами не более одной. Добавим, если возможно, в каждой строке столько плюсов, чтобы в (п — 1) строках стало по 4 плюса, а в оставшейся строке стало 5 плюсов (пусть, скажем, в этой строке нет плюса только в столбце номер q). При этом условие задачи не нарушится. Сосчитаем теперь количество Р пар плюсов, находящихся в одной строке. Суммируя по строкам, получаем р = 6(/i - 1) + 10 = 6/г + 4, так как в строке с четырьмя плюсами 6 пар плюсов, а в строке с пятью плюсами 10 пар. Суммируя по всем парам столбцов, получаем, что "«1,2 + аиз + ... + а56 = 6п + 15-15J-/ii +«1,2 + ^1,3+ ... + а5,б- {И Если \-п\ < -, то Р ^ &п + 15 - 15 • | = &п + 6 > [ 5 J 5 5 Заметим, что дробная доля < — п> может принимать значе- п 1 2 3 4 ния 0, -, -, -, -. о о о о \-п\ < -, [ 5 J 5 > &п + 4 = Р — противоречие. Если же \-п\ = -утоп = 2 (mod 5). Положим п = 51 + 2, [5 J 5 тогда Р = &п + 15 - 15 • - + аг 2 + аг 3 + ... + аъ 6 = о = &п + 3 + аг 2 + #!, з + ••• + а5, б — 6/1 + 4. Значит, ровно одно из 15 чисел аг 2, #i, з> •••» а5, б равно 1, a остальные равны 0. 2 Пусть для определенности а1ш2— 1> тогда Ьх 2 = — п + 1 - - | + 1 = 21 + 2, а &!, з = Ъ1ш 4 = ... = Ь5, б = 2/ + 1. 5 Пусть в £-м столбце s^ плюсов. Обозначим через ^ количество пар плюсов, находящихся в одной строке, для которых один из плюсов содержится в i-м столбце. С одной 104 Ответы. Решения. Указания
стороны, tt есть сумма тех пяти чисел Ь} k, для которых ; = /" или i = fe, т.е. *, = 5(2/+ 1)= 10/+'5 при i Ф 1, 2, и^1 = ^2 = 10/ + 6.С другой стороны, суммируем tt по строкам: каждая из st строк, содержащих плюс в £-м столбце, содержит 3 нужные пары, если всего в этой строке 4 плюса, и содержит 4 нужные пары, если всего в этой строке 5 плюсов. Таким образом, tq = 3sg и tt = 3st + 1 при i Ф q. Итак, с одной стороны, среди чисел tl9 t2, ..., t6 два числа равны, а оставшиеся четыре числа на 1 меньше. С другой стороны, ровно одно из этих чисел делится на 3 — противоречие. 2006 год 06,1-1 Из условия вытекает, что ZABC + ZACB = ZPBA + ZPCA + ZPBC + ZPCB = = 2(ZPBC + ZPCB), откуда ZBPC = 180° - (ZPBC + ZPCB) = 180° - ZABC + ZACB = 180° - (ZIBC + ZICB) = ZBIC. Поскольку точки Р и J лежат в одной полуплоскости относительно ВСУ то точки В9 Iy P, С лежат на одной окружности (рис. 38). Рис. 38 Ответы. Решения. Указания 105
Пусть X — точка пересечения AI с окружностью, описанной около треугольника ABC. Как известно, XI = ХВ = = ХС (см., например, решение задачи 02.2). Значит, X — центр окружности, проходящей через В, J, С. На этой окружности также лежит точка Р. Так как прямая AI проходит через центр этой окружности, то AI — минимальное из расстояний от точки А до точек этой окружности, причем АР > AI в случае, если точка Р не совпадает с J. 06.2.1 Ответ. 1003. Пусть концы некоторой диагонали d разбивают границу многоугольника Р на две части, содержащие соответственно k и 2006 — k сторон, где k <, 1003. Число k назовем длиной диагонали d (сторону многоугольника Р считаем диагональю длины 1). Таким образом, хорошие диагонали (или стороны) — это диагонали нечетной длины. Рассмотрим разбиение многоугольника Р на треугольники 2003 диагоналями. Треугольник из разбиения назовем хорошим у если он равнобедренный и имеет две стороны нечетной длины. В хорошем треугольнике есть две равные боковые стороны (каждая длины I) и основание, длина которого равна 21 или 2006 - 2Z, т. е. четна. Итак, в хорошем треугольнике боковые стороны имеют нечетную длину, а основание — четную длину. Оценим число хороших треугольников. Лемма. Пусть диагональ d длины k из рассматриваемого разбиения делит многоугольник Р на две части. Тогда в меньшей из частей имеется не более — хороших треугольников. (Меньшей назовем часть, не содержащую центр многоугольника Р. Если k = 1003, т. е. d проходит через центр, то меньшей объявим любую из двух частей. Если k = 1, то меньшая часть — отрезок.) Доказательство. При k = 1 утверждение леммы очевидно — в меньшей части вообще нет треугольников. Применим индукцию по fe, т. е. предположим, что лемма верна для диагоналей длины 1, 2, ..., k - 1, и рассмотрим меньшую часть Q для некоторой диагонали АВ длины k ^ 2. В разбиении имеется треугольник АВС9 лежащий в Q. Часть Q разбивается на треугольник ABC и меньшие части R и S для диагоналей АС и ВС (рис. 39). Так как часть Q меньшая, то длины диагоналей АС и ВС меньше k и дают в сумме k. Пусть длины диагоналей АС и ВС равны а и Ь, где а + Ь — k. По предположению индукции в частях R и S соответственно не более — и — хороших треуголь- 106 Ответы. Решения. Указания
Рис. 39 ников. Если треугольник ABC не является хорошим, то в части Q хороших треугольников не больше чем Если же треугольник ABC хороший, то его равными сторонами могут быть только АС и ВС. Тогда а = Ь = — нечетно, и в части Q хороших треугольников не больше чем Переход обоснован, и лемма доказана. Рассмотрим в разбиении треугольник KLM> внутри или на границе которого содержится центр многоугольника Р. Многоугольник разбит на треугольник KLM и меньшие части для диагоналей KL> LM> MK (рис. 40). Пусть длины диагоналей KL> LM> MK равны соответственно х> у, z> где х + у + z = 2006. Воспользуемся леммой. Если треуголь- Рис. 40 М Ответы. Решения. Указания 107
ник KLM не является хорошим, то в разбиении Р хороших треугольников не больше чем [f Если же треугольник KLM хороший, то два из чисел х> у, z нечетны (пусть, скажем, х и у нечетны). Тогда в разбиении Р хороших треугольников не больше чем Приведем пример с 1003 хорошими треугольниками. Занумеруем вершины (подряд по часовой стрелке) числами 1, 2, ..., 2006 и соединим диагоналями попарно вершины 1 и 3, 3 и 5, ..., 2005 и 1. Эти диагонали отрезают 1003 хороших треугольника, а оставшийся 1003-угольник можно разбить на треугольники произвольно. 06.3.1 Ответ. ^. 32 Заметим, что левую часть неравенства можно разложить на множители так: |(Ь - а)(с - Ь)(а - с)(а + Ь + с)\. Это выражение симметрично относительно переменных а, Ь, с, поэтому положим для определенности, что a ^ b ^ с. Обозначим s± = (b- a)2, s2 = (с - b)2, s3 = (а - с)2, s = (a + b + с)2, k = а2 + Ь2 + с2. Легко видеть, что sx + s2 + s3 + s = 3k. Заменим b на среднее арифметическое чисел а и с, т. е. а ~ь с рассмотрим тройку а', Ь', с', где а' = a9b' = , с' = с. Очевидно, Sg = (а' - с')2 = ss. Кроме того, s^ < s^Sg, поскольку а также sx + s2 > s[ + Sg, поскольку (b - a)2 + (c - b)2 = = (c - a)2 - 2(b - a)(c - b) = (c' - a')2 - 2(b - a)(c - b) > > (c' - a')2 - 2(b' - a')(c' - bf) = (b' - a')2 + (c' - b')2. Теперь тройку чисел a', b\ с' сдвинем на некоторое число х9 т. е. рассмотрим тройку а" = а' + х, Ъ" - Ъ' + х, с" = с' + х. Ясно, что s{'= (&" - а")2 = 8[9 s% = {с" - Ь")2 = s'2, S3 = (с" ~ а")2 - S3- При фиксировании а', Ъ\ & квадратичная функция у = (а' + Ь' + с' + Зх)2 принимает все неотрицательные значения, поэтому за счет выбора х добьемся, чтобы s" = (а" + Ь" + с")2 = (а' + V + с' + Зх)2 стало равным 108 Ответы. Решения. Указания
числу 3k - (s" + Sg'+ S3 ) (число 3k - (s{'+ S2 + S3 ) неотрицательно и даже не меньше s, так как 3k - (Sj'+ Sg + S3 ) = (Sx + S2 + S3 + S) - (S{ + Sg + S3) = У троек ay by с тл. а'\ b"y с" совпадают суммы квадратов, так как 3(а2 + Ъ2 + с2) = 3fe = sj' + s£ + s£ + s"= 3(a//2 + b"2 + с"2). Ho sxs2 ^ s'iS2> s3 = S3, s ^ s", значит, s1s2s3s ^ sj's^'Sg's", т. e. квадрат левой части исходного неравенства для тройки а, Ь, с не больше, чем для тройки а", Ь", с". Это означает, что для поиска минимального М достаточно рассматривать только тройки а^Ксс условием Ь — а = с — Ь. Зафиксируем k = а2 + Ь2 + с2 ^ 0. Пусть sx = (b - а)2 = = s2 = (с - Ъ)2 = ty тогда s3 = (с - а)2 = 4t9 s = (а + b + с)2 = = 3k - (sx + s2 + s3) = 3k - 6t9 отсюда квадрат левой части F(t) = ((& -a)(c-b)(a-c)(a + b + с))2 = 12(kts - 2t4). Вычислив производную F'(t) = 12(3kt2 - 8t3) = 12t2(3k - 8t)9 ви- QL QL дим, что F(t) возрастает при t < —, убывает при t > — и о о имеет максимум при t = —. Отсюда F (t) ^ F\ — = . Отсюда F (t) ^ F\ fe, о ^ о j 512 / 81 9л/2 поэтому при М — Л = исходное неравенство верно. V 512 32 С другой стороны, при а = л/2 - 3, Ъ = л/2, с = л/2 + 3 имеем k = 24, t = 9 = —-, и при М = исходное неравен- о 32 ство превращается в равенство. 06.4.1 Ответ. (0; -2), (0; 2), (4; -23), (4; 23). Если х < 0, то 1 < 1 + 2х + 22* + 1 < 1 + 1 + 2 = 4, поэтому решений нет. При д: = 0, 1, 2 левая часть уравнения равна соответственно 4, 11, 37. Точный квадрат получается только при х = 0, что дает пары (0; 2) и (0; -2). Пусть д: ^ 3. Положим у > 0 (пары (jc, 1/) и (jc, -1/) входят в множество решений одновременно). Если у < 2ху то у2 ^ 22х < 1 + 2х + 22х + х, если же у > 2х + 1, то у2 > 22х + 2 = = 22х + 1 + 22х + 22х>\ + 2х + 22х + 1у поэтому возможно только 2х < у < 2х + 1. Преобразуем уравнение к виду (у - 1)(у + 1) = = 2х (2х + 1 + 1). Числа у - 1 и у + 1 оба четные, причем одно из них не делится на 4, значит, другое делится на 2х ~ г и не делится на 2х, т. е. имеет вид 2*-1(2fe - 1) для натурального k. При k = l имеем 2х" l(2k - 1) = 2х" г < 2ху а если Ответы. Решения. Указания 109
k > 3, то 2х ~ г (2k - 1) > 4 • 2х ~ г = 2х + г. Но по доказанному 2х ^ у ± 1 ^ 2х + г9 значит, возможно только k = 2. Остается две возможности: либо у - 1 = 3 • 2х ~ г9 либо у + 1 = 3 • 2х ~ г. Подставляя в исходное уравнение у = 3 • 2х ~ l ± 1, получаем 1 + 2х + 22х + 1 = 9 • 22х~2 ± 6 • 2*"1 + 1, 2* + 22x + l = = 8 • 22х ~ 2 + 22х ~ 2 ± 3 • 2х, 2х = 22х ~ 2 ± 3 • 2х. В одном случае (знак «+») получаем 22х~2 + 2 • 2х = О, что невозможно, в другом случае (знак «-») 4 • 2х = 22х " 2, 2х + 2 = 22х - 2у х + 2 = 2х - 2, jc = 4 и у = ±23. 06.5.1 Нам понадобятся вспомогательные утверждения (леммы). Лемма 1. Если Т(х) — многочлен с целыми коэффициентами, а у и z — различные числа, то Т(у) - T(z) делится на у - z. Доказательство. Действительно, если Т(х) = atxl + а,.!*1"1 + ... + ахх + а0, то Т(у) - T(z) = at(y' - zl) + щ.М-1- г1'1) + ... + ax{y - 2), и каждое из слагаемых делится на у — z. Лемма 2. Пусть а19 а2, ..., aL — различные числа, I ^ 2, и пусть Ь19 Ь29 ..., Ь{ таковы, что для всех 1 ^ i < j ^ I выполнено равенство \аь - ау-| = \bt - 6у-|. Тогда найдется такая линейная функция f(x) = ±х + с, что /(af) = b^ для всех £ = 1, ..., /. Доказательство. Предположим, что в равенстве \at — a,j\ = \bt — bj\ для двух пар индексов (г, s) и (s, t) модуль раскрывается с разным знаком, т. е. пусть ar — as = = br- bs и as- at = bt - bs. Складывая, получаем ar- at = = br+bt-2bg. Ho ar- at = ±(br- bt)> откуда bs = br или bs = bt — противоречие. Из доказанного вытекает, что если аг — а2 = Ьг — Ь2, то для i = 3, 4, ..., I выполнено ах — ai = b1 — bi9 и далее, для всех 1 ^ i < j ^ I выполнено at - ay = bt- by, значит, можно взять f(x) = x + (Ьг — аг). Если же аг — а2 = b2 — bl9 то для i = 3, 4, ..., I выполнено аг — at = bt — bl9 и далее, для всех 1 ^ i < j ^ I выполнено at- а}- Ъ} - biy и можно положить f{x) = -х + (аг + Ьг). Леммы доказаны. Перейдем к решению задачи. Обозначим Qt(x) = = P(P(...P(P(jc))...)), где Р применено I раз. Предположим, что для некоторого k найдутся п + 1 таких различных целых чисел tl9 t2, ..., tn + 1> что Qk(ti) — ti для i=l9 2, ..., п + 1. Тогда tt — tj = Qk(tj) — Qk(tj) и согласно лемме 1 (h-tj) : (Qb-M-Qk-Atj)) : ... ■ (Р(^)-P(tj)) : (tt-tj). 110 Ответы. Решения. Указания
Отсюда \tt — tj\ = \P(tt) — P(tj)\9 значит, по лемме 2 найдется такая линейная функция f(x), что f(tD = P(ti) для всех i = 1, 2, ..., п + 1. Но отсюда следует, что многочлен Р(х) - f(x) степени п имеет п + 1 корней tl9 t2, ..., £n + i — противоречие. 06.6-1 В реп1ении будет использовано понятие суммы Мин- ковского двух выпуклых многоугольников и неравенство Брунна — Минковского (см., например: Васильев Н. Сложение фигур // Квант. — 1976. — № 4). Пусть граница многоугольника Р (при обходе против часовой стрелки) составлена из векторов ~р[, р^, ..., р^~, и при проектировании на прямую ll9 перпендикулярную р^, многоугольник Р перейдет в отрезок длины hx. Ясно, что hx равно полусумме длин проекций векторов р^, р^, ..., р^" на прямую 1г (рис. 41) и -^- — площадь, сопостав- ленная стороне рг. Пусть Р' — многоугольник, полученный из Р центральной симметрией; его граница (при обходе против часовой стрелки) составлена из векторов р[ = —р^, р'2 = —р^-> ..., р'п = -р^ (см. рис. 41). Рассмотрим сумму Минковского многоугольников Р и Р\ т. е. многоугольник Q, граница которого составлена из векторов ~р[, р[, р^, р2, ..., р^~, р'п, взятых в таком порядке, чтобы многоугольник Q оказался выпуклым. В нем стороне рх (и стороне р[) сопоставлена площадь ——-, где Нг равно полусумме длин проекций А векторов У1У р[, р^", р2У ..., р^, р'п на прямую ll9 т. е. Нх = 2hx. Аналогично рассматривая все стороны, получаем, что сумма А(Р) площадей, соответствующих сторонам многоугольника Р, в 4 раза меньше суммы A(Q) площадей, соответствующих сторонам многоугольника Q: р2Н2 р'2Н2 РпНп р'пНп - + -^- + -ir+ ...+ — + — = = 2^/^! + 2p2h2 + ... + 2pnhn = 4Л(Р). Многоугольник Q имеет центр симметрии О. Соединив О с вершинами, разобьем Q на треугольники. Из симметрии следует, что высота треугольника, отвечающего стороне рг (или р[)у равна —. Складывая площади всех тре- Ответы. Решения. Указания 111
Рис. 41 угольников, получаем, что площадь S(Q) многоугольника Q равна р[Нх р2Н2 р'2Н2 РпНп р'пНп + + + + + + Для завершения решения достаточно установить, что S(Q) ^ 4S(P). Но это неравенство получается из применения к многоугольникам Р и Р' неравенства Брунна — Мин- ковского: если два выпуклых многоугольника имеют площади Sx и S2, а их сумма Минковского — площадь S, то 112 Ответы. Решения. Указания
2007 год __. J9ZJLJ а) Пусть d совпадает с числом dt (I ^ Z ^ я). Найдем такие & е {1, ..., /}, те{/+1, ..., л}, что аЛ = тах{а15 ..., аД, ат = min{az, ..., ап}; тогда d = ak — am. Предположим, что \xk ~ ak\ < - и \хт - ат\ < -; тогда (хм - ат) - (xk - ak) < d. Но с другой стороны, (*т ~ ат) ~ (xk - ak) = (ak - ат) + (хт - хк) = — fi А- (у — У \ — и, -г \л,т -^kh что не меньше d, так как т^ к. Полученное противоречие показывает, что xk-ak\, \xm-am\}^d. б) Положим xt = max{al9 ..., а£} - —; легко видеть, что хг ^ х2 ^ ... ^ л:л. Докажем, что \xt — at\ ^ — для любого i е {1, 2, ..., л}. Из пункта «а» тогда будет следовать, что max{|x£ — aj | 1 ^ i ^ /г} = —. Предположим противное: пусть для какого-то I выпол- i i d нено неравенство \хг - at\ > —. Сл Случай 1. Пусть хь - аь >—. Тогда max{a!, ..., at) - - - at > - =* d < max {al7 ..., at) - at ^ ^ max {aly ..., at) - min {az, ..., an) = dt. Противоречие. Случай 2. Пусть xz — at < —. Тогда maxja!, ..., az} at < — => maxjaj, ..., aL} < at. Противоречие. 07.2.\ Введем обозначения: ZCDE = ZCBE = ф (четырехугольник BCED — вписанный), EG = EC = EF = x (радиусы окружности с центром Е), ZCGE = ZGCE = a, ZFCE = = ZCFE = В (рис. 42). Заметим, что AAFD ™ AGFC, так как AD || GC, поэтому ^ = ^ =* g = |g. Далее, ZCEB = = a — ф > О (ZGCE — внешний для треугольника СЕВ), поэтому по теореме синусов для треугольника СЕВ запишем 8-Агаханов Ответы. Решения. Указания 113
Рис. 42 х ВС = sin (а - ш). А из равнобедренного треугольника sincp CEG запишем CG = 2xcosa. Аналогично из треугольника DEF следует, что FD = —^— sin(p - ф), а из треугольника sincp CEF следует, что CF = 2л: cos р. Значит, ВС _ sin (a - ф) FD _ sin(P - ф) GC 2cosasn^' FC ' sin (а - ф) sin(p - ф) тт Л п ~ о п откуда — = п > Но 0 < a < —, 0 < р < —, no- cos a cosP 2 2 этому 0 < a — ф < —, 0 < Р — ф < —. Если предположить, что а > р, то cos a < cosp и sin (a - ф) > sin(p - ф), откуда sin (a - ф) sin(P - ф) cos a cos P Аналогично предположение о том, что a < p, сводится к противоречию. Значит, a = р. Тогда треугольники CEG и CEF равны, откуда CF = CG, т. е. треугольник CFG — равнобедренный. Отсюда ZBAG = ZCFG = ZCGF = ZDAG, т. е. AD — биссектриса угла BAD. 07.3.1 Рассмотрим клику G наибольшего размера | G \ = 2п. Схема решения такова: мы распределим участников по двум комнатам, а затем будем переводить по одному из второй в первую, пока не достигнем требуемого результата. Через Gl9 G2 будем обозначать части клики G в первой и второй комнатах, а через kl9 k2 — наибольшие размеры клик в этих комнатах. 114 Ответы. Решения. Указания
В течение всего процесса будут выполняться два условия: *i ^ k2; (1) каждый человек в первой комнате дружит со всеми из G2. (2) Из условия (2) следует, что кг = \GX\ = 2п — |G2|. Действительно, если бы в первой комнате нашлась клика размером больше |GX|, to вместе с G2 она бы образовала клику размером больше 2п, что невозможно. Приступим к реализации схемы. Сначала произвольно разобьем G на группы Gl9 G2 по п человек и поместим их в соответствующие комнаты. Добавим в первую комнату всех, кто дружит со всеми из G2, всех остальных отправим во вторую. Очевидно, условия (1) и (2) выполнены {h2>\Gi\ = n = \Gl\ = k1). Пока к2 — кг^ 2, будем переводить в первую комнату любого из G2 (если с самого начала к2 — кг < 2, то этот этап не выполняем). При этом кг растет на 1, к2 уменьшается не более чем на 1, поэтому к2 — кг уменьшается, но остается неотрицательным. Очевидно, выполняется и условие (2). Наконец мы достигаем того, что к2 - кг равно 0 или 1. Если к2 — кг = 0, то задача решена, поэтому пусть к2 — кг = 1. Если во второй комнате есть клика К размера к2 и х е G2\K, то переведем х в первую комнату. Число кг увеличится на 1, число к2 не изменится, и задача решена. В противном случае каждая клика К размера к2 целиком содержит G2. Но тогда к2> кг^ n^\G2\, л найдется х е K2\G2. Очевидно, х дружит со всеми из G2; переведем его в первую комнату. При этом кх не изменится, поскольку условие (2) выполнено, a G2 не изменилось. Если к2 уменьшилось на 1, то задача решена. Иначе повторим эту же операцию. До бесконечности мы повторять ее не можем, значит, в какой-то момент к2 уменьшится и станет равным kl9 что и требовалось. 07,4,1 Если точки Р и Q совпадают, то Р — точка пересечения серединных перпендикуляров к сторонам АС и БС, поэтому биссектриса CR угла ВСА является серединным перпендикуляром к АВ. Тогда равенство SPRK = SQRL следует из симметрии треугольников относительно CR. Пусть точки Р и Q не совпадают. Прямые РК и LQ пересекаются в центре О окружности, описанной около треугольника ABC (рис. 43). Из подобных прямоугольных треугольников CLQ и СКР следует, что РК - CQ = QL - СР, Z.CQL = ZCPK. Далее, треугольник OPQ равнобедренный, поэтому точки Р и Q симметричны относительно точки М, являющейся основанием перпендикуляра, опущенного из центра О на 8* Ответы. Решения. Указания 115
Рис. 43 CR. Точка М — середина отрезка СД, поэтому СР = QR, CQ = PR. Отношение PK -PR SQRL sin ZRPK площадей треугольников равно PK-CQ- sinZCPK QL-QR- sin ZRQL QL-CP- sin ZCQL = 1. „QZJLl Если (4a2 — l)2 делится на 4ab — 1, то (4a2 — I)2 — - 2 (4a2 - l)(4ab - 1) + (4afe - I)2 = ((4a2 - 1) - (4ab - I))2 = = (4a)2 (a - b)2 делится на 4ab - 1. Поскольку числа 4ab - 1 и 4a взаимно просты, получаем, что (а — Ъ)2 делится на 4а& — 1, т. е. (a-b)2 = k(4:ab- 1) (1) для некоторого целого k. При k = 0 получаем а = Ь. Докажем, что при фиксированном k > 0 равенство (1) неразрешимо в натуральных а, Ь. Пусть это не так. Тогда рассмотрим все пары натуральных чисел (а, Ь), для которых выполнено равенство (1) (очевидно, для таких пар а Ф Ь), и выберем из них пару (а0, Ьо) с наименьшим Ъ = Ьо; в силу симметрии равенства (1) оно также выполнено для пары (Ьо, а0), поэтому Ьо < а0 = tb0 для некоторого действительного t > 1. Перепишем равенство (1) для a = a0, Ь = b0 в виде а2 - (2 + 4ft)a0b0 + (Ь2 + Л) = 0. (2) 116 Ответы. Решения. Указания
Из равенства (2) следует, что уравнение х2 - (2 + 4k)b0x + + (&q + k) = О имеет целый корень хг = а0, значит, оно имеет и второй целый корень, который в силу теоремы Виета равен х2 = — > 0. Пара (Ьо, х2) удовлетворяет равенст- а0 Ь2 + k ву (1), поэтому х2 > Ьо в силу выбора Ьо. Получаем — > &0, а0 откуда k > bQ(a0 - b0) = bg (f — 1). Подставляя а = tb0, b = b0 в равенство (1), получаем b*(t - I)2 = Ь(МЪ\ -\)>b\(t- l)(4*bg - 1) => -1^1 >4fe. Противоречие. 07,6-1 Ответ. Зп. Примером Зп соответствующих плоскостей, объединение которых содержит данное множество S, могут служить плоскости, задаваемые уравнениями вида х = 1, х = 2, ..., X = п, у = 1, у = 2, ..., у = п, z = 1, z = 2, ..., 2 = п. Очевидно, что каждая точка множества S попадет хотя бы в одну из этих плоскостей, ибо имеет хотя бы одну ненулевую координату. Теперь осталось показать, что объединением плоскостей, количество которых меньше Зп, множество S покрыто быть не может. Проведем доказательство от противного. Пусть множество S может быть покрыто объединением плоскостей, количество которых менее чем Зп. Запишем уравнения этих плоскостей: агх + Ъху + сг2 + dx = 0, а2х + b2y + c2z + d2 = 0, akx + bky + ck2 + dk = 0. Здесь, во-первых, k < Зп и, во-вторых, ни одно из чисел dl9 d2y ..., dk не равно 0, иначе соответствующая плоскость проходила бы через точку (0, 0, 0). Перемножив левые части уравнений этих плоскостей, получим многочлен Q(x, i/, 2) от трех переменных, обращающийся в нуль при подстановке вместо (jc, у, 2) координат любой точки множества S. Отметим, что Q(0, 0, 0) = = dj • d2 - ... • dk Ф 0, а степень многочлена Q(x, у, 2) меньше Зп. Рассмотрим теперь многочлены #i (х) = х (х — 1)... (х — п), 1 ) () ( 1) ... (2-п). Каж- Ответы. Решения. Указания 117
дый из них обращается в нуль во всех точках множества S (эти многочлены можно рассматривать как многочлены трех переменных, степени вхождения двух из которых только нулевые, и в этом качестве подставлять в них координаты точек множества S). Разделим с остатком многочлен Q(x, у, z)> рассмотренный как многочлен от х, на многочлен gx(x). Получим равенство Q(x, у, z) = gx(x) • Аг(х, у, z) + Qx{x, y, z). При этом, во-первых, степень вхождения переменной х в многочлен Qx(x, z/, z) будет не больше п, во-вторых, Qx(x, у, z) обращается в нуль во всех точках множества S (поскольку во всех точках множества S обращаются в нуль многочлены Q(x, уу z) и g1(x)) и, в-третьих, Qj(x, z/, z) не обращается в нуль при х = у = z = 0 (поскольку Q(0, 0, 0) = = Qi(0, 0, 0)). Аналогично получим равенство Qi(x, У, z) = g2(y)A2(x, i/, z) + Q2(x, i/, z), где многочлен Q2(x9 i/, z) обращается в нуль во всех точках множества S, причем степень вхождения каждой из переменных х и у в многочлен Q2(x, у, z) не превосходит п и Q2(0, 0, 0)* 0. Наконец, получаем равенство Q2(x, yy z) = gs(z) • As(xy yy z) + Т(ху у, z), где многочлен Г(х, i/, z) обращается в нуль во всех точках множества S и степень вхождения в многочлен Т(х, у, z) каждой из переменных х, у и z не превосходит п, причем Т(0, 0, 0) ^ 0. Тем самым Т(х, у, z) не является тождественно нулевым многочленом. Рассмотрим теперь одночлен axsylzu (а Ф 0) наибольшей степени s + t + и многочлена Т(х, у, z). Одно из чисел s, t, и должно быть строго меньше п, так как степень многочлена Т(ху у у z) не превосходит степени многочлена Q(x, уу z), которая, в свою очередь, меньше Зп. Не умаляя общности, пусть s < п. Имеет место следующая теорема (см., например, статью «Combinatorial Nullstellen satz» в книге «Задачи Санкт-Петербургской олимпиады школьников по математике 2005 года»): Теорема. Пусть P(x,y,z) — ненулевой многочлен от трех переменных с вещественными коэффициентами, одночлен наибольшей степени которого имеет вид axsyfzu (а Ф 0). Пусть Л, Б, С — конечные множества вещественных чисел, причем в А содержится не менее s + 1 элементов, в В содержится не менее t + 1 элементов, в С содержится не менее и + 1 элементов. Тогда существуют числа х0 е Л, у0 е Б, 20 е С такие, что Р(х0; у0; z0) Ф 0. 118 Ответы. Решения. Указания
Применив указанную теорему к многочлену Т(х, у, г) и множествам А = {1, 2, ..., п}9 В = {О, 1, ..., п}9 С = {О, 1, ..., п}9 приходим к противоречию, поскольку в силу построения многочлена Т(х, у, z) он обращается в нуль, если одновременно х е А, у е В и z e С. 2008 год 08.1.1 Пусть Ло, Бо, Со — середины сторон ВС, СА, АВ соответственно. Пусть данные окружности с центрами в Во и Со пересекаются в точках А' и Н (рис. 44). Так как А'Н _L В0С0, В0С0 || ВС и АН _1_ ВС, то точка Л' лежит на прямой АН. Далее, по теореме о произведении отрезков секущих запишем: АС9 = АН АВг • АВ2 = АН • АА' и АСг откуда АВг - АВ2 = АСг - АС2* АА'9 Рис.44 ~ Л1 Ао Л2 ^ Это означает, что точки В19 Б2, С19 С2 лежат на одной окружности со. Центр О окружности со лежит на перпендикуляре к ВгВ2, проходящем через Бо, т. е. на серединном перпендикуляре к отрезку С А. Аналогично О лежит на серединном перпендикуляре к отрезку АВ. Отсюда следует, что О — это центр окружности, описанной около треугольника ABC. Таким же образом докажем, что точки Al9 A2, Cl9 C2 лежат на окружности со' с тем же центром О. Так как окружности со и со' имеют один и тот же центр и проходят через точку Al9 то они совпадают. Это означает, что указанные в условии шесть точек лежат на одной окружности. Ответы. Решения. Указания 119
08.2,1 Сделаем замену: х у , z = а, —?— = Ь, = с, у - 1 z - 1 где а, Ь, с не равны 1. Заметим, что х, i/, 2 однозначно выражаются через а, Ь, с: _ а _ Ъ _ с а - 1' Ь - 1' с - 1" Условие xyz = 1 переписывается в виде « 2(а + Ь + с - 1) = (а + Ъ + с)2 - (а2 + Ь2 + с2) « « а2 + Ь2 + с2 - 1 = (а + Ь + с)2 - 2 (а + Ь + с) + 1 « 2-й о о -g / I L I ■< \9 « , „ _1_ Л^ -I- /^*^ I ^z I /Т -4- г) -\- (* I 1 Отсюда вытекает нужное неравенство а2 + Ь2 + с2 ^ 1. Кроме того, из приведенных выкладок ясно, что если а ^ 1, Ь ^ 1, с ^ 1 иа + Ь + с—1 = аЬ + Ьс + са, то неравенство обращается в равенство при условии а + Ъ + с = 1. Иначе говоря, условие обращения неравенства в равенство для чисел а Ф 1У 6^1, с ф! эквивалентно системе [а + b + с = 1 Гс=1-а-Ь Ъ + Ъс + са = 0 [а + Ь — а2 — аЬ — Ъ2 = 0. Положим Ь = Л,а и подставим в уравнение а + Ь — а2 — аЬ — - Ь2 = 0. Получим а(1 + А,-а(1 + ?1 + ?12)) = 0. Отсюда следует, что при любом А, тройка чисел 1+ А _ X + X2 -X удовлетворяет системе. Если в качестве X брать натуральные числа, то, как легко видеть, мы получаем бесконечное количество различных троек рациональных чисел а, Ь, с, отличных от 1. Соответственно этим тройкам чисел а, &, с соответствует бесконечное количество различных троек рациональных чисел х, I/, 2, удовлетворяющих условию. Ов.3-1 Пусть N > 20 — натуральное число. Пусть р — простой делитель числа (NI)2 + 1. Очевидно, что p>N. Пусть г — остаток от деления N1 на. р. Положим п = г, если г < —, и п = р - г, если г > —. Тогда п2 + 1 = (±г)2 + 1 = = (ЛГ!)2 + 1 = 0 (mod р)ип<^. Докажем, что число п удовлетворяет условию. Положим п = — , где х> 0 — нечетное число. Тогда 4 (п2 + 1) = 120 Ответы. Решения. Указания
= (р — х)2 + 4 = р2 — 2рх + х2 + 4 делится на р, а значит, jc2 + 4 делится на р. Отсюда л;2 + 4 ^ р, следовательно, х ^ д/р - 4, в частности, jc > л/20- 4 = 4. Имеем 2п + д/гТг = р - jc + yjp - х < р - х + д/р - 4 ^ р, что и требуется. Наконец, заметим, что из приведенных рассуждений вытекает, что га2 + 1 ^р > N, поэтому п > yJN - 1, значит, натуральных чисел п с требуемым свойством бесконечно много. 08.4.1 Ответ. f(x) = х и f(x) = —. Пусть w = x = y = z = a>0. Тогда из исходного уравнения получаем ———-f- = ^-—^- = 1, откуда для всех а > О 2f(az) ar + а2 выполнено равенство (Да))2 = Да2), (1) в частности, (/(I))2 = /(1) и /(1) = 1, так как /(1) Ф 0. Далее, положим w = х = а > 0, у = 1, 2 = а2: 2а2 Согласно равенству (1) /(а4) = (/(а2))2 = (/(а))4, откуда - . Положив f = /(а), имеем а2 » (а2*2 - 1)(а2 - t2) = 0. (/(а))4 + 1 а4 + 1 2*2 2а2 t4+ I ~ а4+ 1 Так как а > 0 и f > 0, то для каждого а > 0 выполнены равенства £ = /(а) = а или £ = /(а) = —. Предположим, что нашлись положительные а и Ь, не равные 1 и такие, что f(a) = a, f(b)=-. Тогда, положив Ь w = a, jc = b, у = ab, z — 1, получим (с учетом равенства (1)) равенство Если f(ab) = ab, то из равенства (2) следует: что противоречит предположению. Ответы. Решения. Указания 121
Если же f(ab) = —, то из равенства (2) следует: ао а2 + — Ь2 а2 л- Ь2 a2b2 а2 + Ь2 - _J_ I + a2b2 b2 I + a2b2 a2b2 => a4b2 + a2 = a2 + b2 => b =0 или a4 = 1, что противоречит предположению. Таким образом, либо для всех х > О выполнено равенство f(x) = х, либо для всех х > О выполнено равенство f(x) = —. Непосредственная подстановка показывает, что 1 функции /(jc) = х и /(jc) = — подходят. X 08-5.1 Ответ. 2*-". По условию М равно количеству последовательностей X = x1x2.-.xk, в которых каждый символ хь равен одному из чисел 1, 2, ..., л и каждое из чисел 1, 2, ..., п встречается нечетное число раз (такие последовательности назовем последовательностями I типа). Число N равно количеству последовательностей У = = yiy2.-.yk> B которых каждый уь равен одному из чисел 1, 2, ..., 2п, причем каждое из чисел 1, 2, ..., п встречается нечетное число раз, а каждое из чисел га+1, га + 2, ..., 2п встречается четное число раз (такие последовательности назовем последовательностями II типа). Каждой последовательности А - аха2...ак II типа поставим в соответствие последовательность В — ЬгЬ2...Ьк, в которой bt = at, если а^ п, и bt = at — п, если at > п. Ясно, что определенная таким образом последовательность В — последовательность I типа. Далее мы установим, что каждая последовательность I типа поставлена в соответствие ровно 2к~п последовательностям II типа — этим завершится решение. Для этого посчитаем число способов восстановить последовательность А II типа по соответствующей последовательности В I типа. Пусть число i встречается в последовательности В ровно 1Ь раз (Z; — нечетные числа, и 1Х + 12 +... + /„ = k). Тогда в последовательности А по сравнению с В четное количество чисел 1 заменено на п + 1; количество вариантов такой замены равно — = 2'1"1 (поскольку количество всех подмножеств множества из 1Х элементов равно 2^, все под- 122 Ответы. Решения. Указания
множества можно разбить на пары так, что в каждую пару входит подмножество вместе со своим дополнением и в каждой паре ровно в одном из двух подмножеств четное число элементов). Аналогично имеется 2/2~ * вариантов замены чисел 2 на п + 2 и т. д., 2Z«~ * вариантов такой замены п на 2п. Замены производятся независимо; таким образом, по последовательности В последовательность А восстанавливается 2*i-i.2*2-i.... - 21*"1 = 2/1+/2+-+/п-л = 2Л"Л способами. 08.6.j Пусть Jf, L, М, Af — точки касания окружности со с прямыми АВ, ВС, CD, DA соответственно. Пусть окружности cOj и со2 касаются отрезка АС в точках Р и Q соответственно. Пусть со3 и со4 — вневписанные окружности треугольников ABC и ADC, касающиеся отрезка АС в точках Qx и Рх соответственно (рис. 45). Рис. 45 Ответы. Решения. Указания 123
Из условия АВ ^ ВС вытекает, что точки Р и Qx не совпадают. Из равенства отрезков касательных имеем АВ + AD = ВК - АК + AN - DN = ВК - DN = = BL- DM = BL-CL + CM - DM = CB + CD. „ AD AC + AB-BC AD AC + CD- AD Так как АР = и АРХ = , получа- ем АР — АР19 и поэтому Р = Рг. Аналогично Q = Q1# Пусть ТТ'у РР'у QQ' — диаметры соответственно окружностей со, со1? со2, проведенные перпендикулярно прямой АС (пусть точки ГиГ обозначены так, что Т ближе к прямой АС, чем Т'). Касательные к окружностям со, со1? со2, проведенные через точки Т, Р', Q' соответственно, параллельны прямой АС. Окружности со, со1? со3 гомотетичны с центром В, поэтому соответственные точки Г, Р', Q этих окружностей лежат на прямой, проходящей через точку В, т. е. точки Б, Р', Q, Т лежат на одной прямой. Также, поскольку окружности со, со2 и со4 гомотетичны с центром D, точки D, P, Q', Т лежат на одной прямой. Из доказанного следует, что существует гомотетия h с центром Г, переводящая точку Q в точку Р'. Заметим, что QQ' \\ Р'РУ а прямая PQ' проходит через точку Т и отлична от прямой P'Q, поэтому под действием гомотетии h отрезок QQ' переходит в отрезок РГР. Гомотетия h переводит окружность со2, построенную на отрезке QQ' как на диаметре, в окружность, построенную на отрезке Р'Р как на диаметре, т. е. в окружность сох. Тогда центр Т гомотетии h принадлежит общим внешним касательным окружностей сох и со2. Таким образом, точка Т на окружности со и является точкой пересечения общих внешних касательных к окружностям сох и со2. Ответы. Решения. Указания
Список обозначений => — следовательно <=> — равносильный переход R — множество действительных чисел Q — множество рациональных чисел Z — множество целых чисел N — множество натуральных чисел Р — множество простых чисел 0 — пустое множество а е А — элемент а принадлежит множеству А а & А — элемент а не принадлежит множеству А {а е А | X) — подмножество элементов а множества А, удовлетворяющих условию X \А\ — количество элементов конечного множества А В а А — множество В является подмножеством множества А A U В — объединение множеств А и В А П В — пересечение множеств А и В А\В — разность множеств А и Б (т. е. множество, содержащее все такие элементы множества А, которые не принадлежат множеству Б) f:A—>B — функция /, определенная на множестве А, значения которой принадлежат множеству Б п ^at — сумма чисел а19 а2, ..., ап п Y\at — произведение чисел а19 а2, ..., ап [х] — целая часть действительного числа х, т. е. наибольшее целое число, не превосходящее х {х} — дробная часть действительного числа х; {х} — х — [х] тах{а1? а2, ..., ап} — наибольшее из чисел а19 а2, ..., ап min{al9 а2, ..., ап} — наименьшее из чисел а19 а2, ..., ап а • Ъ или Ь\ а — а делится на Ъ (или Ъ делит а) а ХЬ — а не делится на Ь а = b (mod п) — а сравнимо с Ь по модулю п (т. е. целые числа а и Ъ дают равные остатки при делении на п) Список обозначений 125
НОД(а, Ь) (или (а, Ь)) — наибольший общий делитель чисел а и Ъ а1а2... ап — десятичная запись числа (а19 а2, ..., а>п — цифры) ААВС — треугольник ABC A ABC °° АА'В'С — треугольник ABC подобен треугольнику А'В'С ZABC — угол ABC а\\Ъ — прямая а параллельна прямой Ъ а _1_ Ь — прямая а перпендикулярна прямой Ъ Z(a, Ъ) — ориентированный угол между прямыми а и Ь, т. е. угол от прямой а до прямой Ь, отсчитываемый против часовой стрелки а, АВ или а, АВ — векторы AC (ABC) — величина дуги АС (величина дуги АС, на которой лежит точка Б) S(M) или SM — площадь многоугольника М п\ — n-факториал, произведение п первых натуральных чисел, п\ = 1 • 2 • ... • п Скп — число сочетаний из п по k, т. е. количество ft-эле- ментных подмножеств гс-элементного множества, deg / — степень многочлена f 126 Список обозначений
Рекомендуемая литература Международные математические олимпиады (1959—1996 гг.) Морозова Е. А., Петраков И. С, Скворцов В. А. Международные математические олимпиады. — М.: Просвещение, 1976. Международные математические олимпиады / Сост. Фомин А. А., Кузнецов Г. М. — М.: Дрофа, 1998. Статьи о международных олимпиадах в журнале «Квант» XXXIX Международная математическая олимпиада. — Квант. — 1999. — № 2. — С. 49. XL Международная математическая олимпиада. — Квант. — 2000. — № 2. — С. 46—47. XLI Международная математическая олимпиада. — Квант. — 2001. — № 2. — С. 48—49. XLII Международная математическая олимпиада. — Квант. — 2002. — № 2. — С. 44—46. XLIII Международная математическая олимпиада. — Квант. — 2003. — № 2. — С. 46—50. XLIV Международная математическая олимпиада. — Квант. — 2004. — № 2. — С. 48—51. XLV Международная математическая олимпиада. — Квант. — 2005. — № 2. — С. 49—53. XLVI Международная математическая олимпиада. — Квант. — 2006. — № 2. — С. 49—51. XLVII Международная математическая олимпиада. — Квант. — 2007. — № 2. — С. 48—51. XLVIII Международная математическая олимпиада. — Квант. — 2008. — № 2. — С. 49—51. XLIX Международная математическая олимпиада. — Квант. — 2008. — № 6. — С. 39—41. Рекомендуемая литература 127
Содержание Введение 3 Задачи Международных математических олимпиад ... 33 Ответы. Решения. Указания 48 Список обозначений 125 Рекомендуемая литература 127 Учебное издание Серия «Пять колец» Агаханов Назар Хангельдыевич Кожевников Павел Александрович Терешин Дмитрий Александрович МАТЕМАТИКА МЕЖДУНАРОДНЫЕ ОЛИМПИАДЫ Зав. редакцией Т. А. Бурмистрова Редактор Л. Н. Белоновская Младшие редакторы Е. А. Андреенкова, Е. В. Трошко Художники О. П. Богомолова, И. В. Калинина Художественный редактор О. П. Богомолова Компьютерная графика И. В. Губиной Технический редактор и верстальщик А. Г. Хутпоровская Корректоры А. К. Райхчин, И. П. Ткаченко Налоговая льгота — Общероссийский классификатор продукции ОК 005-93— 953000. Изд. лиц. Серия ИД № 05824 от 12.09.01. Подписано в печать с оригинал-макета 25.08.09. Формат 60X90 х/16. Бумага писчая. Гарнитура Школьная. Печать офсетная. Уч.-изд. л. 5,93. Тираж 3000 экз. Заказ № 23484 ш-sm). Открытое акционерное общество «Издательство «Просвещение». 127521, Москва, 3-й проезд Марьиной рощи, 41. Открытое акционерное общество «Смоленский полиграфический комбинат». 214020, г. Смоленск, ул. Смольянинова, 1.